The status quo of electric cars: better batteries, same range

This is a guest post by Kris De Decker. It was previously published by Low Tech Magazine.


Columbia electric car 1902 (picture credit)

Electric motors and batteries have improved substantially over the past one hundred years, but today's much hyped electric cars have a range that is - at best - comparable to that of their predecessors at the beginning of the 20th century. Weight, comfort, speed and performance have eaten up any real progress. We don't need better batteries, we need better cars.

From about 1895 to the mid-1920s, and following the bicycle craze of the 1890s, electric cars shared the road with petrol and steam powered cars. EV's were comparatively slow, heavy, and had a smaller range than their alternatives. During the very early years, however, electric automobiles were the most popular option for a short time, mainly because of two reasons.

Firstly, they were easy to start, while a gasoline car had to be cranked up and a steam powered car required a long firing-up time (not unlike a wood gas car). Secondly, there were few paved roads outside the city at the turn of the 20th century, which made the limited range of EV's not that problematic. The production of electric vehicles peaked in 1912: during that time there were 30,000 EV's on the road in the United States, two-thirds of these were used as private passenger cars. Europe had around 4,000 electric vehicles.

By 1912, the gasoline car had already taken over the largest share of the automobile sales (more than 90 percent). They were faster and could drive longer distances - not only because of their better range but also because of a more elaborate refuelling infrastructure. The rapidly expanding paved road network worked in their favour, too.

Internal combustion engines became much cheaper than electrics. In 1908, Ford introduced its mass-produced (and gasoline powered) Model-T, which initially sold for 850 dollars - two to three times less than the price of a similar electric vehicle. In 1912, the price of the Model-T came down to 650 dollars. That same year, the electrical starter for gasoline vehicles appeared, and took away one of the last selling points of EV's. Last but not least, gasoline had become much cheaper than it had been at the end of the 19th century.

The only advantage left was the (potential) cleanliness and noiselessness of electric vehicles, the reason we want them back today. In 1914, Henry Ford announced the marketing of a cheap mass-produced electric vehicle, but this automobile was never produced. In Europe, electric passenger cars were gone in 1920, in the US they survived for a decade longer. Electric trucks, outside the scope of this article, remained successful for a longer period.

The manufacturers of early electric cars made several strategic mistakes. For instance, it took them until 1910 to develop a standard for the charging of the batteries. But, at the heart of the failure of the early electric car lay the limited capacity of the storage battery.

Then and now: 100 miles

If today's supporters of EV's would dig into the specifications and the sales brochures of early 20th century electric "horseless carriages", their enthusiasm would quickly disappear. Fast-charged batteries (to 80% capacity in 10 minutes), automated battery swapping stations, public charging poles, load balancing, the entire business plan of Better Place, in-wheel motors, regenerative braking: it was all there in the late 1800s or the early 1900s. It did not help. Most surprisingly, however, is the seemingly non-existent progress of battery technology. 


The 100 mile Fritchle electric poster

The Nissan Leaf and the Mitsubishi i-MiEV, two electric cars to be introduced on the market in 2010, have exactly the same range as the 1908 Fritchle Model A Victoria: 100 miles (160 kilometres) on a single charge. The "100-mile Fritchle" was a progressive engineering feat for its time, but it was not the only early electric that boasted a 100 mile range. I have only chosen it because its specifications are most complete, and because its range was certified.

The first electric cars (1894 - 1900) had a range of 20 to 40 miles (32 to 64 kilometres), still better than the 20 km "range" of a horse. The average second generation EV (1901 - 1910) already boasted a mileage of 50 to 80 miles (80 to 130 km). The third generation of early electric cars (1911-1920), including larger vehicles that could seat 5 people comfortably, could travel 75 to more than 100 miles (120 to more than 160 km) on a single charge - and this is still the range of electric cars today. (See our overview on early electrics for the specifications of individual vehicles).

100 miles = upper limit

In fact, the range of the Nissan Leaf or the Mitsubishi i-MiEV may be far worse than that of the 1908 Fritchle. The range of the latter was (officially) recorded during an 1800 mile (2,900 km) race over a period of 21 driving days in the winter of 1908. The stock vehicle was driven in varied weather, terrain and road conditions (often bad and muddy roads). The average range on a single charge was 90 miles, the maximum range recorded was 108 miles. (sources:  1 / 2 ).

The range of the Mitsibushi i-MiEV and the Nissan Leaf was tested in a very different manner. On rollers instead of on actual roads, and in a protected environment, but that's not all. Both manufacturers advertise the US "EPA city" range, a test that supposes a 22 minutes drive cycle at an average speed of 19.59 mph (31.5 km/h), including one acceleration to 40 mph (64 km/h) during no more than 100 seconds.

Critics blame today's manufacturers for not displaying the "EPA combined cycle" range, which also includes trips on the motorway (the "EPA highway cycle"). Contrary to vehicles with an internal combustion engine, electric cars are more fuel efficient in cities than at steady speed on a highway - an electric motor uses no energy when it is idling, and regenerative braking works best in city traffic. Darryl Siry, former CMO of Tesla, estimates that the correct range of the Nissan (and other modern electric cars) will be around 70% of the advertised range. That would bring the range of today's electrics to the same level as the 1901 Krieger Electrolette (68 miles).

Even the "EPA combined cycle" figures should be considered as an upper limit. Firstly, with an average speed of 48 mph (77 km/h) the highway tests are outdated. Secondly, the range of a car is also affected by other factors: not only excessive speeding and fast accelerations, but also the use of headlights at night, the use of heating or air-conditioning, the use of other options onboard, driving over hilly roads or in headwinds - or all of these factors combined (the EPA has added new test cycles in 2008 to address these points, but the results are not yet available for the EV's we are talking about).

Some of these factors not only concern today's electrics, but also those of yesteryear. However, the Fritchle's range was tested on varied terrain and in varied weather conditions, which was not the case for the Nissan or the Mitsubishi. Moreover, early electrics had no air-conditioning and few had heating systems - drivers and passengers dressed warm in winter. Mitsubishi warns its clients that the use of the heater might cut the range in half. All in all, the range of a 2010 electric vehicle will be closer to 50 miles (80 km) than to 100 miles (160 km). And that's to be expected from a battery at the beginning of its life - after 5 years, the capacity will be at least 20 percent less.

Better batteries

In spite of this, the 2010 vehicle has a much better battery under the hood than the 1908 vehicle. The Fritchle Electric had lead-acid batteries, like all its contemporaries, with an energy density between 20 and 40 Wh/kg (early 1900 batteries had energy densities of only 10 to 15 Wh/kg). The Nissan and the Mitsubishi have a more powerful lithium-ion battery with an energy density of around 140 Wh/kg.

The Nissan's battery can thus store 3.5 to 7 times more energy for a given weight than an average early electric from about 1910. This could have resulted in a vehicle with a 3.5 to 7 times better range (350 to 700 miles or 560 to 1,130 km), but this is not the case. The technological improvements could also have been translated into a 3.5 to 7 times lighter (and smaller) battery, and consequently a lighter and more fuel efficient vehicle, but this is not the case either.

The battery of the Nissan Leaf is only 1.6 times lighter than the battery of the Fritchle: 220 kg (480 pounds) versus 360 kg (800 pounds). The Nissan vehicle (including the battery) weighs more than the Fritchle: 1,271 kg (2,800 pounds) versus 950 kg (2,100 pounds).

Motor output, speed & acceleration

The most obvious difference between the specifications of the old and new cars is the power of their motors. The 1908 car had a 10 HP motor, the 2010 car has a 110 HP motor. In other words, the Nissan Leaf has the motor output of 11 electric Fritchles. The smaller and lighter Mitsubishi i-MiEV (1,080 kg or 2,400 pounds) has the motor power of 6.5 electric Fritchles.


Columbia Victoria Mark VII

The maximum speed of the Fritchle was 40 km/h (25 mph), the Nissan does 140 km/h (87 mph) and the i-MiEV is not far behind (130 km/h or 81 mph). Acceleration data cannot be compared, but there is no doubt that the 2010 cars will accelerate many times faster (and can climb hills much more easily) than their early 1900 cousins. Today, fast acceleration times are one of the selling points of EV's.

The risks of more powerful electric motors were already recognized in the early 1900s. The Hawkins Electrical guide (1914) states:

"Very quick acceleration is an objectionable feature in electric vehicle design, because a vehicle constructed with this feature puts a heavy overdraft on the battery".

A few years earlier, members of the Electric Vehicle Association of America tried to impose a standard maximum speed of 32 km/h (20 mph) for electric vehicles, because power requirements increased rapidly above that limit. They feared that higher speeds would threaten the all-important range of the automobiles. They did not succeed. Too many manufacturers tried to compete with gasoline cars (and with each other) by designing faster electric vehicles.

A car consumes 4 times more fuel to drive twice the speed, so it seems clear that velocity is the reason why the range of today's electric cars did not improve in spite of better batteries. However, it is more complicated than that. The "EPA-city" range that the modern EV's advertise, is based on an average speed of 20 mph or 31 km/h - below the 25 mph top speed of the Fritchle, and almost exactly the same as the speed at which the vehicle could drive 100 miles on one charge.

While high speeds are definitely a significant factor when considering the real-world range of today's electric cars, it cannot explain the disappointing "official" range. Faster acceleration might play a role, but the EPA-tests described above do not consider aggressive driving either so there must be other factors at play.

Oversized cars & motors

The first is weight. While the battery of the Nissan is lighter than the battery of the Fritchle, the Nissan vehicle including the battery is 321 kg (706 pounds) heavier. Without the battery, the Nissan weighs almost twice as much as the Fritchle: 1,051 kg (2,310 pounds) versus 590 kg (1,300 pounds).


Early electric car chassis

So while batteries became more than 3 times lighter in 100 years time, the weight of the vehicle itself (without battery) doubled. The extra weight of the Nissan already nullifies a significant portion of the progress: a 35 percent higher mass can lead to a 28 percent reduction in range (sources: 1 / 2).

The second factor is directly related to the massive increase in horse power. Electric motors are (generally) most efficient around 75 percent of their rated load. Their efficiency drops dramatically below 25 percent. The Fritchle was most efficient at a speed of around 20 mph. The much more powerful motor of the Nissan Leaf, however, is most efficient at a speed of around 105 km/h, far above the average speed in the tests. Today's EV's consume less energy at low speeds than at high speeds because of other factors, but compared to early electrics with their much less powerful motor they are probably less efficient at speeds of around 20 mph. (source - pdf).

Computers on wheels

The third factor is the electronics. Modern cars have, depending on the model, 30 to 100 microprocessor-based electronic control units onboard (source). These computers add weight but also consume energy in a direct way. Part of this direct energy consumption is not included in the EPA-tests - electronically adjustable windows and mirrors, for example.

However, many other electronics are activated whenever the vehicle is driving. Examples are power brakes, active suspension, safety sensors, dashboard instrumentation and the management of the battery itself (not required for a lead-acid battery but critical for lithium-ion storage technology). All this electrical energy has to be supplied by the battery.

While a higher performance cannot explain the relatively low official range of today's EV's, all factors described above are at least partially a consequence of it. Lower speeds would make most safety-related electronics unnecessary and they would do away with the need for larger motors and batteries which, just like the electronics, add more weight.

Tesla Roadster

Some of you might wonder why I don't compare the 1908 Fritchle to the 2008 Tesla Roadster. This car has a range of 244 miles (393 kilometres), 2.44 times better than the old timer and the modern Japanese cars - and this according to the "EPA combined cycle", not the "EPA city" figures. (Although the "EPA-combined" range advertised by Tesla is of course as much suited for a sports car than the "EPA-city" range is suited for a family vehicle like the Nissan Leaf ).

The Tesla Roadster is less progressive than it seems, though. The battery of the sports car weighs twice as much (450 kg) as the battery of the Nissan (220 kg). Since both batteries have a similar energy density, you don't have to be a rocket scientist to calculate that the heavier battery has about twice the capacity: 53 kWh to be exact, compared to 24 kWh for the Nissan's battery (and 16 kWh for the i-MiEV). Considering the fact that both cars have a similar weight, a 2.5 better range for a battery with more than double (2.2 times) the capacity is far from a revolutionary engineering feat.

Embodied energy of EV batteries

Doubling battery capacity is one way to increase the range of an electric vehicle (see also the Mini E, which sacrifices its rear seat for a larger battery and gets 104 miles), but this option is far from sustainable since it also doubles the amount of energy needed to manufacture the battery. It also doubles the costs, of course. The battery of the $ 109,000 Tesla Roadster sells for $ 30,000, as much as an entire Nissan or Mitsubishi vehicle.

Nobody has investigated how much energy it takes to produce a Tesla Roadster battery, or any other EV battery for that matter, but you can get an idea of it using an online tool from Carnegie Mellon University. Corresponding to these data, $ 30,000 of economic activity in the storage battery sector (including the production of li-ion batteries) equals an energy consumption of 23,222 kWh - that's almost 6 years of electricity consumption by an average British household. The battery has to be replaced after a maximum of 7 years.

These figures suggest that the embodied energy of the battery - not considered in any research paper that investigates the ecological advantages of electric cars - makes up for a substantial amount of the total energy cost of an electric automobile. At the advertised energy use of 21 kWh per 100 miles, 23,222 kWh would take the Tesla 109,938 miles 176,929 km) far. That's almost 30,000 km (18,600 miles) per year, or 80 km (51 miles) per day. The low "fuel" costs are only half the story if the "fuel tank" itself is that energy-intensive.

Today, just like 100 years ago, EV proponents are divided on the question of how to market electric vehicles. Some keep emphasizing the fact that most people never drive further than 30 miles per day - therefore the current batteries are well suited to perform their task. Most cars will be charged overnight, battery charging stations and fast-charging will do the rest.

Others, however, keep hoping for a revolutionary storage technology that will eventually give EV's a similar range to that of gasoline cars. This belief is supported by press releases like this:  "Nanowire battery can hold 10 times the charge of lithium-ion". It is interesting to note that the arrival of such a miracle battery has been "just around the corner" for over 100 years now:

"A large number of people interested in stored power are looking forward to a revolution in the generating power of storage batteries, and it is the opinion of many that the long-looked-for, light weight, high capacity battery will soon be discovered." (source, 1901).

"The demand for a proper automobile storage battery is so crying that it soon must result in the appearance of the desired accumulator [battery]. Everywhere in the history of industrial progress, invention has followed close in the wake of necessity" (Electrical Review, 1901).

Edison himself promised a radical improvement to the lead-acid battery at the turn of the 20th century. It took almost a decade before the Edison battery appeared on the market, and even though it had some advantages over the others, it was very expensive (the price of a gasoline powered Ford Model-T) and far from revolutionary.

The promise of a miracle storage technology reared its head again in the 1960s and 1970s, when electric cars went through a short revival:

"The consensus among EV proponents and major battery manufacturers is that a high-energy, high power-density battery - a true breakthrough in electrochemistry - could be accomplished in just 5 years" (Machine Design, 1974).



The range of most electric (concept) cars in the 1960s and 1970s was considerably lower than that of early 1900 electrics. This was because they were still making use of similar lead-acid batteries, while the cars themselves were already much heavier and more powerful.

Realistic electric vehicles - scenario 1

The miracle battery might one day arrive, but history teaches us not to count on it. What would definitely yield results, on the other hand, is to use existing technology and downsize the car. There are two ways to do this, as was briefly noted above. The first is to go back to early 20th century electric vehicles and equip them with modern batteries. This would extend their range spectacularly, as much as a (not yet existing) nanowire battery could.


Charging early electric car 1909

If you were to put the lithium-ion battery of the Nissan Leaf in the 1908 Fritchle, the vehicle would have a range of about 644 km (400 miles). If you put a lithium-ion battery with the same weight of the Fritchle-battery inside, you get about 700 miles (1,127 km) range. Add to this the fact that we now also have lighter and more efficient motors (and other vehicle parts) and the range will become even greater.

Even with the headlights and the heating on, driving home over windy hills and muddy roads, such a car would give a safe and comfortable range, similar to that of today's gasoline vehicles. Moreover, it would consume less energy: the Fritchle used around 7 kWh/100 km, the Nissan Leaf at least 15 kWh/100 km.

A better range is much more than a convenience for the driver. It would also mean that we need fewer charging and battery swapping stations, which would greatly lower the costs and the embodied energy of the required infrastructure. In short, slower EV's would make EV's a whole lot more likely. Interestingly, we don't even have to streamline them. Early electrics had style, and at low speeds aerodynamics is not an important factor in energy consumption.

Realistic electric vehicles - scenario 2

Of course, slow vehicles with the appearance of a horse carriage will not appeal to everybody. But there is another way. We could also downsize the electric car by designing much lighter and fuel efficient vehicles. This is shown by a concept EV like the Trev. This vehicle's performance is comparable to that of the Nissan Leaf or the Mitsubishi i-MiEV: it has a top speed of 120 km/h (74.5 mph) and it accelerates from 0 to 100 km/h (60 mph) in less than 10 seconds.

However, its battery is almost 5 times lighter (45 kg or 99 pounds) and the vehicle itself (including the battery) weighs only 300 kg (660 pounds). In spite of its higher performance, it consumes as much energy as the Fritchle: 6.2 kWh/100 km, half the fuel consumption of the Nissan. Yet, the range of the Trev is similar to that of the Nissan or the Fritchle: 150 km or 93 miles. The reason is of course that if you design a much lighter vehicle, it will also have a much smaller battery that consequently holds less energy. With gasoline powered automobiles, the potential of weight reduction is much larger.

Nevertheless, a vehicle like the Trev would have almost as much benefits as a Fritchle with a 2010 battery. It would still require an elaborate charging infrastructure, but because of its much smaller battery it would seriously relieve the problem of peak demand: fast-charging could become a realistic option without the need to build hundreds of new power plants. It would also have the substantial advantage of holding a battery that is much less energy-intensive to produce.

We cannot have it all

Of course, there are many more possibilities than the two scenario's outlined here. It would not kill us to drive at speeds of 20 mph, on the contrary, but there is so much potential in downsizing the automobile that we don't have to go all the way back to the early 1900s to get a decent range.


Babcock electric roadster 3

We could tune them up a bit so that they could get 60 km/h or 40 mph (only slightly faster than the 1911 Babcock Electric Roadster pictured above) and accelerate just fast enough to leave a crime scene or flee from a mad elephant.

At 60 km/h or 40 mph a trip of 600 kilometres or 400 miles would take 10 hours, instead of 5 hours at a common motorway speed. This does not sound like the end of the world. It's definitely a whole lot faster than going by foot (120 hours) or by bike (30 hours). We could also equip the Trev with a somewhat larger battery so that it gets a better mileage at the expense of a somewhat lower speed. Or, yet another possibility: keep the Trev like it is but limit its speed to that of the Fritchle.

If we want more speed, we have to sacrifice range. If we want more range, we have to sacrifice speed. If we want to keep the (energy) costs of the charging infrastructure within reasonable limits, we have to sacrifice speed or size. The lesson to be learned here, is that we cannot have it all: range, speed and size. And yet, that's what we are trying to do.

© Kris De Decker (edited by Vincent Grosjean)

Overview of early electric cars : specifications & pictures.

Sources(in order of importance):

End of car culture in the coming decade (PDF)
http://www.crudeoilpeak.com/pdfs/17

Planning Australia's Major Cities (PDF)
http://www.crudeoilpeak.com/pdfs/35

The airwaves are still bombarded with auto advertisements with shiny, happy people. Kill the car.

Yes, the electric car with all the features consumers expect is a pipe dream for the foreseeable future. The author left out another important fact -- charge cycles. Those expensive new batteries can charge/discharge around 1000 times before they lose significant capacity and have to be replaced.

That is about 3 years of daily recharges, what is near enough to the life of the bateries to not being worth of extra consideration.

Unless the battery prices come WAY DOWN, a three year battery life will be a disaster for the consumer anmd the car maker;despite the bad publicity new cars get these days, the better models typically run over a hundred thousan miles without an expensive breakdown.

We have three vehicles on the road, all mid nineties models, American made, all over 150,000 miles, with only one serious repair among the three, and that recent and less than a thousand dollars.

My old Toyota has never had a serious repair, other than a rusted out frame, which I fixed myself, but it will need a new clutch soon.It has over tow hundred and twenty thousand miles already, and I expect it to make it to three hundred thousand easily.

Electric cars are not going to be trouble free,and a five thousand dollar battery problem is not going to be funny.

But hopefully the batteries are going to cost less aND LAST LONGER.

That is about 3 years of daily recharges

OFM, you may have missed that last bit. Unless you are travelling cross-country or using the car as a taxi, you would not normally need a recharge every day for 3 years straight.

I agree that a lot of peple will not need a recharge every day, but it seems likely that the real market for the electric car for the next decade is the person who needs it every day to get to and from work.

There will probably be a only much smaller market for electric cars among people who want one mostly for the environmental benefits rather than the potential savings.

People with longer commutes, near the maximum practical range of the car, will potentially save enough on gasoline to justify the extra cost of the car.

But the shorter the commute, or the less they use the car, the smaller the potential savings.

My elderly Escort averages over thirty mpg,and even if gas were ten dollars a gallon,it would be cheaper to keep it than to trade for an electric, so long as I only drive it occasionally and for short distances.Actually I don't drive it all all at the moment;the insurance for it costs more than the gasoline savings available compared to my elderly compact Toyota pickup , so I just drive my old Toyota pickup exclusively.

And while I agree that in principle electric motors are more dependable than ices, only somebody lacking a hands on acquaintance with the automobile industry will believe that they are going to be trouble free, or cheaper to maintain,overall, than a conventional car, for the first decade or so of mass production.

Any little tiny part of the whole electrical system will be a "dealer only" item, and dealers typically charge about three times on up to five or six times as much for parts as aftermarket equivalents when there are no equivalent aftermarket parts.A headlight damaged in an accident costs from two hundred dollars on up at a dealer;but equally good aftermarket headlights sell for as little as twenty percent of the dealer price if the car is a popular one and aftermarket parts are available.

Furthermore there will be extremely few qualified mechanics, and they will be in dealer shops.Any independents trying to break in will have a hard time getting hold of the necessary diagnostic equipment and software, which will run into several tens of thousands of dollars.With only a very few customers in the early days, an independent shop will have a very hard time justifying the purchase..

The manufacturers will change the specs and the parts used every two or three years because they can save a buck by doing so, and make an additional buck by selling parts and labor at high prices to a captive market.

only somebody lacking a hands on acquaintance with the automobile industry will believe that they are going to be trouble free, or cheaper to maintain,overall, than a conventional car, for the first decade or so of mass production.

The Prius has been much more reliable than conventional cars in it's first decade.

Nick,

I can't speak to the Prius, but my company did buy a 2000 Honda Insight as a test vehicle. Many of the points raised by Mac turned out to be true. While the conventional parts and systems were of typical Honda quality and reliability, the battery and associated electronics were not. Of the hundreds of vehicles we purchased over the years, this car was by far the worst from an economic perspective. I'm still glad we bought it as we learned a lot.

I've not checked the Honda Insight owner's forum in many years. It would be interesting to see how much they have improved in the last 10 years.

I have a 2000 Honda Insight with 130,000 miles on it now. The only issue I ever had was with the battery charger controller, which was replaced for free.

We had two fail, unfortunately both were out of warranty, we tend to put miles on vehicles quickly. :-(

PS. The battery went at about 150,000 miles, just in case you are thinking about future changes.

Will,

I'm 95% sure you already know this website, but just in case: http://www.insightcentral.net/

Yes, from day one, thanks.

How many changes of spark plugs (and wires), oil, oil filters, air filters, antifreeze, etc (not to count the emissions checks) will there be for EVs in the first 100,000 miles? Or after that?

hi Will,

You have a powerful argument in that the routine maintainence of a conventional car isn't cheap.Fifteen or twenty oil changes and a couple of tumeups aren't going to cost Joe Sixpack less thantwelve hundred to two thousand bucks or so.

This said, most people pay unnecessarily high prices for these services, especially in that people interested in self sufficiency can do thier own routine maintainence IF THEY BUY A CAR that is easy to work on.

Actually I am a fan of the electric car and believe they are going to sell a lot sooner , and a lot faster, than most people think.

The typical new car buyer trades frequently, and will mostly not have to worry much about repairs , as they will be warranted.

Ts will hit the fan when they are bought second hand and out fo warranty by people looking to save money.If I am right , the resale valyue will fall off pretty fast unless the price of gasoline is VERY high, Or it is rationed.

duplicate

duplicate

you would not normally need a recharge every day for 3 years straight.

But you might be well-advised to charge as often as possible, because depth of discharge is a very big factor in battery life (the mechanical cycling of the electrodes due to expansion and contraction is a big deal).  Side reactions if the charge level is too low or too high are another issue.

A gently-used battery can last several times as long as one on a tougher cycle.  Example from Optima Yellow Tops:

Do you have a source for that? A123systems batteries last about 5,000 cycles.

The range of the Mitsibushi i-MiEV and the Nissan Leaf was tested in a very different manner. On rollers instead of on actual roads, and in a protected environment, but that's not all. Both manufacturers advertise the US "EPA city" range, a test that supposes a 22 minutes drive cycle at an average speed of 19.59 mph (31.5 km/h), including one acceleration to 40 mph (64 km/h) during no more than 100 seconds.

Here is the EPA LA4 Driving schedule:

Reference: http://www.epa.gov/nvfel/methods/uddsdds.gif

that the Leaf says it can do on a 100-mile per charge basis.

Reference: http://www.nissan-zeroemission.com/EN/LEAF/specs.html

Note the close to 60 mph top speed in the test and the frequent stop and goes. Did the author miss the acceleration to
approximately 60 mph? There are two accelerations to about 40 mph. The are numberous accelerations to around 30 mph. This is a tough driving schedule even for an internal combustion engined car. I think the visual of the test versus
what is said about the nature of the graph sets the author's standards for the rest of the article. What's missing? What's misleading?

When you deal with Electric Vehicle (EV) range, you always have to ask "at what speed?" I have a friend who
drives a lead acid EV. He has a 30/60 - 60/30 rule of thumb. He can drive 30 miles at 60 mph or 60 miles at 30 mph. I
saw the range versus distance for a Tesla and at 30 mph, the range was over 300 miles. I suspect that some of the range loss in these Tesla figures is due to a "lead foot" factor. The Tesla is using the tZero technology. The tZero has been driven over 300 miles on a charge at 60 miles per hour. What would its range be at 30 mph?
Reference: http://www.acpropulsion.com/

This leads me back to ask the question: What were the speeds of those turn of the century vehicles for the quoted ranges? I suspect that they were in the 10 to 20 mph range which was probably quite reckless for the condition of the roads back then. At 17 mph, the Tesla range is close to 400 miles. Driving 17 mph is reckless for today's city's streets. A Tesla driver traveling at that speed would likely be rear ended or run over.

After 9/11, I started to look for alternative transportation. I looked at hydrogen/fuel cells and wondered why the Main Stream Media did not see through or report all the technological barriers that I easily found? I also did not want to trade "eating for driving". When I read about the all electric tZero going 300 miles on a charge using about 150 whr/kg "laptop" batteries, I became interested in the technology. I read that the energy density potential for lithium based batteries were in 1500 to 3000 whr/kg range. At full potential, the tZero would be able to go coast to coast and back again on one charge. Even if only half or one-quarter that potential were realized, that would be a minimum of 750 miles -- at 60 mph.

In viewing a graph of the cycle life of A123 batteries, it showed a cycle life of over 7,000 cycles:


Reference: http://www.a123systems.com/a123/technology/life

How valid is that graph? A123 offers a set of 6 batteries for testing. Their batteries are in the Dewalt 36v cordless tool packs.

An acquaintance told me that when you build bigger and bigger packs, the cycle life decrees. How badly depends on a lot on the nature of the chemistry, tightness of the specs, etc. So is a pack of A123 batteries closer to the author's 1,000 cycles or 7,000 cycles?

The more you deep discharge lead acid batteries, the shorter the cycle life. Below 80% Depth of Discharge (DOD), the cycle life of lead acid batteries drops below 300 cycles. You would need to go below 80% DOD to achieve those 100 miles in those old EVs. Southern California Edison has been running a fleet of Toyota Rav4 Electrics with NiMH packs. The ranges traveled are typically 80 for highway to 120 miles for in-town driving. The packs are regularly discharged to less than 80% DOD. The fleet has over 160K miles on it which would mean approximate cycle lifes of over 1600 per vehicle.

The rough rule of thumb for range loss for EV heating and air conditioning are 5% and 10% respectively. One of the recommended heaters for an EV has a rating of 1.5 kw. The Leaf with its 24 Kwhr pack would have to be stuck in a traffic jam for 8 hours before 50% of the pack is consumed. While the author supplies numbers, there are a number of disconnects, innuendos, and a definite "down on EVs" attitude.

For me, a Leaf would be close to an ideal commuter, gofer vehicle. One to two recharges a week. A way of bypassing the gasoline pumps. $2.50 for a "fill up". Couple this by using the electricity banked from the electrical savings of a solar hot water heater and I should not see an increase in my electric bill.

Before we start to praise or pan something, I think we ought to do some critical thinking by asking pertinent questions. There are some things that need to be praised and some things that need to be panned almost without question. But this article begs for a lot of questions to be asked.

Peter, the graph you refer to (your first link) is the same I link to here:

"Darryl Siry, former CMO of Tesla, estimates that the correct range of the Nissan (and other modern electric cars) will be around 70% of the advertised range."

You are correct to say that I made a mistake here. In the text accompanying this graph, Darryl Siry writes: "As you can see below, this test cycle assumes an average driving speed of 19.59 mph and in the 22 minute driving cycle, it assumes you only break 40 mph once, for about 100 seconds, and never exceed about 58 mph."

I mistakenly translated this into a "top speed" of 40mph, I don't know why. But what influence does this have on my story? Nothing at all. Because I build further upon the conclusion of Darryl Siry, which is correct.

I agree that actual range is likely to be, on average, significantly less than advertised, but...

1) Tesla is a competitor, so they have to be taken as a hostile witness, and

2) The official range is certainly reachable with careful driving.

I agree with your second point, but where I live careful driving is a very rare thing. I also think that modern cars provoke an aggressive driving style. It takes less effort to accelerate fast than to accelerate slow, and you can drive 100mph without even realizing it.

Concerning your first point: I doubt that the Tesla Roadster is a competitor for the Nissan or the Mitsubishi, considering its price tag. And Siry did not just state that the range will be 30 percent less, without any further comment. He sums up all the reasons why and you have to admit that he makes a conservative estimate.

Of course, as I state in the article, his criticism backfires: the EPA-combined range advertised by Tesla is as much suited for a sports car than the EPA-city range is suited for a family vehicle like the Nissan Leaf.

I don't know where you live, but I've not even approached 100 MPH without knowing it very well.  (And none of those were in cars with speed limiters; one a turbocharged I4, one a 24-valve V6, one a 5.7 liter V8.  EVs typically have programmed limits well below that.)

If driver education trained people for careful driving, things would be different.  During the fuel shortages after Hurricane Katrina, things were different even as far away as Michigan (people drove 60 MPH on freeways with 70 MPH speed limits).  Even the response curve of the accelerator pedal has an influence, as does the strength of its return spring (cf. the 2CV).

Just because something is normal today, does not mean it will be next year... or even tomorrow.  Think outside the box.

If driver education trained people for careful driving, things would be different.

You are daydreaming. Driver education will not achieve anything as long as car manufacturers sell us vehicles that are basically racing cars.

I've not even approached 100 MPH without knowing it very well.

Then American cars must be inferior to (large) European cars. Try a BMW, for instance. I have no experience with driving on American motorways, but in many European countries a car passing you at 100 mph (or above) is far from exceptional. And I am not only talking about Germany here. From my own experience, the average speed on a European motorway (except when congested of course) is about 87 mph (140 km/h), well above the maximum speed limit.

In several countries, you only risk a fine if you exceed the speed limit by more than 10 or 15 percent. Radio stations alert drivers of radar controls. Excessive speeds are socially accepted by most people, including politicians.

I can assure you Kris, that in most highways even remotely near cities, 100mph is very rare. We don't have the Autobahns/etc that can be found in Europe. I didn't notice the speeds you talked about when in Italy (biking on highways at times), Norway, or France, though as a tourist, I was not immersed in traffic there on a continual basis.

American cars must be inferior to (large) European cars. Try a BMW, for instance.

I once had a VW Golf which wouldn't even make it to 100 MPH.  I currently drive a Passat which makes it to ~140 kph fast enough, but you KNOW you're moving.  I've never driven it at 160 kph, and probably never will.  The roads and traffic here just don't allow it.

The fastest car I've ever owned was American, and it would hit 100 MPH with ease.  I believe it topped out somewhere around 140 MPH, and I actually did have it at that speed one day.  But there was no way to avoid feeling that speed.

Back in my misspent youth I too had a Dodge Dart which would do 140 mph (225 km/h). What I found disconcerting was that it wouldn't stop from 140 mph. In a hard stop, terminal brake fade would set in before I got it down to a full stop, and one of the rear wheels would lock up. I don't think the tires and the engine would have lasted very long in sustained cruise at that speed, either.

A friend of mine had a Dodge which the dealer claimed would do 150 mph. It did, too, but only once. After that he needed a new engine. Another friend bought a Chevy equally as fast, and crashed it sideways into a parked car driving it home from the dealership. Fairly quickly I realized that all that power was just totally impractical in North America, so I sold it after a year and used the money to get another college degree.

The thing about German cars is that if they are capable of going 225 km/h, they are also capable of cruising all day at that speed, cornering at that speed, and stopping as many times as you want from that speed.

I recall reading an article by a journalist who once had a job delivering exotic cars. He was delivering one of the first Lamborghini Muiras (modified for racing) from NY to LA for a client, and was sitting at a stoplight in a small Midwestern town when a Corvette came up alongside and started revving its engine. The Corvette took off in a cloud of rubber smoke, so he tucked in behind and followed it to see what it could do. Eventually, it got up to 150 mph, and he was thinking, "That's awfully fast for an American car", when he saw sparks coming out of the tailpipe. He shifted down a gear, hit the throttle, and shifted into top gear at 180 mph as he passed the Corvette, with his V-12 screaming away at 8000 rpm. When he looked back, the Corvette was pulling over in a cloud of engine smoke.

American cars are not really designed for driving, they are designed to have lots of power, look good, and sound good. And guzzle huge amounts of gas. I always drive them very carefully because you never know what is going to happen if you push them too hard.

The Lamborghini Miura was also interesting in that it got about the same fuel economy as my Dodge Dart.

I think it's fair to point out that the European cars you're describing are much more expensive than the American cars.

Yes, but many other Euro cars are not. A VW golf GTI or an Audi A4, or the Mini, are all 100mph capable and are not that expensive. more than a Kia or a similar sized Chevy, but not outrageously expensive. If you are going to be the best you can;t be the cheapest. The fact that the Euro versions of most of these have a diesel option adds to their outstanding fuel efficiency. This is part of the reason why Europe is not in such a rush to embrace EV's.

If you are going to be the best you can;t be the cheapest.

RockyMtnGuys' comments were really about quality, and Consumer Reports says that European cars are no better than American cars quality-wise. Asian car companies are leading them both, by a small margin.

There is more to quality than what consumer reports says. Driving performance and safety at 100mph, for example, is not one of their criteria, but Euro cars are specifically designed for this - American ones are not.

When I said quality, I had in mind the reliability and lack of defects the previous poster was talking about. I would use the words performance and handling to describe what you're talking about.

And, yes, I can imagine that some Euro cars are specifically designed for this but I regard that as a bit of a niche market. It's certainly not a speed I drive at much.

where I live careful driving is a very rare thing

Still, the fact that one can achieve the 100 mile range is meaningful. If the driver's commute is at the outer range of the 100 miles, and she wants to avoid plugging in during the day, she can do so.

I doubt that the Tesla Roadster is a competitor for the Nissan or the Mitsubishi, considering its price tag

Tesla's goal is not to sell $110K sportscars. It's goal is to move to much less expensive cars, which will compete directly with the Leaf, Volt, etc. Unfortunately, Tesla and it's employees have shown no hesitation to spread "FUD" about it's competitors.

where I live careful driving is a very rare thing

I missed Kris writing that.  Where I live, it can be a mile or more between crosswalks which allow bicycles and pedestrians to get across a 4-lane road safely.  Getting people out of cars is going to be very, very hard; it requires pushing against a lot of institutional inertia and matters like traffic-control budgets.  But eliminating fuel consumption during the first 10-20 miles per trip isn't all that expensive compared to the car.  If that's even 50% of total fuel consumption, it is an enormous amount:  the USA uses some 9 million barrels/day.

I agree.

The Toyota PHEV-10 is very likely the economic sweet spot. OTOH, I think most people would find that a little unsatisfying - why plug in for only 10 miles of benefit? I think a PHEV-40 is about the right balance for overall convenience and perceptions of benefit.

Except for going to the airport, evacuating a hurricane, visiting a sick friend and a trip down river to sigh see, I cannot remember a trip > 10 miles that I have taken in the last 8 or so years.

Alan

Yeah, me too. But we're unusual.

We're not really the normal new-car market either - we keep our cars until they're 20-30 years old.

I bought a Prius with the 48/45 EPA rating. I find I can duplicate the Consumer Report test drive figure of 42 mph or I can do better than the EPA ratings. If I drive on level ground, keep the tire inflation at or above recommended levels, and stay at or below 65 mph, I can score in the low 50 mpg range. If I drive it like a conventional car with faster accelerations, tire pressure below the recommended level, turn on the A/C, etc. I can achieve worse than the Consumer Reports rating. It's all a matter on how I drive the car and how I configure critical parts of the car like tire pressure.

With respect to Mr. Siry's comment about 70% of advertised range for the Leaf, I will have to disagree. The LA4 is a very strenuous test for any car whether it will be an EV or ICE. It is probably about the worse set of conditions for an EV pack that I've seen. If the Leaf only gets about 70% of the advertised range, I would expect **a lot** of lawsuits. The Prius's Consumer Reports 42 over the EPA 48 mpg is about 88% of advertised range and there were many complaints and law suits over those figures. What would 70% of range produce??? I doubt any corporation wants to face that many lawyers and that many class action suits.

I've driven in an LA4 type environment when I was 17 years old in NYC. I hit 60 mph on Broadway and vied with NYC Taxi Cab Drivers for lane changes. That type of driving is murder on a pack (and nerves). I suspect that the Leaf will achieve well over a 100 mile range if I drive a Leaf like I drive my Prius today instead of when I was 17 in NYC. I would be interesting to see what range degradation a typical EV driver gets when he tries to do an LA4 test with his EV.

Still the pack **will** degrade over time and Nissan says that they expect it to be replaced when it reaches about **70%** of capacity. At that point Siry maybe right about the range **at the original posted speed**. But all a driver has to do is to slow down and they could **double** their range just like they did in those turn of the century EVs. Mr. Siry is playing on our current mind set. Not the one we will have to adopt to when gasoline becomes scarce and a lot more people are driving in EV mode. When you run out of gasoline, you stop. An EV's performance slows and lets you know that you have to drive much slower and get to a receptacle sooner than later.

The Tesla also has the same situation in that pack capacity degrades over time. What to do? Drive slower and you can achieve your original range albeit at a slower speed.

As an aside, we also had the opportunity to talk with the Leaf engineers at a "Drive and Ride". They drove the car up and down the hills of San Francisco and they are satisfied with its performance.

I think I can sum it up in this refrain: "Well Mr. Boeing, what do you mean by In-Flight Meals? Orville Wright didn't even have time to unwrap a sandwich!!!" Bon Appetite on your next flight.

BTW Kris, I too have enjoyed your Low Tech articles. I was first introduced to them via http://www.energybulletin.net/node/50715 ; your article on Hoffman Kilns.

Peter,

You mention something that is often overlook as an advantage of electric vehicles: "For me, a Leaf would be close to an ideal commuter, gofer vehicle. One to two recharges a week. A way of bypassing the gasoline pumps"

The inconvenience of having to stand at a gas pump in the heat and the cold and doing away with them are seldom mentioned as advantages of the electric car.

I once knew a woman who bought a hybrid Camry due to its high range between fill ups, thus saving her the frequent stops at the gas pump. When I drove a Mercedes Diesel 350SDL (good fuel mileage, big tank) I took this advantage for granted...500 miles between fuel stops. When I finally wore the old thing out, and switched back to gasoline, I found the all too frequent gas stops a real pain in the ass.

I agree with you that for me, the Leaf would be more than enough car for 90% of my driving, but the advantage is somewhat offset by the fact that I buy so little gasoline now it is not a big factor in my life...I spend more for information (cable TV and internet, cell phone service, books) than I do for energy (including my transportation, climate control and electricity use in my apartment) than I spend directly for energy!

RC

Thank you for this great post. Shows the stupidity and futility of trying to have it all.

There is a certain irony here. From an energy and economic perspective, it makes sense to get the biggest bank for the buck or kwh by limiting EVs with respect to speed and range, which means they will mainly make sense in or close to cities. This would take care of most people's driving needs. On the other hand, it is within and around cities that we should consider substantially doing away with the need for a private auto and concentrate on feet, bikes, electric bikes, electric scooters, buses, street cars, and subways.

Yes, we could run EVs on solar and wind, but additional renewable electricity should be focused on replacing electric uses, not satisfying the needs for new uses.

Assuming,however, that we won't move to car free cities for a long time, bring on low speed, low weight EVs and quit ignoring the sustantial energy and material costs associated with big batteries. No great breakthroughs required, including the fact that in many communities people are using golf carts to get around with great success.

For the time being, money is being wasted on long range "solutions". Long ranges should be taken care of by trains and buses.

I first learned about Low Tech Magazine thru TOD, and have been reading it ever since.

Living a low energy low cash small farm life, you learn ti adapt to a little inconvenience faster than most people realize.

If as I expect fron reading TOD, gasoline prices ruse sharply in the not so distant future,electric cars capable of forty to fifty mph criusing speeds and fity to sixty mle round trips wil satisfy most drivers, especially if gasoiline is rationed.

There will be at that time many tens of millions of second hand , low mileage, but economicalto drive ice powered cars available for use as a second car to be used for weekend trips, etc.

Most people seem to think the change over needs to come overnight for some reason, but it will come over a decade, like the pc , or the cell phone.

I think the new elec. car versions/options are all too complicated. Electric cars need to be light and simple. Take out all the electronic crap except a radio. We tend to forget that cars are transportation, and not wombs or psychological fulfillment.

I needed a plain work truck. I purchased a rebuilt 86 Toyota due to its simplicity and lack of plastic add-ons. It works awesome and looks great. I forget it is almost 25 years old until I get asked about it. Oh yeah, it was 25% of a new one, and provided a friend with employment as he rebuilt it from top to bottom.

If peak oil or dodgy finance hits us like many believe, this marketing nonsense (Tesla, etc) will be outrageous luxuries, and probably not too realistic. I don't think we'll see them driven by regular folks....if at all.

Seems we are ignoring the first principle of design. KISS (keep it simple,stupid). While eventually PEV's may emerge as superior personal transportation vehicles they are nowhere near there yet.

Give me a year 2000 Honda Insight, a VW Luppo 1.2 diesel engine, and a $100,000 development budget and I'll deliver a production-ready $20,000
car that will go 100 mph and get 100 mpg at 60mph. I'll simply remove all the weight of the hybrid system and batteries, lighten a very light car even more by making the hood, doors and fender skirts out of carbon fiber instead of aluminum, replace the mirrors with a back-up tv camera and refine the engine room airflow. Not rocket science-- but superior to 90% of the entrants in the Progressive X prize. Light and simple triumphs over techie unless the tech is really revolutionary.

Seems we are ignoring the first principle of design. KISS (keep it simple,stupid). While eventually PEV's may emerge as superior personal transportation vehicles they are nowhere near there yet.

Give me a year 2000 Honda Insight, a VW Luppo 1.2 diesel engine, and a $100,000 development budget and I'll deliver a production-ready $20,000
car that will go 100 mph and get 100 mpg at 60mph. I'll simply remove all the weight of the hybrid system and batteries, lighten a very light car even more by making the hood, doors and fender skirts out of carbon fiber instead of aluminum, replace the mirrors with a back-up tv camera and refine the engine room airflow. Not rocket science-- but superior to 90% of the entrants in the Progressive X prize. Light and simple triumphs over techie unless the tech is really revolutionary.

HorizonStar,

Also checkout the Audi A2, you will find it even more along your idea of thinking. But I have to tell you your not going to build a $20,000 vehicle with carbon fiber. You will turn it into a $2 million car.

Not build the chassis out of carbon fiber-- just simple secondary components. Check out the Fiberforge process which automates production of this level of component.

In any case the 50# or so saved won't make or break the design concept, and might be a good bargaining chip to toss the bean counters if you were trying to get such a car actually built in volume.

HorizonStar,

To start with 90+ % of the alumiumn in todays cars is in the transmission case, cylinder heads & block, wheels and suspenion. Good luck driving around with carbon fiber cylinder heads. The chassis (about 25% of the mass of the car) is your major area where you can save some weight. There is also a safety issue replacing steel parts with other materials. Car Engineers spend their life examing these issues.

DownToTheLastCookie said,

"Car Engineers spend their life examing these issues."

Yes, but "car engineers" working for the large mass manufacturers of cars have to play to the lowest common denominator, which means the SUV driver or "sport sedan" driver who is not all that concerned about fuel mileage...there is really no great reward (financially) in studying efficiency.

The great work in the immediate postwar period was done either (a) in nations with highly taxed gasoline or (b) in small bore racing, where engine size was limited by the rules.

There was some very creative work in those areas, the Citroen 2CV, the very beautiful and light Lotus Elite sports and race cars, the fantastic efficiency of the work of Frank Costin (cofounder of the race engine firm Cosworth) in his very rare plywood chassis sports racing cars and the record seting (for both long distance and fuel economy at very high speed over long distance) of the C111 Mercedes speed record Diesel cars. These were the "car engineers" willing to work at the limit of efficiency and found some reward in it, more the labor of love than money.

RC

ThatsItImount,

Thank you for helping me make my point. The Citroen 2CV, Lotus Elite Sports and race cars, C111 Mercedes all cost more than $20,000.

The Citroen 2CV, Lotus Elite Sports and race cars, C111 Mercedes all cost more than $20,000.

Huh? The Citroen 2CV was one of the cheapest cars on the planet. In continental Europe it cost about half as much as a Volkswagen Beetle. It got about 80 miles per gallon and could drive across a plowed field without breaking the eggs - which indicates its primary market.

It didn't go very fast (the original version only went 40 mph) but that wasn't the point. The point was to drive to or from the market at minimum cost without breaking the eggs.

I think the modern equivalent would be the Tata Nano. It goes faster but would probably break a lot of the eggs driving across the field.

Your right, I shouldn't have put he Citroen on the list being first produced back in 1948. But again, try building one out of carbon fiber for less than $20,000. It's not going to happen.

That should tell us all that carbon fibre is not worth pursuing. It is energy intensive to manufacture, and needs oil based epoxies

Aluminium panels work fine, or, if you want nature's original carbon fibre, use wood. You can do lots of things with wood, be it strip built, plywood etc. Some things made of wood even float, or fly. One of the most successful aircraft of WWII, the DeHavilland Mosquito was made of plywood. It was cheaper, lighter and stronger than aluminium planes, and easier to repair after being shot up!

The very reason why they used plywood was that it did not need energy intensive metal (aluminium) and could be made using labour from woodworkers instead of expensive machinery - does that sound like a good solution for today or what?

To see an example of what can be done with wood for a car, check out;

http://www.joeharmondesign.com/pod.html

One of the most successful aircraft of WWII, the DeHavilland Mosquito was made of plywood. It was cheaper, lighter and stronger than aluminium planes, and easier to repair after being shot up!

Why was wood abandoned?

After the war, the focus of the military turned to jet engines, and subsequently, supersonic flight. For these applications, they not only used aluminium, but titanium alloys, which are higher strength, more energy intensive and more expensive. Light weight is not as much of an issue on these planes, as much as the internal volume, and metal is a winner there.

For large passenger planes, they also went to titanium - the 747 could not have been built without it.

Light aircraft are a different story. Any that are built out of fibreglass, can be built out of wood. And the same goes for car bodies, and even frames.
The Morgan, made in the UK, still has a wood frame, as it since the 30's.
http://jalopnik.com/5226093/morgan-aero-supersports-gives-us-wood

Not the light weight of this vehicle.

I think wood panels are considered old stuff and not high tech and sexy, but they can be done.

Not that many people know how to do sophisticated woodwork any more. Wood is inconsistent and tricky to work with compared to aluminum, and it tends to burn up at supersonic speeds. Designers who want something better than aluminum use titanium.

The real reason the British built the Mosquito out of wood was that it didn't affect the production of aluminum aircraft such as the Spitfire. They were built in different factories by people with different skills.

Or as Göring said when one of his speeches was bombed out by a Mosquito raid, "Every #$%^!&@ piano factory in England is building those $#%^!&@ things". Or words in German to that effect.

Actually RMG, you might be surprised at how many people do this sort of woodwork today. There is a quite a large community of wooden boat builders. Most of these boats are home built because the production ones are fibreglass. I am a self taught woodworker, and have built a cedar strip canoe, and a plywood dinghy.
The modern plywood/epoxy method, called stitch and glue, is suprisingly fast, and produces boats that are lighter and stronger than fibreglass. It does use glass cloth, so technically, you have a "wood cored" fibreglass. My cedar strip canoe weighs in at under 30lbs!
The plywood method can be partly automated, as the plywood can be cut to pattern by CNC machine. If the shapes are designed correctly (and the plywood boat designers have got this sorted), you can get nice compound curves out of the plywood.

http://www.bandbyachtdesigns.com/cs17kit.htm

Not as efficient as a metal stamping machine to be sure, but a wood shop that was set up for this, with appropriate jigs etc could turn out panels/bodies quite efficiently. As a production process, it uses more labour and less equipment, and most importantly, can be done at small volume.

Back to the Mosquito (my favourite plane of all time) DeHavilland knew before the war that aluminium would be in short supply, but they also knew they could build faster in wood - it took a year off the time need to build the prototype!

From the Wiki site for the Mosquito, De Havilland said this to the RAF;
We believe we can produce a twin-engined bomber which would have a performance so outstanding that little defensive equipment would be needed. This would employ the well tried out method of construction used in the Comet and Albatross and being of a wood or composite construction would not encroach on the labour and material used in the expanding RAF. It is especially suited to really high speeds because all surfaces are smooth, free from rivets, overlapped plates and undulations and it also lends itself very rapid and subsequent production.

From Day one, the Mosquito was the fastest plane in the world, and remained so until the end of 1944.
In the time it took a four engine Lancaster to do one mission, a Mosquito could do the same mission twice. It had the lowest operational loss record, per hour flying time, of any combat plane. It also was the first bomber to not have any defensive guns, and none have had so ever since.

As for Goring, this is what he actually said, in 1943, after the Mosquito became the first plane to bomb Berlin;

In 1940 I could at least fly as far as Glasgow in most of my aircraft, but not now! It makes me furious when I see the Mosquito. I turn green and yellow with envy. The British, who can afford aluminium better than we can, knock together a beautiful wooden aircraft that every piano factory over there is building, and they give it a speed which they have now increased yet again. What do you make of that? There is nothing the British do not have. They have the geniuses and we have the nincompoops. After the war is over I'm going to buy a British radio set - then at least I'll own something that has always worked.

Funny thing is that, post war, the British cars, aside from the Mini, have not been that reliable, and the Germans took over the mantle of the best cars in the world.

Amazing what happens when you mention carbon fiber. Now my mention of making a few secondary body components out of the material has me building the cylinder heads out carbon. Why not go the whole way and argue that any future vehicle can't work because it has to be made out of Unobtanium?

For a real world 1250# vehicle's potential pay attention to the following report.

"I attended a test session of the FVT (Future Vehicles Technology) eVaro on February 23, 2009. A series of test runs were conducted near Agassiz, BC on BC Highway #7, immediately east of Chowat Road. All test runs were conducted between 1430 and 1730 local time. Each test run consisted of a 1km outbound leg, a 180 degree turn, and a 1km return. Battery voltage and current was acquired and logged during each run through the use of transducers and software provided by
Motec Engine Management and Data Acquisition Systems.Meteorlogical conditions for the test period reported at Aggasiz by Environment Canada show a temperature range from 10.0 to 10.8 degrees Celsius, winds were from the North between 6 and 11km/h, light rain though not reported by Environment Canada, was evident toward the end of
the test session.
The section of highway used for the test session is a busily traveled portion of public highway, it is reasonably flat and the exact grade of the highway should be a matter of public record if it is required. The accuracy of the onboard data acquisition system was verified both prior to the test runs and then at random intervals throughout the test session by comparison reading obtained using a Fluke 190 Series Scopemeter. The margin of error between the measuring instruments was very small, and the repeatability of the results was good.
The eVaro achieved an average of 275 mpg for city driving 20-60k) and an average of 165 mpg for highway driving (70-120k)."
Randy Kelley
Electronics Instructor
University of the Fraser Valley
5579 Tyson Road
Chilliwack, BC
V2R 0H9

HorizonStar,

Also checkout the Audi A2, you will find it even more along your idea of thinking. But I have to tell you your not going to build a $20,000 vehicle with carbon fiber. You will turn it into a $2 million car.

The A2 may be more practical, but it never can be a 100/100 vehicle.
I'll take your 2 million any day, but mass producing small component parts in foam/carbon is closer two 2x aluminum than to 100x.

Most people seem to think the change over needs to come overnight for some reason, but it will come over a decade

I am counting on it.

One of my friends is fairly happy with his new Zap Xebra, 45mph top speed, 40 mile range. I think it has been discounted some lately from $11,900 list.

If the Volt were to work as advertised, and everyone switched, that would be a major game changer in the amount of petroleum we consume in the US. Yes, I know it's prohibitively expensive right now, and even if it was affordable for the masses, folks wouldn't switch as long as gasoline is cheap, but I'm commenting hypothetically.

Another game changer would be if there were a sudden change in standards that would allow the gas sipping diesels that are available in Europe to become available in the US. Cars like the 64MPG Ford Fiesta ECOnetic. Yea, that's about the best they have, but they have many other sippers that get better than 50 MPG. This fall, the US will have only two non hybrid cars that get better than an EPA rating of 40MPG, the Smart, and the Chevy Cruze Eco.

Now if thew Obama administration really wanted to do something useful, doable, and popular, they would introduce a bill enabling the people in this country to buy these European model cars.

The conservatives who are generally (and often justiafiably) blamed for the failure of such initiatives will not be able to say much at all if the change is presented as one of enabling the market to work and people to decide for themselves.This is not a case of FORCING car companies to build high mileage cars, or FORCING consumers to buy them.

After all, it would be a case of getting the govt OFF THE BACKS OF THE PEOPLE, RIGHT?

Unfortunately my personal estimation is that the nanny state is more interested in power and the status quo that in easily and readily implemented solutions to our problems.

oldfarmermac,

EXACTLY CORRECT. There is no reason to believe that the cars built in Europe or Japan are less safe than U.S. cars. The only reason cars are not marketable across national borders into (and out of the U.S.) is good old fashioned protectionism and the remaining power of the labor unions...in a world that claims "globalism" as its mantra, we are still very far from a global economy, and with the recent financial problems will probably get even further from it as everybody throws fences up to protect their home producers.

RC

Great article. Thanks.

It is obvious that we will have to consume far less energy in the future. With regard to our existing transportation fleet, the simplest (only?) solution will be to drive less and to significantly reduce speed limits. When we take these necessary steps I wonder if the case for EVs will vaporize? Maybe precious treasure being invested in EVs should be redirected to wiser things like home energy efficiency and farmland preservation/restoration.

On the embodied energy in a battery, calculating that from the cost of the battery is misleading. It involves intellectual property. The great thing about intellectual property is that once something is discovered, it's discovered for all time, and sharing information doesn't take it away from others. How much would state of the art batteries cost if not for any patents etc?

Could be true, but there is no other way to get an idea of the embodied energy of an electric car battery. There does not seem to exist any Life Cycle Analysis. Which is very weird if you consider the large number of studies that conclude that electric cars are less damaging to the environment than ICE vehicles.

Kris,

Here's one source: http://www.transportation.anl.gov/pdfs/HV/458.pdf

Page 17 seems useful. We might have to reverse the calculations to get the original BTUs or KWHs per KWH. I think they're using lifetime miles:

"For those interested in doing some sensitivity analysis computations concerning our current estimates of the energy and emissions impacts of the batteries, we provide this separate table (Table 7) of the energy use and emissions due to the battery packs alone, for one battery serving the entire vehicle life."

From section 5.2, page 16.

I would use 150,000 miles, as that's the Volt battery warranty.

--------------------------

Here's another: Table 1 in Impacts of EV Battery Production and Recycling (Linda Gaines and Margaret Singh, Argonne National Laboratory, Symposium Date: April 29 - 30, 1996) http://www.transportation.anl.gov/pdfs/B/239.pdf indicates:

energy to manufacture a 25 kW·hr lead-acid battery: 11.7 million BTU.
energy to recycle a 25 kW·hr lead-acid battery: 2.5 million BTU.

------------------------

Now, I couldn't get the LCA link you provided to open. Does it use BTUs, and convert them to KWHs? This would introduce a big inaccuracy.

Nick, thanks for the links.

I made a quick calculation based on the information found in the first link you provided (the second link only deals with lead-acid batteries as far as I understand).

The result I got for the Tesla battery is 29,269 kWh of embodied energy. That's more than the 23,222 kWh I mentioned in the article...

I used the 150,000 miles you suggested, but I think that's a bit too optimistic. As I remember it correctly, the warranty for the tesla battery is only 100,000 miles. In that case the result is 19,400 kWh, slightly below the figure I gave in the text.

My calculation can be wrong of course. This is how I got it:

49.6 kg of battery = 48,500 J/km (information from your first link)
multiplied by 241,401 kilometres (150,000 miles)
= 11,707,948,500 joule
= 3,252 kWh for 49.6 kg of battery
= 29,269 kWh for 450 kg of battery

Correct me if I am wrong.

The LCA link I provided gives the results in joules.

I think the chart from ANL for the 458.pdf link is probably most useful since it actually deals with Li-ion batteries.

One thing that has to be made clear is what km number they divided for the energy use figure. The note says "* We divide by the annual kilometers of use of the vehicle". Average annual usage in the US is 12k mile = 19.3k km. Plugging it in works out to 260kWh for the PHEV16 (Saturn Vue with 8kWh battery). Normalizing to the Tesla's 53kWh works out to 1722.5kWh (6500 miles of driving in the Tesla, 1500 miles of driving in an Elise), which is a lot better than all of your figures. The report said "The share of total pathway emissions caused by batteries is not negligible – a few percent at most," which is why I doubt the ~20k kWh (which is quite major in impact) which you arrived at.

Of course to really put these figures into prospective, I think you have to find the impact of making a car in the first place and also the energy content of 100k miles gasoline driving. If you find this figure, I think you will see the relative energy figure for battery production really isn't as huge as it is made out to be.

LCA link is certainly very interesting, but it does kind of disadvantage li-ion batteries since li-ion has the highest $/kWh, while having the lowest mass/kWh (meaning less material usage per kWh), so the figures there may be misleading, esp since the "storage battery" category includes the lead acid battery market. Also, the data is from 2002, and given the (commodity) li-ion battery industry moves at ~8% density improvement per year (meaning ~8% better capacity for the same battery weight/volume), the data is outdated (for example, in 2002, laptop cells were less than 1Ah, today it reaches 3.1ah, more than 3x improvement).

First, I suppose we probably should use the figures for the PHEV64, which are slightly lower. That gives us 28,203 KWH for the Tesla.

2nd, in this case you can't really convert joules to KWH. It's very likely that much or most of the energy input was in the form of directly consumed FF's, which are much less efficient. ICE vehicles in the US use the equivalent of 1.5KWH/mile, which is about 6x the electricity of a comparable EV.

In any case, the author makes things simple for us: we see in table 7 that the green house gas contribution of the Volt/PHEV64 battery is 8.67 grams/km, versus 142 g/km for the most efficent, least polluting source of electricity (combined cycle natural gas). That's only about 6% for the worst case scenario: compare the 3.84 g/km for the PHEV16, to the 283 g/km for electricity from old coal generation, which is about 1%.

The paper says: "The share of total pathway emissions caused by batteries is not negligible – a few percent at most".

Not negligible, but a few percent at most. And that's a comparison to PHEV electricity consumption. In a comparison to ICE vehicles, that contribution really would be negligible.

Again, an EREV like the Volt is "good enough". Further efficiency improvements would be nice, but are not necessary.

The perfect is the enemy of the good.

So if I get you guys right I did wrong by multiplying the result by the lifetime mileage (originally suggested by Nick) and should use annual mileage instead?

Considering the wildly differing figures and the many uncertainties in the study that were pointed out above, I tend to go back to my original conclusion: we badly need a decent life cycle analysis of a lithium-ion car battery.

I find it very hard to believe that manufacturing the 450 kg battery pack of the Tesla Roadster, consisting of more than 6,500 lithium-ion cells (which corresponds to more than 1,000 laptop batteries) and full of rare metals would only take 1,722 kWh.

Producing 450 kg of wood (from standing timber) already corresponds to an embodied energy use of up to 900 kWh. Manufacturing 450 kg of plastic (from crude oil) takes up to 14,400 kWh.

http://www.lowtechmagazine.com/what-is-the-embodied-energy-of-materials....

So if I get you guys right I did wrong by multiplying the result by the lifetime mileage (originally suggested by Nick) and should use annual mileage instead?

I have to admit that I don't understand the calculations in the paper. I think it's simplest to use the tables the way they were meant to be used, in terms of inputs per KM. In those terms, it's clear that the battery contribution to the overall energy consumption of the vehicle is small.

we badly need a decent life cycle analysis of a lithium-ion car battery.

I've been told that the lead author of this study will complete a li-ion LCA around now. I've followed up with an email to her.

Some thoughts:

1st, I think you need to be very careful with conversions from joules to KWH. Basically, I think it's a mistake. Joules are generally understood to be a generic unit of energy (despite it's definition as watt-seconds), whereas KWH is only a unit of electricity. For instance, a US gallon of gasoline has about 35"KWH" of energy, but burning it to produce electricity is likely to only produce about 13 KWHs, and in a US ICE vehicle it will only take you 22 miles, while a comparable EV could go about 140 miles.

I'd use BTUs for heat, joules if you really like ISI units, and KWH only for electricity.

2nd, I think you're overestimating the energy consumption of manufacturing. In general, it's mighty efficient: energy costs are only a small % of manufacturing costs, and they're generally improving.

I'd be careful about using the upper range of the numbers for materials' embedded energy cost. Some may be old, some probably represent sub-optimal processes.

In many situations, energy costs have not been the primary focus of cost reduction and efficiency efforts. When they appear in the "sights" of the manufacturing engineers responsible for such things, they'll drop to the low end of those ranges and well below.

For example: gaseous diffusion for uranium isotope separation takes enormous amounts of energy. Further, the existing gaseous diffusion plants in the US are old and very leaky. Any new separation plant would use centrifuges, which use only 2% as much energy. Well, the range of efficiency of existing separation plants has a 50:1 range, and you can't really use the high end for planning purposes.

I think you need to be very careful with conversions from joules to KWH. Basically, I think it's a mistake. Joules are generally understood to be a generic unit of energy (despite it's definition as watt-seconds), whereas KWH is only a unit of electricity.

I think you should rethink this statement, on several levels. Firstly, joules are the basic unit of energy and are defined as a force of one Newton applied over a distance of one metre. A watt, of course, is the basic unit of power, which is energy per unit time, and is one joule per second. A kilowatt is 1000 joules per second, and a kilowatt-hour is 1000 joules per second for 3600 seconds, which is 3.6 million joules, or 3.6 MJ. By the way, the correct notation for a kilowatt-hour is kWh, not KWH.

So converting from joules to kWh and back is very simple - one kWh is 3.6MJ, and one MJ is 0.28kWh

A BTU is a completely different unit of energy, the amount needed to raise one pound of water by one degree Farenheit, and bears no fundamental relationship to kWh.

"a US gallon of gasoline has about 35"KWH" of energy, but burning it to produce electricity is likely to only produce about 13 KWHs, and in a US ICE vehicle it will only take you 22 miles, while a comparable EV could go about 140 miles."

This is also a misleading statement. So one gallon will get an average car 22miles (or an efficient car 30 miles), and if we make electricity out of it, to get 13 kWh, you are saying an electric car will go 140 miles, or about 11 miles per kWh. the Leaf has a 24kWh battery, and Nissan is claiming a 100 mile range, or about 4 miles per kWh, and for our 13kWh you will get about 52 miles, not 140.

If you take 35kWh of electricity to start with, then yes, you get 140 miles, but you don;t start with electricity, you start with something else (coal, NG, uranium) to make electricity.

The simple fact is, that care must be taken when converting from fuel energy to electricity, and back. The kWh is the simplest unit to use, as it can measure both, for easy comparison. Introducing btu's into this equation makes things unnecessarily complicated. if someone is contemplating spending $25k+ on buying an EV, then they should educate themselves about these units - they will be much better informed buyers for it.

It's important to keep in mind how people use words, despite their technical definitions.

As a practical matter, if you refer to a kWh, it generally suggests electricity. Of course, that's a correct understanding.

Firstly, joules are the basic unit of energy and are defined as a force of one Newton applied over a distance of one metre.

Sure, but joules started out as a measure of electrical energy:

"The joule was recognized internationally in 1889, at the second International Electrical Conference, as a derived addition to the practical units of the c.g.s. system; it was defined as the energy dissipated in 1 second by current of 1 ampere flowing through a resistance of 1 ohm. Hence the practical joule. " http://www.answers.com/topic/joule

the correct notation for a kilowatt-hour is kWh, not KWH.

Sure, but that's harder to read. Let's be practical.

converting from joules to kWh and back is very simple

Of course it is - it's just very misleading. Heat, raw fossil fuels and electricity are all very different.

A BTU is a completely different unit of energy, the amount needed to raise one pound of water by one degree Farenheit, and bears no fundamental relationship to kWh.

Of course. Nevertheless, it's unambiguously a measure of heat, and useful therefore. In the US, it's rather more useful than joules.

if we make electricity out of it, to get 13 kWh, you are saying an electric car will go 140 miles, or about 11 miles per kWh.

No, if we start with 35 KWH in the form of electricity, it will propel a Volt or Leaf about 140 miles.

If you take 35kWh of electricity to start with, then yes, you get 140 miles, but you don;t start with electricity, you start with something else (coal, NG, uranium) to make electricity.

Exactly my point: it takes roughly 3x as much primary energy to create 1 unit of energy in the form of electricity. In general, that relation holds: electricity is much more useful than energy in the form of FF, or heat.

The kWh is the simplest unit to use, as it can measure both, for easy comparison.

Yes, and it will create many confusions and misleading results.

Strictly speaking, the Joule is a derived unit, not a fundamental one, so I shouldn;t say "defined as", since you can get to it from two different pathways.
However, in terms of SI base units, it is m^2.kg^2/s^2. The SI system uses mks as its fundamental units, cgs has been out of use for some time.

http://en.wikipedia.org/wiki/SI_derived_unit

the correct notation for a kilowatt-hour is kWh, not KWH. -
Sure, but that's harder to read. Let's be practical.

Ease of reading is not the primary concern, accuracy of information is. the capital H stands for Henrys, the unit of Inductance, and capital K is for degrees kelvin. The reason why the SI system was established was to standardise such terminolgy and units into a coherent system. Making arbitrary changes as you suggest to make it "easier to read" goes against it's purpose and can lead to confusion, and sometimes serious mistakes. If we are discussing thee technical matters, things are simplified if everyone sticks to the conventions. This may seem nit picking, but with electric motor design, inductance comes into play so H is used. If people want to get into the physics of electric cars (or anything else) then they should learn to read the proper notation, not change it.

A BTU is a completely different unit of energy, the amount needed to raise one pound of water by one degree Farenheit, and bears no fundamental relationship to kWh.
- Of course. Nevertheless, it's unambiguously a measure of heat, and useful therefore. In the US, it's rather more useful than joules.

Actually, a BTU is not that useful at all these days. It was developed as a unit relating to heating of water in boilers for steam engines - it is really a measure of the heat capacity of water. This has nothing to do with combustion engines and even less to do with electricity. Kilowatts and newtons relate to energy and moving objects - which is what this is fundamentally about, not raising temperatures. if fuel energy is measured in Joules of even kWh to start with, there is one less conversion needed, a step saved, and a potential opportunity to make a mistake is eliminated. Just as with energy itself, the less conversions you have to do, the better.

The kWh is the simplest unit to use, as it can measure both, for easy comparison.
- Yes, and it will create many confusions and misleading results.

Only amongst people who can't be bothered to educate themselves otherwise it leads to good understanding and accurate results. If someone partakes in these discussions without understanding these things first, they are wasting everyone else's time.

If people want to get into the physics of electric cars (or anything else) then they should learn to read the proper notation, not change it.

You've got a point, but still, who is going to read KWH as Kelvin-Watt-Henry?

kWh...will create many confusions and misleading results...Only amongst people who can't be bothered to educate themselves otherwise it leads to good understanding and accurate results. If someone partakes in these discussions without understanding these things first, they are wasting everyone else's time.

If only that were true. I see the mistake of using kWhs as a generic unit of energy constantly, and it's constantly causing trouble.

Here's an example:

David Mackay the physicist at the Cavendish Lab, Cambridge Uni UK looks at renewables in his book, Sustainable Energy - without the hot air http://www.withouthotair.com/download.html.

He says "if we covered the windiest 10% of the country with windmills (delivering 2W/m2), we would be able to generate 20 kWh/d per person, which is half of the power used by driving an average fossil-fuel car 50 km per day."

Well, that's just goofy. We're not going to power FF cars with electricity, we're going to power electric cars. Even if his figure of 50 km/day were to be correct (it's not - the average km/day/person in the UK is only 30), we'd only need 7.8 kWhs (20% of the figure given of 40 kWh) to drive that far.

Mackay, a physicist who presents himself as an energy authority who will demystify energy questions, has fallen squarely into the trap of confusing kWhs with generic energy.

-----------------------------------------------

The basic problem here is that the joule is a unit that we don't use in everyday life. kWhs are familiar, so we want to use them, so people have some intuitive idea of what we're talking about1. Sadly, that very familiarity will mislead people, as they'll use kWhs to measure primary energy, instead of electricity, to which it should be confined.

Perhaps we could create a term like BOE: "kWh-equivalent" to prevent confusion. Even that, of course, causes endless confusion, as people forget that a BOE is not a barrel, and begin to think that oil is the only energy source that matters.

1 which is why BTUs are useful in the US: they're much more familiar than joules.

Nick, I hope no one ever reads it as Kelvin Watt Henry, which is a meaningless unit. But what about Ah (amp-hours) vs AH (amp-henrys) in an electric system with storage and inductance (i.e. any AC system with a battery, inverter and a motor) you will have both of these, but the reader will have to get the meaning from context, which leads to confusion. If people start using K or H where they mean k or h, then it is only a matter of time before real mistakes are made - get everyone speaking the same language and problems are minimised.

I don't think David Mackay is making that mistake because of units, he is making it because of faulty thinking. No unit system will change that.

As for the Joule, in Australia, and most metric countries, it IS the energy unit used in everyday life and it works just fine. It is Americas desire to cling to an archaic system of units that is the problem - however, I am a realist and realist that won;t change, so we just have to dumb things down to that level, instead of America stepping up.
The more we move to an electric society and away from a heat one, the more logical metric becomes, as electricity IS measured in metric in America.

It is Americas desire to cling to an archaic system of units that is the problem

Then why the heck this strong desire to use a misleading unit like the kWh?

however, I am a realist and realist that won;t change, so we just have to dumb things down to that level, instead of America stepping up.

I think you'll have greater success with your communication if you use more respect.

As far as metric goes - there's no question it's a good idea. OTOH, older systems really do have advantages. For instance, a liquid system of measurement like gallons, quarts, cups and ounces that is binary is far more useful on daily basis than a decimal system. Heck, a base-2 system is a lot more modern than a decimal system, which is based on the number of fingers and toes!

IIRC, the French tried to go to a decimal time and calendar system - fortunately, that got tossed out fairly quickly. The base-60 hour and minute is enormously useful, and the base-24 day is too. Imagine dividing the day into 3 equal decimal parts....

Nick, the kWh is not a misleading unit, In fact is THE consistent unit for electrical energy measurement in every country in the world. Electricity is used by more people than heating, or gasoline for driving. I can't think of a better unit tos use as the basis for comparison, and especially where electric vehicles are involved.

Point taken about respect. But as for the units themselves, I have spent half my professional life having to convert things into American units, and I can honestly say that differing units are the cause of much confusion, mistakes and simple wasted time. In the water business, in metric you have litres and cubic metres, which of course, is just 1000 litres. In US units, water utilities variously use US gallons, Imperial gallons, hundreds of cubic feet, and acre feet. Every time you have a needless conversion, often involving unusual numbers (326,000 gallons to an acre foot).
The old units developed because they were intuitive (e.g. feet and inches, about a finger digit), and builders invented them and they are still used today. But when you convert from one physical quantity to another, that is what metric was designed for, and works way better. Serious engineering mistakes have occurred because of improper unit conversions.
As an aside, US construction companies are less preferred for work in places like China than European ones because of this. IF you are speaking different languages, at least you are using the same measurement system, having different measurment systems just adds another level of complexity.

I'll disagree with gallons, cups quarts and ounces. Does an ounce of sugar mean a volume or a weight? why four different units, and four notations, whne you can just use orders of magnitude for the one unit. Works fine with kilowatts, megawatts and gigawatts - everyone knows what you are talking about. Doesn't mater whether the orders of magnitude are binary or decimal, they are a systematic numerical adjective for the same noun, why then have four different nouns?

It will be interesting to see if the US is still on US units in 100yrs - I think 50/50 at best. A lot of progress in a lot fields is enabled by standardisation - the US does not gain anything by being the holdout. Eventually, like Beta v VHS, it will have to change.

Don't think you will get many Americans here arguing against the metric system. But remember. We Americans don't believe in evolution either.

the kWh is not a misleading unit, In fact is THE consistent unit for electrical energy measurement in every country in the world. Electricity is used by more people than heating, or gasoline for driving. I can't think of a better unit tos use as the basis for comparison, and especially where electric vehicles are involved.

It's certainly a good measurement of electricity1. But, expressions of heat energy or FF energy as kWh are misleading. The Original Post uses kWh, and the poster refers to an article about materials' embedded energy which uses kWh for everything, and it's even more misleading than BOE.

For instance, the Original Post says: "$ 30,000 of economic activity in the storage battery sector (including the production of li-ion batteries) equals an energy consumption of 23,222 kWh - that's almost 6 years of electricity consumption by an average British household. "

This is a very misleading comparison. Those 23,222 "kWh" are not comparable to "6 years of electricity consumption by an average British household".

As for metric - well, I feel your pain. As I tried to say earlier, I have no disagreement about metric being better. I just thought it might help restore a bit of respect for Americans' clinging to old units to point out that they have some virtues. For instance, using different nouns for liquid measurements isn't a problem if they're very familiar. And, yes, an ounce of sugar is not a problem, because recipes specify things like sugar in volume-based measurements like quarter-cups or table spoons, not ounces.

1 and it brings us back to the proposition that kWhs are used because they're more familiar than joules.

he kWh is not a misleading unit, In fact is THE consistent unit for electrical energy measurement in every country in the world.

As noted above, when it leads to confusion between energy as heat and electricity, people often get things very, very wrong.

It might make the physical type of energy clearer if work was measured in MJ or kWh and heat was measured in mega-calories.  One Mcal is only about 1.15 kWh, so it's not even hard to use for estimates.

I disagree with calories, then you introducing another unit, which cannot be derived directly from the other SI units - Joules are the best unit for heat

Believe it or not, it is often appropriate to use kWh for heat. My house has electric heat and hot water - the kWh describes exactly how much heat I get. Much of the US steel industry uses electric arc mini mills - so the heat input is indeed in kWh.
An electric furnace is the same and so on.

If we must distinguish, far better to stay within the SI system, and denote electrical energy as kWeh, as kWe is already widely used, especially in renewable energy.

And, of course, kWh is often used to measure mechanical work from engines.

There isn't much problem if everyone is in metric, and knows what they are talking about.

My house has electric heat and hot water - the kWh describes exactly how much heat I get.

Shouldn't you convert to a heat pump and get 3x as much heat?

Actually, I use a high efficiency wood burning fireplace insert for heating, cut the wood myself, which I enjoy doing. This supplies 75% of the heat requirements. I will replace the HW tank with a heat pump type - it will serve as a dehumidifier in the basement.

I have convinced a neighbour to go with said heat pump - it is surprising how few people think of them. Another neighnour, building a new house, was talked out of it by his contractor - who said it would cost $12k! Usual story from contractors who just want to keep what they have been doing.

It's amazing how hard it is to get contractors and tradesmen to try new things. There must be a very large effective cost to starting a new learning curve.

Yep, just look at the reaction of auto mechanics when fuel injection came in. They all complained about how fiddly the systems were, harder to repair etc, and that they would have carburettor any day. Fuel injection of course is much more efficient, and better performance. It was, eventually, driven by customer demand (and the extra $1k or so being no object) rather than the mechanics opinions.

The people who service things are generally reluctant to embrace new stuff if the old stuff was reliable - fuel efficiency is not the serviceman's problem

That's because they WERE fiddly and hard to repair, not the least for being unfamiliar.  Today, some of the electronics diagnose themselves and the trouble codes tell you where the problem probably is.

I don't dispute that, and so are diesel injectors (and always have been), but the fuel savings are worth it. The servicemen would have had us not go to them at all.
I am not a fan of complexity, it has to justify its existence, and injectors obviously have.

Actually MacKay hasn't made an error - it is just the way his book is laid out. He starts by comparing all kWh as if they were the same then points out in a later section that this is wrong as it ignores conversion efficiencies and specifically uses a shift to electric cars and electric heat pump heating to reduce demand as a major part of any solution.

That seems deliberately misleading. That chapter specifically says that wind power's electrical output is inadequate.

If he says later that using electricity properly is a major part of the solution, he's gone to a great deal of trouble to make wind look bad.

Could you point me to the specific section where he says this? I'd like to see if he circles back and repairs the damage to perceptions of wind's adequacy.

For transport specifically see chapter 20, chapter 26 discusses problems of wind intermittence and EVs (and using their batteries to back wind) is one of the main suggestions for coping with it. The main bit is in chapter 27 though (page 204 in my printed edition) where electrification of transport is expected to roughly halve its primary fuel use despite assumed economic growth, with further savings from electric heat pumps. One of his proposed solutions has 32 kWh/d of wind although he seems very pessimistic that enough back up storage is practicable, oddly none of his plans suggest using lots of wind and lots of nuclear even though with molten salt heat storage nuclear could back up wind intermittence quite well.

My main objection to his book is the assumption that further growth in energy use from economic growth must be met rather than saying the economy must match the energy available.

I think you are reading to much into his initial matching of wind generated electricity with fossil fuelled road transport - his next two items are PV electricity and air travel and electric aeroplanes are clearly only possible by making synthetic fuel where the conversion efficiency is the opposite to electrifying land travel.

I'll take a look, when I have time.

One note: air travel is less than 10% of oil consumption and about 3% of overall energy consumption, while personal transportation is about about 5x larger. Air travel can run on the remaining oil for quite a long time. Further, aviation is a very nice thing to have, but it's really optional.

Sodium cooled reactors will never be built in the USA. I will be personally protesting such an inherently unsafe technology. We all know what happens when H2O and Na meet.

In my lifetime, I never expect to see a large scale commercial (non-experimental) nuclear reactor completed that is not a Gen 3 (or Gen 2, Watts Bar 2). Such as AP 1000, EPR, Toshiba's latest, Advanced BWR, Advanced CANDU, etc.

So forget some new drawing board nuke having an impact.

Best Hopes for at least 5 new US nukes in the next decade,

Alan

I absolutely agree that all new nukes must be established designs at least for the next decade or two and likely much longer, but there is no reason why heat storage cannot be applied to any system where the steam could be diverted from the turbines to the heat store. The heat is recovered later with the main source used to upgrade the temperature back to optimum before feeding the turbine, this is not done at present because nuclear heat is so cheap in fuel cost just venting off the steam occasionally is cheaper than building the storage and gas peaking also costs less to build. In the UK at least we can start the process by building new nukes for base load (replacing existing ones that are life expired and existing coal that is mostly even older) and adding a lot more wind (plus as much tidal as geography allows) with the already built gas turbine plants doing the back up. MacKay was looking at a 40 year plan (by which time just about all UK generating capacity will have to be replaced), the key is to actually have a plan!

Storing heat means that the steam systems would have to be super-sized for peak loads.  Since steam turbines are enormous anyway (because of the extremely low density of steam as it gets to exhaust conditions), this means big hikes in capital cost.  I'm sure the designers have done the calculations and decided that peaking is best done in other ways.

One of the ways it's done is with open-cycle gas turbines, running on air.  Air at atmospheric is much more dense (~1.25 kg/m³) than saturated steam at 10 kPa (0.068 kg/m³).  Storing electricity as compressed air, then using nuclear heat to re-heat it before re-expansion, would have lower capital costs (as well as being able to store power from sources other than nuclear).  If the heat-to-electric efficiency is 80% as in some CAES projections†, the output from a 4 GWth reactor could hit 3.2 GWe (compared to 1.4 GWe from a 35%-efficient steam cycle).

† If the CAES system stores and regenerates the heat of compression, this may not be necessary or even desirable.  See the web site of General Compression.

It's not true that once something is discovered it's discovered for all time.
Knowledge has been forgotten, lost. It is a common occurrence for the arts and sciences of formerly great civilizations to become lost; forever.
The people of the Middle Ages could not maintain Roman technology. We still cannot agree on exactly how the Egyptians built the pyramids. I think that our own time is going to turn out to be especially vulnerable to losing its technical knowledge. Once the educational system collapses, forget about new electrical engineers, chemical engineers, etc. Within just a generation or two we could find ourselves in a world where very few people can make heads or tails out of the simplest wiring diagram.

The great thing about intellectual property is that once something is discovered, it's discovered for all time, and sharing information doesn't take it away from others.

Plenty of times in the "long ago" past ideas and ways have been 'lost' only to be 'rediscovered' or 'reinvented'.

And in the near now you hear of how the way the apollo rockets were done is "lost" or how firm X changes product Y's parts/way its made only to discover that the way it was done was for a reason they didn't understand/know (aka lost to them)

So discoveries are not forever.

Great article with lots of good info. Two comments:
1 - The only problem with lowering the top speed or max acceleration of EV's is that they have to share the roads with existing gasoline-powered vehicles. A lightweight car with a very low top speed and slow off-the-line acceleration will get run over by the monster SUV's and 18-wheelers already on the road. Unless we lower the limit for everybody, it won't be practical to have one group of cars moving more slowly than the others.
2 - With so much current electricty production coming from coal, I wonder what the net environmental benefit of an EV really is, when one factors in the coal-based electricity needed to create the batteries and then run the vehicle.

A lightweight car with a very low top speed and slow off-the-line acceleration will get run over by the monster SUV's and 18-wheelers already on the road. Unless we lower the limit for everybody, it won't be practical to have one group of cars moving more slowly than the others.

Unless they all accelerate together as a train... which is easy to do with sensors and software.

With so much current electricty production coming from coal, I wonder what the net environmental benefit of an EV really is

I keep seeing this canard appearing everywhere.  US electricity is shifting from coal to natural gas, and wind power is growing many times faster than EV charging energy.  By the time EVs are a factor, wind power will be expanding faster than EVs can be produced.

US electricity is shifting from coal to natural gas, and wind power is growing many times faster than EV charging energy.

You're missing the point. Read this please: http://www.lowtechmagazine.com/2009/11/renewable-energy-is-not-enough.html

Think about the NET benefit, which is the subject of the posts above. Sure, renewables can't power BAU, but we're taking steps to move away from it, it's just a matter of how fast and how much.

For a very low carbon footprint for those who are in commuting situations that are more than 20 miles each way that would require something more than an electric bike or velomobile, the Twike represents the right type of choice (if they can only get that price down a little more);

See the Twike Manual for more details.

Kris,

You're missing several things.

First, we're still in the early stages of growth for wind and solar. Just because they aren't supplying all of new generation, or starting to replace coal and other FF's, doesn't mean that they won't be able to in the near future.

2nd, the wind power required to power an EV like the Leaf forever would cost about $2,000. That's a one-time cost.

Building renewables (wind, especially) is the easy way to eliminate fossil fuels. Drastic conservation measures are the hard way. If we can't get the overall society to go with the easy way, it's highly unrealistic to think that we can get it to follow the harder path. Why would we tell people to accept a greatly inferior vehicle to save $500 or $1,000? Why would we try to fight such a difficult, unnecessary battle? If we want to eliminate FFs, it's not that expensive. If we don't have the will to do even that much, we certainly aren't going to be able to sell a conversion to very small, low-powered EVs.

3rd, EVs actually support the buildout of renewables, especially wind, by providing night time demand and soaking up intermittency. We don't have to choose between EVs and the reduction of emissions - the two goals are actually synergistic.

The straightforward solution to AGW is to build out wind, solar and nuclear (if you like nuclear) ASAP. In the meantime, the straightforward primary solution to PO is to build out electric transportation. Whether we do it with .15KWH/km vehicles or .05KWH/km vehicles really doesn't matter, and the .15KWH/km vehicles will be a lot easier to sell.

Building renewables (wind, especially) is the easy way to eliminate fossil fuels. Drastic conservation measures are the hard way.

If wind was the easy way to replace fossil fuels, it would have been done, somewhere, by now. Even in The Netherlands and Denmark, the home of wind power, it supplies less than 25% of their electricity, on average. In the last 20yrs, since they started seriously building wind turbines, the fossil fuel use in those countries has not decreased. There are plenty of days when wind power supplies less than 1% of it;s installed capacity, so I'd say it is far from an easy way to replace fossil fuels. When the wind is blowing, it will displace fossil fuel, but there are no grid system where it has replaced fossil fuels, and there no indications that it can yet do so.

EV's do not necessarily support the build out of wind. On California summer days (and nights) there is often no wind, but those EV's still need to be charged. with intelligent charging timing systems, they *can* add load in off peak periods, generally, but that does nothing to change the inherent uncontrollability of wind power - there will be plenty of times when your EV is running of electricity from somewhere else.

The solution to PO is not just electric cars - it is reducing the need for transportation, and in particular, large, single occupancy passenger vehicles. There are many way to do this other than just EV's (trains, urban change, etc)- almost all of the alternatives are cheaper in the long run.

If wind was the easy way to replace fossil fuels, it would have been done, somewhere, by now.

That doesn't follow. Wind power in it's modern form is relatively new.

Even in The Netherlands and Denmark, the home of wind power, it supplies less than 25% of their electricity, on average.

Sure. Denmark wasn't trying to eliminate fossil fuels, they were trying to eliminate imported oil, and they were pretty successful.

When the wind is blowing, it will displace fossil fuel, but there are no grid system where it has replaced fossil fuels, and there no indications that it can yet do so.

And the same would have been said about France and nuclear, 50 years ago.

On California summer days (and nights) there is often no wind, but those EV's still need to be charged.

1) that's a problem for the EVs, not for wind. EVs will support the buildout of wind by providing demand for wind power. 2) do you have a source for CA wind data? It would be an interesting analysis.

- there will be plenty of times when your EV is running of electricity from somewhere else.

There may be a few, though as wind expands they will become fewer and fewer. Keep in mind that wind power still provides a very small % of CA power.

almost all of the alternatives are cheaper in the long run.

Do you have calculations to support that? I think if you look at all of the costs, you'll find that's not true.

Denmark wasn't trying to eliminate fossil fuels, they were trying to eliminate imported oil, and they were pretty successful.
They were trying to eliminate using oil for electricity, and yes they were successful, but so has been almost every other non oil exporting country. There is very little oil used for grid electricity in any OECD country. Denmark still uses 160bn cubic feet of nat gas per year, so the world's leading wind energy country has hardly been successful in eliminating fossil fuels for electricity - they largely substituted NG for oil.

Wind power in it's modern form (large three blade turbines) has been around for over three decades, and no country/grid has decided to run substantially from wind - because you can't. Yet, France decided to go nuclear when power reactors had been around for less time than that. The difference is that you can RELY on a nuclear reactor, and you cannot do the same for wind. Reactors have better than 95% availability. Average for wind is less than 30%, and, of course, when there is no wind, there is usually none anywhere in that grid, so you have to have 100% backup. With nukes you only need peak +20%.

EVs will support the buildout of wind by providing demand for wind power.
But what do they demand when there is no wind. Every new kW of wind needs a kW of something else to back it up, so it builds demand for both.
As for calculations, for california, there is this, from a previous post on TOD, which you have read;
http://europe.theoildrum.com/node/6418#comment-617890

Jim Detmers, VP of Operations for the California ISO, gave a speech at an energy conference at Stanford in 2007 where he said that wind is not so reliable:

About wind's unreliability he says: "Wind is not produced on peak. This last summer, when we went across the summer peak, I had 3,000 megawatts of capacity of wind. How much did I have on the summer peak, back in August? No, no, no, I didn't have zero. I had a total of 63 out of 3,000. And we're investing all of this money in wind..." (at around 10:15). Clearly, the wind let him down when he needed it most.

Later on, he talks about energy produced when it's not needed and how his capacity to store energy is limited. "I actually get to the point where I have to pay people to take the energy off the system. Get this straight: I am paying people to take the energy away from me because I am in over supply. Because wind comes up when I can't predict it, and it's all coming in off-peak. I deal with that system today. If we only increase that... help us. Please help us." (14:00). "The wind, this is the one that I like the most, because it presents the most challenges for me on the grid. And the train has already left the station. Contracts are already in place now to bring on about 7,500 additional megawatts of wind on to the system, and I have no place to put it off-peak." (17:30) "I hope everyone in this room walks away and clearly understands that wind has what I call an inverted supply curve. The maximum production of wind is off-peak. Do I need power off-peak? The answer is no! I'm already swimming in the megawatts today! Because I have to keep on-line all of those generators to be there when the wind's not there on the peak. I have to keep them on. Are we achieving and economic benefit from that? ... We may be costing [ourselves] an enormous amount of money... We're making decisions as an industry, as a state, as the United States and around the world because we want it so bad. And we're using existing technology, existing wind, but we're not marrying with that storage capability." (18:00)

The Midwest ISO at one point recorded 2% of nameplate output from wind during summer demand peak. So wind isn't always there when you need it.

EV's, with smart charging, increase demand for of peak electricity, of which there is already excess capacity. The mere prospect of them being charged on peak means additional non wind capacity is needed to handle them.

if we must go with wind+ storage+ smart grid, purely to allow millions of electric vehicles, then yes, the alternatives are cheaper. It is cheaper to densify cities, add electric rail transit, and reduce miles of road to be maintained than to do everything that needs to be done to maintain a road system and build up more grid + wind+ backup for EV's.

But what do they demand when there is no wind. Every new kW of wind needs a kW of something else to back it up, so it builds demand for both.

No it doesn't.  The peaking capacity and spinning reserve would be there anyway.  Up to about 20%, wind reduces the amount of fuel they use without requiring more generation resources.  We're about to see if energy storage can push that figure higher; the currently planned CAES systems will have a thermal efficiency (gas to electric) of 80%, which is about 80% greater than the best simple-cycle gas-turbine plants.  If this allows wind to supply 35% of grid power directly and another 35% via stored air at 80% gas-to-electric efficiency, the effective gas-to-electric efficiency is 160%.

Jim Detmers, VP of Operations for the California ISO....

Representing his major interests, namely fossil-fired generators and merchant plant operators.

EV's, with smart charging, increase demand for of peak electricity, of which there is already excess capacity.

Wrong.  Smart charging does not increase peak demand for electricity.  That's the whole point.

if we must go with wind+ storage+ smart grid, purely to allow millions of electric vehicles, then yes, the alternatives are cheaper. It is cheaper to densify cities, add electric rail transit, and reduce miles of road to be maintained than to do everything that needs to be done to maintain a road system and build up more grid + wind+ backup for EV's.

You don't seem to grasp that this isn't an either/or proposition, it is a both/and proposition.  EVs will be added to the Smart Grid and add benefits like massively increased spinning reserve and regulation.  Wind and storage will cut the amount of coal and NG required.  Electric rail transit will use the Smart Grid reserve and regulation resources to work better than historical systems could.  It's all good.

No it doesn't. The peaking capacity and spinning reserve would be there anyway.
If the spinning *reserve* is there anyway, then, by definition, that means there is backup.

Up to about 20%, wind reduces the amount of fuel they use without requiring more generation resources.
This is true, but he was talking about replacing fossil fuels - displacing a mere 20% of them is not the same.

We're about to see if energy storage can push that figure higher;
Energy storage IS backup generation. It's just that any other form of generation (except solar) doesn;t need it.

Representing his major interests, His major interest is delivering electricity. Regardless of that, the numbers are the numbers - there are times when wind makes zero, or very minimal net contribution to the grid, and these times are not regular or predictable, or controllable.
Unless, of course coupled with storage.

EV's, with smart charging, increase demand for of peak electricity, of which there is already excess capacity.
Sorry, typo there - was meant to read off peak electricity, as that is indeed the whole point of smart charging.

The smart grid doesn;t have to be an either/or thing, but then again, it could be. We don;t have it today, and can probably function for quite some time, if not indefinitely, without it. The people who are calling the loudest for it are the wind supporters. The fact is that for wind to be more than 20% of supply, you need storage + smart grid, so this makes wind very expensive.

Electric rail has been around for over a century without the smart grid. It uses such an inconsequentially small amount of electricity that it just doesn;t matter. We would be much better off to spend the smart grid money on rail projects instead.

Denmark still uses 160bn cubic feet of nat gas per year, so the world's leading wind energy country has hardly been successful in eliminating fossil fuels for electricity

Again, that wasn't their goal. Why cite Denmark, if that wasn't their goal??

Wind power in it's modern form (large three blade turbines) has been around for over three decades

That's either not true, or technically true but misleading. Wind turbines 30 years ago were much smaller and much more expensive. They weren't ready -they are now, which is why they're growing so strongly.

Reactors have better than 95% availability.

Not in France: the French capacity factor is around 80%, IIRC, and it would be 60% if they weren't part of a much larger European grid. The US is the best in the world, running very old and well-tuned reactors, and they get 90%.

Average for wind is less than 30%

New wind installations in the US are getting 35%. More importantly, that's not a flaw. That's part of the design. Nuclear is designed to operate at 90%, and wind is designed to operate at 15-45%, depending on the resource. Again, higher would be nice, but it's not a failure.

and, of course, when there is no wind, there is usually none anywhere in that grid

Not really true and, of course, that's why you want interconnections with other grids.

you have to have 100% backup.

Sigh. This is really not true. It might be true for one wind turbine, but wind farms rarely reach 85% generation, and regions will peak at much lower levels - geographical diversity is powerful. Second, Demand Side Management (aka demand response) is much cheaper and more effective than backup capacity.

With nukes you only need peak +20%.

Nuclear can trip in minutes, and be out for days (or much longer). Ireland has chosen not to consider nuclear, because they can't afford the 100% backup needed for a single 1GW plant. Sure, nuclear reliability is "good enough" for a large grid, but so is wind, properly managed.

As for calculations, for california, there is this, from a previous post on TOD, which you have read; http://europe.theoildrum.com/node/6418#comment-617890 - Jim Detmers....

As E-P noted, ISO's aren't really unbiased witnesses. Further, this is an anecdote, not a quantitative analysis. That's why I asked for calculations.

It is cheaper to densify cities, add electric rail transit, and reduce miles of road to be maintained than to do everything that needs to be done to maintain a road system and build up more grid + wind+ backup for EV's.

This is highly unrealistic. The current housing stock not served by rail (and too spread out to be served by rail) would cost perhaps $2 trillion to replace in dense cities. Further, it would take a very, very long time. EVs are faster by a factor of at least 5:1.

Why cite Denmark, if that wasn't their goal??
Because they are the leading wind producer. I'm sure they would get off Russian natural gas if they could. if they are not the country with the highest % of wind, they are close, and they are a leader in the wind industry - I think that makes them a very good example. However, I am willing to defer to you if you show me an example of any grid that has replaced fossil fuels, entirely by the use of wind, or at least has definitive plans to do so. I;ll even settle for one where more then 50% of its kWh comes from wind.

That's either not true, or technically true but misleading. Wind turbines 30 years ago were much smaller and much more expensive.
Cars in the 50's and 60's were smaller. less powerful, and relatively speaking, more expensive, than they are now - does that mean that modern cars did not exist then? Commercial airplanes were smaller and more expensive then, do we say that the air travel industry did not exist? Wind is growing strongly now primarily because of government subsidies, and renewable portfolio mandates. Where these don't exist, they are no growing much. Many of the turbines at Altamont pass were built in the late 70's, and then when the subsidies were stopped, nothing got built for decades, until they came in again. There is a definite causal relationship there, regardless of turbine size.

Reactors have better than 95% availability. -
Not in France: the French capacity factor is around 80%, IIRC, and it would be 60% if they weren't part of a much larger European grid. The US is the best in the world, running very old and well-tuned reactors, and they get 90%.

PLEASE understand the terminology before discussing it. Availability, in utility terms, means the period of time that the generator is *available* to produce electricity, *on demand*. Capacity factor, which I was not talking about, is the ratio of average power production to rated capacity, in other words, how much it is used, not how much it is available. Capacity factor is always less than availability. The fact that the French reactors operate at 80% capacity factor, does not mean that they could not do more. A car has an average capacity factor of about 5-10%, but has an availability of 98% (you would need to spend 2% of each day refueling, an EV this will be 10-30% depending on how you charge it).
Wind turbines do not have an availability in the traditional sense, as they cannot be called upon, on demand, which is their fundamental drawback.

And, of course, when there is no wind, there is usually none anywhere in that grid.
- Not really true and, of course, that's why you want interconnections with other grids.

So there is more expense, to interconnect grids, primarily to accommodate wind. Generally speaking, in summertime, wind production is less across the continent, and there are some weather situtations where it is minimal everywhere. But what about when you can't interconnect, like say Hawaii or New Zealand? A cable to somewhere else is impractical - what then do you suggest?

you have to have 100% backup.
Sigh. This is really not true.

Actually, it is true - you must have the ability to serve 100% of your peak loads with something other than wind. If you have a grid that is 100% wind turbines, and nothing else, how do you manage this? Storage counts as a backup, as that is exactly what it is, though an expensive one. Please show me an example, or even a plan for, a system, any system, that has more generation from wind than from all other sources combined, and what is the plan for when there is no wind.

Demand side management is a beautiful thing ( I work in this area, though mainly with water), but unless you have completely discretionary loads, for an indefinite period of time, it does not help if you are trying to get majority wind power production. Now matter how much you reduce your peak demand, lack of wind will reduce your peak generation faster.

Sure, nuclear reliability is "good enough" for a large grid, but so is wind, properly managed.
And just how do you "manage" wind to get power out of it when there is no wind? What does Ireland do on a windless day?

As for calculations, for california, there is this, from a previous post on TOD, which you have read; http://europe.theoildrum.com/node/6418#comment-617890 - Jim Detmers....

Yes, and in that were these numbers from LenGould;

In the past 12 months, the max output was 1017 MW, so there's at least that much online, quite widely distributed accross the 500 mile width of the southern part of the province near the great lakes (purportedly excellent wind resource territory).

On April 20th from 8:00 to 10:00 AM, the output averaged 3.5 MW. (0.34%)
On Mar 16th from 11:00AM to 1:00 PM, the output averaged 4.0 MW. (0.39%)
On Mar 9th from 10:00AM to 6:00 PM, the output averaged 6.7 MW. (0.66%)

So there is an 8 hour period where 1000MW of wind is producing 0.66% capacity. If that 1000 MW of wind was thew whole, or even half of the system, as you propose, then what? Without backup of some kind, the ONLY alternative is to reduce load to this level. To say just interconnect and get it from somewhere else doesn;t work either. In the case of Ontario, that means they need to source the remaining 99.33% of demand from wind from somewhere else. That means at least 2x the capacity and truly massive interconnections - very expensive, and *still* no *guarantee* that; a) the wind is blowing in the other system and b) they are willing, and have the excess, to sell to you. Statistically, there will be times when they can't, so then what? Can you provide an answer to that which does not involve backup generation?

Meanwhile, here is a good analysis of that data for Ontario..
http://mobjectivist.blogspot.com/2010/05/wind-energy-dispersion-analysis...
The graph, with thousands of data points, shows that for 45% of the time, the wind turbines are operating at less than 10% capacity, and for 10% of the time, they are less than 1.5% of capacity, which is effectively zero. And this 10% is not controllable - when it happens, you have to meet your demand (be it peak or off peak) from somewhere else. if this 1000MW of turbines is all the generation for a "grid" how do they then manage their power? Assuming we have complete ability to limit demand, what would be the average peak demand that could reliably serviced with this system. Reliably means 99% of the time - can you rely on imports in low wind periods that reliably? Would you want to? Are you willing to accept there will be times when you can't? All of these show why wind always needs backup. To be clear, if this was our "grid, I am not saying we nee 1000MW of something else, what I am saying is that if we said our 1000MW can service a peak of 100MW (55% of the time), we still need 100MW of something else, be it storage, imports or other generators. As long as wind is less than 20%, this isn't a problem, but when it's more than 50% it's the problem.


ISO's are the people whose job it is to match the power supply to the demand, and they do this all day, every day. They don't much care how the power is produced, just what it's characteristics and controllability are. It is the same as central traffic management in large cities. They couldn't care less who owns the cars or whether they are gasoline, diesel or electric, they just have to keep them flowing. If the ISO's don't match production to demand, blackouts occur, and they get sued. So their bias is towards maintaing reliable supply - I think that is reasonable. But since you don't, I'll be happy to look at data from some other system operator (not a producer) who has to balance these competing factors.

the current housing stock not served by rail (and too spread out to be served by rail) would cost perhaps $2 trillion to replace in dense cities.
If the city is dense, you don't need to replace the housing, just start building trains. Even if it is not really dense (e.g. Calgary) build the trains anyway, just pick the best places, people will use them. It makes it cheaper for them to live too, as they do not need to own a car (or two), and the railcars have a capacity factor of probably 50% whereas for cars it is 2% - which is, ultimately, a better use of resources? You can't serve everyone by rail, but you can sure serve a lot, as the European cities show us.

I am willing to defer to you if you show me an example of any grid that has replaced fossil fuels, entirely by the use of wind, or at least has definitive plans to do so. I;ll even settle for one where more then 50% of its kWh comes from wind.

No country is planning to replace all of their cell phones with land-lines - does that mean that cell phones aren't competitive? Of course not. No one is planning for a grid where more then 50% of its kWh comes from wind, because it's too soon. Probably that will never be a realistic goal, even with a zero-FF grid - diversity of energy sources is too valuable.

Cars in the 50's and 60's were smaller. less powerful, and relatively speaking, more expensive, than they are now - does that mean that modern cars did not exist then

That's a bad analogy. A better analogy is 1985 cell phones. They were expensive bricks, now they're taking over the world. Come now - are you being serious? Wind energy cost perhaps 5x as much per kWh 30 years ago - wind power then and now aren't seriously comparable.

Wind is growing strongly now primarily because of government subsidies, and renewable portfolio mandates. Where these don't exist, they are no growing much.

Roughly the same is true of all other sources. Nuclear wouldn't exist in the US without Price-Anderson, and wouldn't have started without a military application. Oil is getting a $500B subsidy right now, in the form of overseas wars. Coal gets enormous subsidies in the form of free pollution allowances.

More later....

Nick, since you just said;

Of course not. No one is planning for a grid where more then 50% of its kWh comes from wind, because it's too soon. Probably that will never be a realistic goal, even with a zero-FF grid

then why, earlier in this thread, did you say this;

Building renewables (wind, especially) is the easy way to eliminate fossil fuels.

This statement implies that focusing on wind can *eliminate* fossil fuels, yet have have admitted that this is not a realistic goal. That is my issue, I have no problem with wind per se, but to state that it can eliminate fossil fuels, is just wrong. It can be used as part of a system to replace fossil fuels, but we must recognise that such system must have the ability to serve peak load with zero contribution from wind power at certain times (like 10% of the time).
Wind can displace some, or even a lot of fossil fuel use, but , presently to say that it can eliminate it is not realistic. If we start out on that basis there is a lot less confusion. Wind could possibly even provide all the energy for all the ev's, just not all the time.

The higher the % of wind in your system, the more backup you need. If you were building a grid, from scratch, today, and wanted a lot of wind, you still have to have a lot of backup, and a lot of transmission lines, it is an inherently more expensive system to build. A train is useless without the tracks to go with it, and no one disconnects these two costs. But with wind, people often ignore the other costs that come with wind, if we want it to be a major generation source.

As for wind turbines in 1985, well, have a look at this; held it's value much better than a car from that era..

http://www.mywindpowersystem.com/marketplace/25-x-used-howden-300-kw-win...

Yes, the modern ones are bigger, better and cheaper, but so are modern CCGT's, and modern nukes, and modern coal plants, it's only a matter of degree. The first electric wind turbine was built in 1907, so they have been around for a long time, and their fundamental drawback, unpredictability, has yet to be effectively resolved.

As for subsidies, GT generation doesn't seem to be getting any, and it has been the fastest growing segment for the last two decades...
Agree about nukes, and the wars are about oil, which is not used for electricity, so is not relevant here.

Funny thing is, while American soldiers die in Iraq, China is buying oil assets in Canada - would have been much easier for the US to just buy them instead, but we can both agree that the war for oil (if that is the real reason)is stupid. I wonder, if given the stark choice, american people will choose higher gas prices/reduced supply, or more wars and dead soldiers...

A few quick thoughts,

-Don't forget about DSM - it's very, very cheap and effective,
-don't overestimate the cost and need for long distance transmission,
-nat gas also has emissions, though of course they're lower,
-all sources of generation have variance to some degree,and depend on a grid,
-the grid breaks up kWhs and capacity factor payments, so all sources get paid according to their contribution to capacity factor.

I'll add more later.

I am a very big proponent of DSM, and it goes hand in hand with time of day charging. I think identifying opportunistic loads, such as over cooling of cold storage, over heating of buildings and water, and even EV charging, are actually a better way to use wind than storage - store the work produced by the electricity. When I managed a small electric utility (a ski resort village) we were able to get to 25% of the load being discretionary - where we could turn it on and off when we wanted. This is better than permanent shifting of loads to off peak.

Generally speaking, I am not in favour of long distance transmission,if it can be avoided, as it is a cost, and you do lose some energy along the way. Energy miles is the same as food miles, the less you have to do the better. Also, having more local production keeps the money side in the local economy, and makes people more aware of their power sources. Relying on importing from somewhere else is not always reliable - the other grids will supply themselves first.

The emissions from NG are extremely low, especially from CCGT plants - 1/4 that of coal, I am not advocating this, but if we did nothing other than replace all coal with NG, we make major progress.

The difference of variability is whether it is by choice or circumstance. Controllable variability (e.g GT) is the highest value power, non variability (other than dump output) such as coal and nuke is next, and uncontrolled variability (solar, wind) is the lowest value. The article about wind power in germany confirms that, as wind sometimes has a negative value. Customers that can adapt their operations to maximise buying in these periods will do well, but not many are able to do so, and certainly not indefinitely.

Not sure exactly what you mean by "the grid breaks up kWh and capacity factor payments..

I think we're getting much closer to agreement here.

We see DSM the same way, I think. It's very, very cheap and effective, and should be used first before other things. That means that the cost of integrating wind power into a grid is much lower than it would be if we demanded large amounts of central-utility storage, long-distance transmission, and backup generation. This will continue to be true at much higher market penetrations because the same public-policy orientation that would make such market penetrations possible would also cause dramatic growth in EREV/EVs, which use DSM to provide night-time demand and soak up variance.

Yes, NG emissions are much lower than coal - I like NG. OTOH, we're still not charging NG for those emissions, which means it's being subsidized vs very-low-CO2 generators.

Yes, variance in a generator has a cost. OTOH, that cost is much lower than is generally appreciated, in large part because utilities like to build generation as a grid-stability strategy, due to ROI regulations, rather than use the almost-zero-cost DSM.

the grid breaks up kWh and capacity factor payments

In the US, generators are paid for their peak capacity factor, and paid separately for KWHs. That neatly accounts for the value of reliability, and means that one source of power does not subsidize another that is less reliable, as is often suggested by wind opponents.

Yes, the modern ones are bigger, better and cheaper, but so are modern CCGT's, and modern nukes, and modern coal plants, it's only a matter of degree.

Modern CCGT's, nukes and coal haven't fallen in capital cost by 80%. Actually, nukes and coal have gotten more expensive.

fundamental drawback, unpredictability

Actually, in the 1-3 hour time-frame that matters, wind is pretty predictable. Similarly, the solar diurnal cycle is very predictable, even if it does have a lot of variance.

As for wind turbines in 1985, well, have a look at this; held it's value much better than a car from that era..

That's a pretty poor thing to compare it to.  You might as well compare it to a loaf of Wonder Bread.

A nuclear powerplant of 1985 vintage is less than halfway through the 60-year extended lifespan, and is probably worth more in current dollars than when it was built.

As for subsidies, GT generation doesn't seem to be getting any, and it has been the fastest growing segment for the last two decades...

Jerome Guillet will write you a book on this if he sees it, because he puts it down to gross defects in the market and regulatory regime.  I'm not sure I agree with him fully, but I think he's far more right than wrong.

Availability, in utility terms, means the period of time that the generator is *available* to produce electricity, *on demand*. Capacity factor, which I was not talking about, is the ratio of average power production to rated capacity

Every where but France, with nukes, availability = capacity factor. Ontario used to have a surplus of nuke power at times, but that has gone away.

And even in France, with 12 GW of Swiss pumped storage coming on-line

And no nukes load follow (I have proven here on TOD that French nukes do not, despite their claims, from french hour by hour stats). So nukes have very little value except for base load UNLESS massive amounts of pumped storage is built. And pumped storage is exactly what wind needs too.

Best Hopes for a Rush to Wind, Pumped Storage and HV DC with an economic, reasonable build-out of new nukes (5 to 8 in the next decade for the USA),

Alan

Alan,

Here in BC, with lots of hydro, BC Hydro works with availability meaning the unit can be operated. In Alberta, Trans Alta (with mostly coal fired) uses the same terminology. Capacity factor represents how much the unit is operated.

In downtown Vancouver is an old style gas fired power plant, it is the most expensive power producer BC Hydro owns, and only operates during peak periods, so it's capacity factor is low, but it can be used anytime they want, so it;s availability is very high - they are quite different terms.
Any peaking plant must by definition, be available more than it is used,for if it used all the time it is available, then it is a baseload plant.

Nukes can, and do actually load follow, just in a wasteful way. They run at full output, and if you want fast/short term turndown, you simply waste steam. This is not "fuel efficient" but uranium fuel is actually very cheap (less than 0.5c/kWh).
Particularly for older style nukes, at night, they may just run one turbine, shut the 2nd one down and waste steam. Again, there would be a higher availability than capacity factor.
Coal plants do the same too, of course, it;s just that this is very wasteful, and pollutes air for zero benefit.

Much as I like pumped storage, it is very limited where it can be done, and often not in the places where the wind is. I think a much better way to go is to look at discretionary/opportunistic loads that can be switched on, or at least given preference, when the wind is blowing. Over cooling of cold storage is an example, as it over heating of water and buildings. Sometime the easiest way to store electricity is to store the product of its work.
Since the wind blows more at night, such an exercise to identify these loads will result in flattening of the load curve generally, which is a great benefit.

I cannot stand the repetition of these straw-man arguments:

If you have a grid that is 100% wind turbines, and nothing else....

then you're a nitwit, because nobody but the anti's ever suggested such a thing.  Being able to supply all ENERGY and having nothing else for GENERATION are two very different things, and nobody is even seriously talking about the first.

C'mon EP, you can do better than that. Before you start calling me names, re read above and you'll see this statement that Nick made, which started this thread;
"Building renewables (wind, especially) is the easy way to eliminate fossil fuels."

That clearly implies that lots of wind power can eliminate fossil fuels, and, by extension the fossil fueled generating plants. So Nick was talking about the first. Now in theory, wind can eliminate other generation, if you have one hell of a lot of storage, or lots of nukes, which, technically are not "fossil" fuels, though neither are they "renewable".

I am trying to get the point across that it is unrealistic to make these claims, that wind can replace fossil fuels for electricity, on a grid scale.

You can run a grid with 100% nukes, and not storage, or 100% coal, or GT, or (run of the river) hydro, without any storage. You would need total generation of about 120% of peak, and your capacity factor would probably be 40% overall. But try to do a 100% wind grid, and it illustrates what else is needed to reliably power your grid - lots of storage or interconnection with somewhere else, if that is available, and reliable.
As long as wind is a minority producer, and the utilities seem to converge at around 20%, you don;t have a problem, but if it is to be a majority producer, you do have a problem, You cannot decommision all, or maybe even any, of the non wind generation, and at least part what you do have must be reconfigured for peaking. Or you build interconnection, and hope they will sell to you when you need it - which cannot be guaranteed. The wind industry does not like to acknowledge these issues and certainly does not want to pay for them. If we are OK now without HVDC etc, but if we need this to get more wind, then wind should pay for it, or at least part of it.

I am not anti wind, I am pro- realism. Wind cannot generate all our energy needs, as you said, but other people ARE saying that, and not acknowledging the additional infrastructure costs needed for large scale wind power. If we factor in those costs, then we can make intelligent decisions about how to move forward. Personally, I think that large scale storage is trying to force a round peg through a square hole, better to try to find customers/applications that can have discretionary loads to use up excess wind power when available. Store the work produced, not the energy itself.

That clearly implies that lots of wind power can eliminate fossil fuels, and, by extension the fossil fueled generating plants. So Nick was talking about the first.

Well, no, I was talking about a system in which wind was a leading component. Right now, we can build wind ASAP without much worry about overloading the grid. Given the need for low-CO2 power, we should do that. In the long run, we'll need wind and solar, as well as many other things. Neither I, nor anyone else, is proposing more than roughly 50% wind. Of course, that's enough to eliminate 95% of coal.

Now in theory, wind can eliminate other generation, if you have one hell of a lot of storage, or lots of nukes, which, technically are not "fossil" fuels, though neither are they "renewable".

We agree that DSM is much more important than storage, as you say lower down. That's much, much cheaper.

Before you start calling me names

First, I didn't apply that label to you, I applied it to your hypothetical designer (I'd accuse you of erecting a straw man).  Second, if we built an electric system on principles like the pre-REA wind energy systems sold in the Great Plains (e.g. a freezer capable of keeping ice cream frozen for 3 days without power), we would be able to use lots of intermittent energy and we'd think nothing of it.

Obviously, that's not what we've got to work with now.  We're going to reach a different result because choices made then have eliminated some options... and created others.  We can still manage.

As long as wind is a minority producer, and the utilities seem to converge at around 20%, you don;t have a problem, but if it is to be a majority producer, you do have a problem

You don't seem to realize that storage is a boon for several different generation technologies.  Today's nuclear likes to run at 100% for 18 months and then refuel; to service peak loads cheaply, you'd like to store power off-peak and use it on-peak.  The same sort of storage plays well with wind also.  But it's a direct competitor for gas turbines, so you can see that it's a political problem too.

Store the work produced, not the energy itself.

I've been touting the Ice Bear for years.

EP, I'm well aware of the benefits of storage - I live in a province (British Columbia) that has huge amounts of it, and BC Hydro takes full advantage of that. BC is actually a net electricity importer (by about 15%) but makes an annual net profit on its export and import operations. Would have actually made more still if California paid its bill, but that's a different story,

What I am saying is that there are substantial costs, and efficiency losses, associated with feed-in storage, which includes pumped hydro. When it comes to wind, unless it is matched with either storage or dispatchable peak power generation from another source, it is not dependable.
At least with nukes, or any others, you (generally) know when that maintenance shut down is coming.

When it comes to wind, unless it is matched with either storage or dispatchable peak power generation from another source, it is not dependable.

A couple of thoughts - storage or backup aren't necessary if DSM will do the job.

2nd, I think it would help to take a more statistical perspective here. If a system is 99.999% reliable, that's good enough. That's how all other generation sources are handled - statistically.

99.999% is indeed good enough, but wind is not anywhere near that, and that is the problem. For Ontario;s 1000 MW of wind, for 90% reliability, the level is 14MW, and that is four orders of magnitude less reliability than what we want. Not a problem as long as the total peak demand can be met from other sources, but as soon as we start decomissioning other generators, and get to the point where (non-wind generation) is less than peak, we need to rely on wind, and then we have a very low reliability. You can improve it storage and so on, but the inherent reliability is still less than 99.x %. That is where I am coming from - wind is fine as supplemental generation, but you must always have enough other generation to meet the peak when there is no wind, is this will happen at least 10% of the time.

The dependability statistics are not kind to wind.

We make up for it either by building huge amounts of wind, or storage, or interconnection, or not decomissioning other plants, but it has to be made up for somehow.

Paul,

How did you come up with "For Ontario;s 1000 MW of wind, for 90% reliability, the level is 14MW"? What time period, which data, and what kind of analysis?

I've downloaded their stats in the past, but I haven't seen good data on their nameplate capacity, and when each increment was added.

I took 99.999% out of a hat as an arbitrary example - that's not what ISO's use when they analyse wind. They tend to develop peak capacity factors of between 10% and 20%, which seems to me really isn't bad compared to average output around 30%.

you must always have enough other generation to meet the peak when there is no wind, is this will happen at least 10% of the time.

That's the case for a very small wind resource, where there isn't much opportunity for "law of large numbers" effects. Ontario, for example, is a pretty small wind wind resource, and most of it is concentrated in a small area (much smaller than the overall province).

We make up for it either by building huge amounts of wind, or storage, or interconnection, or not decomissioning other plants, but it has to be made up for somehow.

Well, the first thing to use DSM, which is almost free.

For Ontario;s 1000 MW of wind, for 90% reliability, the level is 14MW,

That's your claim, but the facts appear to be rather different:

26.6% capacity factor means it's probably running at 1.4% much less than 10% of the time.  You're also misusing reliability; the grid as a whole needs to be 99.999%, which is achieved by massive parallelism and redundancy.  Wind fits in just fine up to 20% or so even without storage.

You also seem to be misusing the concept of capacity factor, which you'll find illustrated here.

DSM is a huge factor.  For instance, consider the possibilities of smart water heaters.  Adding a little computer on the gas valve would let your heater figure out when you need hot water.  Add a 3.3 kW electric element to the cold-water tube on a gas water heater under control of that computer, and it can substitute electricity for NG.  Give the computer info about the forecast wind power price and supply, and it can level the net power supply by consuming the excess.  It can leave the gas off until the wind rises, and then heat your water for less than you'd pay for fuel.

That's just one small thing with big possibilities.

EP, See my comments downthread to Nick about the wind data. Upon shortening the data set to just the last year (to exclude data from when there was less than 1017MW),there is actually 2.2% of the time when it is at 1.4% or less, and it is at 10% or less production for 24% of the time. The average production is 279MW, for a capacity factor of 27.4%.

But when you are trying to operate your grid, the average is meaningless, it is the instantaneous that matters, and 24% of the time, they have less than 101.7 of their 1017MW of wind on hand.

I do not discount DSM I make my living doing it (for water supply), and have worked with BC's DSM program extensively. It is almost always cheaper than the cheapest new generation.

But the point here is not about DSM, its about wind, and I am saying that wind cannot effectively replace traditional generating plants. It can, depending on the nature of the plants, reduce the time they run, and fuel they use, but it can't decommission them. That can only be, reliably, replaced by either storage or other dispatchable generation, or DSM.

There are just too many times when the wind doesn't blow hard enough to depend upon.

Please see my other recent reply.

I should be studying arcana or sleeping, but I'll try to answer you.

when you are trying to operate your grid, the average is meaningless, it is the instantaneous that matters

Okay.  Let's suppose a couple of things:

  1. Generation which can go from 100% to nothing in hours is paired with storage systems which can go from 100% charge to 100% discharge in seconds.
  2. On-demand systems for things like A/C and DHW are largely converted to storage systems with hours to days of storage.

What then?  My calculation for the USA some time ago was that electric vehicles could take roughly 45% of existing electric generation (180 GW increase over ~450 GW average).  If that could be deferred or advanced by even an hour, let alone a day, the storage potential would be enormous.

24% of the time, they have less than 101.7 of their 1017MW of wind on hand.

And 76% of the time, they have as much or more.

If carbon cost money, it might be cost-effective to pair a well-insulated solar/electric (depending on the season) DHW tank with a small gas-fired instant heater to bring the water temperature up to spec.  The shower and bathtub could be plumbed to take the RE-heated source direct if it was warm enough, and perhaps the washing machine also (3 inlet valves instead of 2).  If a lot of the DHW demand could be satisfied by electricity from wind as much as 24 hours ahead, the DSM capability of the system would be quite large.  If the sun shines or the wind blows, your water is hot; the displaced gas is there to provide electricity when all else fails.

Electric vehicles can do the same.  If they're plugged in when parked, their juice can come anytime before it's needed.  A Volt-class vehicle has about 8.8 kWh of usable storage, for about 40 miles/charge of all-electric range.  If it's 20 miles to work and you're one of 80 million commuters, that's 352 GWh of demand which can be deferred as much as 24 hours.  That's about 6-7% of total US electric power demand right there.  If the wind is up, you charge everything to 100%; if it's down, you run enough other generation to give everyone the minimum for the next trip.  That's DSM.  What's the problem with that?

What's the problem with that?

Well, it's an impressive scheme, and I've no reason to doubt your numbers (I rarely do), but whether they can be achieved, and at what cost, is the question.

Pairing generation and storage is obvious, just give me a price (either to build or per kWh delivered) and effective output and duration curve for your paired system. Really this means the characteristics of the storage, the generation type is now irrelevant, but it still costs.

Dual fueling for hot water is also a good one. I beat my head to a pulp with a former employer trying to convince them of the merits of this for a heated pool facility, to be able to take advantage of the cheapest fuel at any time, and flatten my electric utility's load curve. But then you need two HW systems, and capital cost was king. I'm sure we can do better today.

Now, for the electric vehicles, what we really have is a (distributed) utility scale battery storage system, on wheels. So we need 350GWh of storage, and 180GW of grid tie inverter capacity, and a lot of charging points. Any EV that is all DC will then need to have an inverter aboard for this reason only. WE need three charging points per vehicle there are, on average, three times as many parking spaces in a city as there are vehicles) .

Hmm, a lot of infrastructure cost there, and if I decide to divert somewhere on my way home from work, on a wind-less day, I can;t because the V2G has run down my EV's battery. if all the cars are Volt hybrids, then that is different, but that means we have effectively a 100% fossil fueled backup for each car.

Which comes back to my original point - after all the DSM is done, be it for the grid of the Volt, you still have to have 100% backup to meet the remaining load.
And given that wind capacity will likely increase much faster than EV market share, what then do the grid operators do when the coal plants start getting decomissioned? I think they will demand the wind suppliers provide their own backup, by whatever means, so that it can be turned on or off the same as any other power source.

we need 350GWh of storage, and 180GW of grid tie inverter capacity, and a lot of charging points.

Why so much grid capacity?  The 16 kWh of storage in the Volt only needs Level 1 (110 VAC @ 12 A avg, 1.32 kW) to maintain it, even with some DSM variations.  Supplying e.g. a Nissan Leaf with 50 miles of juice over an 8-hour working day would take about 1.3 kW average, which is a lot less than a C/2 charging rate of 12 kW (assuming a 24 kWh battery).  You might want to have a 220 VAC 30 A connection (about C/4) for DSM purposes.  Most times you'd be running at a small fraction of that.

Any EV that is all DC will then need to have an inverter aboard for this reason only.

There aren't that many EVs which are all DC.  The AC-150 drivetrain's reductive charger has a 3-phase inverter and can use it plus the motor's windings to connect to the grid, for either charging or V2G.  It's just extra software.

if I decide to divert somewhere on my way home from work, on a wind-less day, I can;t because the V2G has run down my EV's battery.

You didn't mention V2G before, and only V2G for peaking would affect the battery charge significantly; grid regulation varies charge by perhaps 1%, which you'd likely never notice.  Using DSM to manage the difference between minimum charge (plus safety factor) and full charge is a horse of a different color.

after all the DSM is done, be it for the grid of the Volt, you still have to have 100% backup to meet the remaining load.

Sure.  And if you've saved the fuel to meet that instantaneous load during the smaller periods it has to come from fuel, you're all set.

given that wind capacity will likely increase much faster than EV market share, what then do the grid operators do when the coal plants start getting decomissioned?

Rely more heavily on NG turbines, I suspect.

"why so much grid capacity?"

I was just using your 180GW, but if you want to use less, than fine.

I am not convinced that once ev's, or phev's are mainstream, everyone will accept slow charging, even with differential pricing, same as not everyone cares about gas mileage (or they wouldn't buy all those F-150's)
Article like this suggest there might be a trend towards faster charging
http://blogs.edmunds.com/greencaradvisor/2010/05/first-commercial-fast-c...
Part of the reason for a car is the personal freedom, and the recharging time was a big negative 100 years ago, I think many people will take the fast charge option, if available. Short charging ability will be a selling feature, and and a good portion of people will use it.

As for V2G, I had assumed that what was what you meant, but on re-reading, you are just talking about the charging side, not discharging, indeed a different beast.

But it still boils back to what demand has to be met by "fuel" sources, an yes, IF you have saved the fuel, then you are OK. Problem is, I don't yet see a lot of fuel being saved. I am with you that NG turbines will likely be the choice for the near future, but given the increasing peaking demand, they may not be CCGT, and so then they are less fuel efficient (but more profitable). When the offpeak wholesale rates in Germany have gone negative from all the nighttime wind, and Bonneville is telling wind turbines to shut down at night, what is the point of paying the extra for combined cycle?

A major development in storage may change the picture for wind, but I don't know what it would be. I think the real problem for wind is that if they rely on everyone else to do stuff (storage, HVDC etc) it won't get done soon enough, (and EV's wont; become large enough, soon enough) and if they do it themselves, they are uneconomic - a standoff is looming there.

DSM stands out as being the best value for money, and there is no shortage of things that can be done, but getting customers to agree to do it, even when it's free, is still not a sure thing. And there is still the question of who pays for it.

You're missing the point. Read this please: http://www.lowtechmagazine.com/2009/11/renewable-energy-is-not-enough.html

Enough with the straw-man arguments already.  You won't even quote the summary at the top, which happens to be factually wrong (numbered listing added):

  1. Increasing the share of renewable energy will not make us any less dependent on fossil fuels
  2. as long as total energy consumption keeps rising.
  3. Renewable energy sources do not replace coal, oil or gas plants, they only meet (part of) the growing demand.
  4. The solution is simple: set an absolute limit to total energy production.
  5. Why should we not be able to cope in 2030 with the amount of energy we consume today?

Going over it point by point:

  1. It's already doing so in places like Aruba and Iowa.
  2. At least in the USA, RE is rising while coal consumption is falling.  Electric generation from coal fell particularly rapidly last year, while generation from natural gas increased.  This puts the lie to the claim that "electric vehicles run on coal", which you made and then tried to evade by changing the subject to all fossil fuels.  Of course, nuclear power runs EVs just as well.
  3. Not disputed, also not particularly relevant or truthful.  RE in the USA grew as total electric consumption shrank.
  4. An absurd prescription.  Why should energy from wind or solar be limited to deal with fossil-fuel issues?  This is the wet dream of the haters of industrial civilization.
  5. The answer is simple:  because there is no need to do so, and no reason we should want to.

Factors like the higher efficiency of electric systems over fuel-burning may make it worthwhile to use less energy overall, but given the fantastic abundance of energy from non-combustion sources there's no reason to even think about it as a goal.

Enough with the straw-man arguments already. You won't even quote the summary at the top, which happens to be factually wrong (numbered listing added):

Everything you write following this quote above betrays that you *only* read the introduction of the article. And you simply refuse to even consider the possibility that it might be true.

What do you want me to do then? Past the whole article here?

Nevertheless, the introduction to your article is sufficiently confused that Engineer-Poet's post was justified. Few things are as counter-productive as dismissing rapid alternative energy development because it has not been able to keep up with more rapidly increasing energy use. We do have to cut down on our fossil fuel energy usage, but the simple political fact is we haven't yet implemented the useful ways of doing this (carbon taxes or carbon cap-and-trade), and no one will ever implement notions like your suggested capping total energy usage. So your introduction ensured that anyone who didn't already agree with you never makes it past the introduction.

Everything you write following this quote above betrays that you *only* read the introduction of the article.

This leaves exactly two possibilities:

  1. The author (you) wrote the introduction to mislead about the conclusions.
  2. The article was thrown together through some combination of sloppy authorial and editorial work and is not coherent.

Neither reflects well on you.

And you simply refuse to even consider the possibility that it might be true.

No, I considered it and found it completely implausible.  For instance, if the projection of 72 TW of available wind power in the world is even half true, it rubbishes every claim you've made.  That's ignoring the potential of solar and nuclear.  The "power down" position can only be held by people mired in ignorance, whether accidental or willful.

This leaves exactly two possibilities:

1. The author (you) wrote the introduction to mislead about the conclusions.
2. The article was thrown together through some combination of sloppy authorial and editorial work and is not coherent.

Neither reflects well on you.

The introduction is correct. You will realize that once you finally read the article, which you obviously still did not do.

Sentences 3 and 4 of the summary are indeed, as the poster above noted, an opinion. Sentences 1 and 2 are facts. They are explained in the text.

On a side note, I think your way of argumenting is below all levels. And yes, I realize you are a contributor here at TOD.

4. An absurd prescription. Why should energy from wind or solar be limited to deal with fossil-fuel issues? This is the wet dream of the haters of industrial civilization.

Because it takes fossil fuels to manufacture solar panels and wind turbines. If renewable energy replaces fossil fuels, this is not a problem. Then we actually do save energy. If renewable energy satisfies additional demand, then the fossil fuels used to manufacture the solar panels and wind turbines add more use of fossil fuel energy. We become not less but more dependent on fossil fuels.

1. Increasing the share of renewable energy will not make us any less dependent on fossil fuels
2. as long as total energy consumption keeps rising.

As a way of illustrating your dishonest way of argumenting: why do you cut the first sentence of the summary in two and then attack both parts separately? Talking about setting up a strawman...

it takes fossil fuels to manufacture solar panels and wind turbines.

Aside from resins required for fiber composites (which will probably come from petroleum for quite some time), there's little in either which needs fossil fuels.  Iron used to be made with charcoal, and iron oxide can be reduced with hydrogen from any source.  Most of the steel used in the USA is reclaimed from scrap; all it takes is an electric furnace to re-melt it, and the electricity can come from anything.

1. Increasing the share of renewable energy will not make us any less dependent on fossil fuels
2. as long as total energy consumption keeps rising.

As a way of illustrating your dishonest way of argumenting: why do you cut the first sentence of the summary in two and then attack both parts separately?

Because there are prominent counter-examples which prove that the first half is false, which removes the basis for the second.  One of the biggest is US electric generation from 1978 to present.  The USA made a lot of juice with oil until the second OPEC price shock.  That fraction tumbled steeply after 1978, most of it replaced by nuclear power (which now generates more than twice what oil ever did).  US electric generation from oil is now about 12% of its peak, and most of that in places like Hawaii where common substitutes are difficult to apply (note that the total includes petcoke, which is a coal-like waste product of thermal cracking of heavy fractions and isn't really oil).

That same table shows that US wind power in 2008 was about where nuclear power was in 1972.  Note also that nuclear didn't just replace oil and stop; it exploded until it now accounts for 2.2 times as much as oil ever did.  That's despite concerted political opposition and public hysteria.  Another nuclear expansion is in its early stages, and this time it will be joined by wind.  All you need for alternatives to expand at the expense of fossil power is for them to be cheaper and have good certainty of ROI (which requires a reasonable regulatory environment).

As a way of illustrating your dishonest way of argumenting: why do you cut the first sentence of the summary in two and then attack both parts separately?

You claim "If A, then B", and I'm refuting B by showing A is false.  If you don't like me using Aristotelian logic against your argument, fix your argument.

Total energy consumption is irrelevant.  What's important is fossil energy consumption, and claims to the contrary look dishonest to me.  The argument that conversion from e.g. petroleum to electric is undesirable also looks dishonest.  There are many reasons to do so, not the least of which is that it is an enabling technology for DSM and an expansion of intermittent RE to well beyond 20% of grid supply.  For instance, Denmark used 181,100 bbl/day in 2008 (down about 5% from 2007); at 6.1 GJ/bbl, that's 12.8 GWth.  If half of this is motor fuel burned at 30% efficiency, it represents a load of 1.92 GW or 16.8 TWH/year.  That's about 45% of Denmark's electric consumption, and if it goes into EV batteries most of it isn't time-critical (it can be supplied at most any time over the previous 24 hours or so, as long as cars plug in when parked).  This schedulable-load property of EVs is perfectly suited for wind power, which could expand from about 20% of Denmark's current supply to 46% of a much larger total even without the added fossil displacement allowed by the greater production under low-wind conditions.  That's one way that increasing the renewable share can cut fossil consumption.

When you make claims which are contentious and likely false, it appears that you've got a hidden agenda.  Some doomers don't just predict the collapse of industrial civilization, it is one of their goals.  Pushing against replacement of oil by electricity is a way of guaranteeing contraction and possible collapse as oil supplies decline.  You think I'm suspicious of your motives?  You're damn right I am.

Aside from resins required for fiber composites (which will probably come from petroleum for quite some time), there's little in either which needs fossil fuels.

I don't think that point should be conceded - resins may be mostly conveniently derived from oil for a long time, but that's not "necessary".

Most of the steel used in the USA is reclaimed from scrap

And, of course, when these industries become mature, 99% of their steel will come from scrap.

US electric generation from oil is now about 12% of its peak

In the last two years it's declined by about another 50%, to about .8% of all electrical generation.

Have Hawaii and Puerto Rico built coal, propane or other power plants ?

Those two are the intractable sources of oil fired electricity for the USA.

Alan

Hawaii is only .2% of US electrical consumption, and only has about 67MW of oil fired generation.

http://www.eia.doe.gov/state/state_energy_profiles.cfm?sid=HI

According to your link, Hawaii has 47.9% of US oil fired electrical generation. PR does not count towards US totals in EIA stats apparently.

Alan

Yes, I don't remember seeing PR in EIA stats.

They probably have good trade winds - they should imitate Aruba and Guantanamo and install some wind power.

It surprised me that it actually doesn't look all that good:

That's too bad.

Now, that's at 50 meters, not 80, so it might just improve with an updated wind survey.

Aside from resins required for fiber composites (which will probably come from petroleum for quite some time), there's little in either which needs fossil fuels. Iron used to be made with charcoal, and iron oxide can be reduced with hydrogen from any source. Most of the steel used in the USA is reclaimed from scrap; all it takes is an electric furnace to re-melt it, and the electricity can come from anything.

The reality is that solar panels and wind turbines are produced by electricty made from coal or gas. Of course electricity can be produced by wind turbines or solar panels (in their turn produced by electricity made by solar panels and wind turbines, and so on). The point is: this is not what is happening. We prefer to use the electricity produced by solar panels and wind turbines to power an ever increasing plethora of electronic gadgets, for instance. Not to green the existing electricity infrastructure.

You claim "If A, then B", and I'm refuting B by showing A is false. If you don't like me using Aristotelian logic against your argument, fix your argument.

OK. I will fix my argument (additions between brackets) :

"Increasing the share of renewable energy will not make us any less dependent on fossil fuels as long as total (fossil fuel) energy consumption keeps rising (faster than renewable energy consumption)."

Happy now? In the article you refuse to read, I prove that this is true for Spain, the Netherlands, the US and the world as a whole. It might not be true for Hawaii or Aruba, but who cares?

If you look at the orginal sentence in isolation, and you're a hair-splitter, you could indeed argue that it is not entirely correct. But it is immediately followed by this one, and the combination of both sentences rules out any ambiguity: "Renewable energy sources do not replace coal, oil or gas plants, they only meet (part of) the growing demand".

You're just trying to distract attention from the thing you prefer not to discuss.

The reason why we don't agree is that I talk about what is happening in the real world, and you talk about things that might one day become true. Generating 72 TW of wind powered electricity might be theoretically possible, but it is not happening. And even if it would be a reality one day, it would not take us any step further if fossil fuel powered electricity grows to 150 TW by the same time.

Kris,

I hate to repeat myself, but...we don't seem to have made progress, so I will.

First, we're still in the early stages of growth for wind and solar. Just because they aren't supplying all of new generation, or starting to replace coal and other FF's, doesn't mean that they won't be able to in the near future.

2nd, the wind power required to power an EV like the Leaf forever would cost about $2,000. That's a one-time cost.

Building renewables (wind, especially) is the easy way to eliminate fossil fuels. Drastic conservation measures are the hard way. If we can't get the overall society to go with the easy way, it's highly unrealistic to think that we can get it to follow the harder path. Why would we tell people to accept a greatly inferior vehicle to save $500 or $1,000? Why would we try to fight such a difficult, unnecessary battle? If we want to eliminate FFs, it's not that expensive. If we don't have the will to do even that much, we certainly aren't going to be able to sell a conversion to very small, low-powered EVs.

3rd, EVs actually support the buildout of renewables, especially wind, by providing night time demand and soaking up intermittency. We don't have to choose between EVs and the reduction of emissions - the two goals are actually synergistic.

The straightforward solution to AGW is to build out wind, solar and nuclear (if you like nuclear) ASAP. In the meantime, the straightforward primary solution to PO is to build out electric transportation. Whether we do it with .15KWH/km vehicles or .05KWH/km vehicles really doesn't matter, and the .15KWH/km vehicles will be a lot easier to sell.

Building renewables (wind, especially) is the easy way to eliminate fossil fuels. Drastic conservation measures are the hard way.

We need to do both things. One of each is not enough.

I hate to repeat myself, but...we don't seem to have made progress, so I will.

We disagree, Nick. It's no use to repeat yourself. It's even more useless to copy/paste an earlier comment. Just try to live with the fact that people can disagree.

We need to do both things. One of each is not enough.

It would be desirable to do both, but it's not realistic.

People can buy a $12k vehicle which gets 35MPG. They're willing to spend $15K more upfront, and another $1,500 per year to get something larger. Doesn't it seem obvious that people will be more willing to spend $10/mo more on electricity to aggressively build out wind? Until we get people to that point, why push the other?

Further, EV electrical demand is supportive of wind, right?

Finally, I'm not sure I really buy the premise at all. We could eliminate both 95% of coal electrical generation with wind, and 90% of personal transportation oil consumption with EREVs in 20 years - that's pretty good.

We disagree, Nick. It's no use to repeat yourself. It's even more useless to copy/paste an earlier comment. Just try to live with the fact that people can disagree.

Much better to have a good dialogue, and move towards agreement. Why not consider what I said, and reply to it?

It's no use to repeat yourself. It's even more useless to copy/paste an earlier comment.

If you had a counter-argument to the synergy of intermittent RE sources and EVs, it would be very worthwhile if you'd lay it out.  The people arguing for Liquid Fluoride Thorium Reactors (LFTRs) wound up convincing me that they had something very important.  If there is no such argument in favor of your conclusions, maybe they aren't worth your advocacy.

People can disagree about many things, but nobody's entitled to their own facts.  We should be able to agree on those.  So far, I don't see it.

EVs will increase peak, not night load.

Joe and Jane 6Pak will plug in as soon as they get home at 5:30 to 6 PM.

6 PM weekdays is a nearly universal primary or secondary peak.

Alan

Boy, we have the same conversation over and over again.

Joe and Jane 6Pak will plug in as soon as they get home at 5:30 to 6 PM.

Not if they have a PHEV or EREV, and time of day pricing.

6 PM weekdays is a nearly universal primary or secondary peak.

Sure. It's purely the artificial result of flat pricing. Commercial/Industrial users have a primitive form of time-of-day pricing,and as a result steel mills operate at night.

Joe and Jane 6Pak will plug in as soon as they get home at 5:30 to 6 PM.

When they plug in and when the charger switches on are two different things.  If Joe & Jane need to run errands at 7, they may be willing to pay 30¢/kWh to charge right away; down the street, Dawn and Don will sell them the remaining charge in their battery at 25¢/kWh, and recoup it at 7¢/kWh in the wee hours.  It all balances out.

the wind power required to power an EV like the Leaf forever would cost about $2,000.

I'm not sure what your assumptions are, but:

  1. Average 30 miles/day (mostly commuting)
  2. 250 Wh/mi
  3. 35% capacity factor for wind
  4. $2000/kW nameplate

I get 312 watts average, and $1790 in generation to supply it.  So it's a tad less than that.

Carry on.

I've usually used 30% capacity factor. You're right, though - recent US projects have averaged 35%, so maybe I should indeed use that.

That's all well and good, but now, as the system operator, I want you to give me a dependability factor. Using the Ontario wind distribution, the dependability is as follows;

95% of the time - 0.8%
90% of the time - 1.4%
80% of the time -3%
70% -5%
60% -8%
50% -10%

For any other system the 95% dependability is 100%

So for half the time, I can only rely on 10% output from your wind farms. As has been said, you can make this up with storage, etc, so now add that factor into your nameplate capacity, to create a "wind based system", and tell me what is the power output that can be depended upon for 95% of the time, and what your new "nameplate" cost is.

For example, if you just want to make it up by sheer volume of wind generation, you need 100/0.8 = 125x the capacity. So a "dependable" 100MW needs an eye watering 1250MW of installed capacity, and the "nameplate price is now 125x$1700x100=$21bn! And this doesn;t include the transmission lines that may be needed.

so clearly, that is an expensive way to make it dependable, any storage is cheaper than that, but how much. Factor that in to the nameplate price and then you have something that can meaningfully start to displace other plants and decommission them. Until then, those coal plants will hang around, and if that's the case, they will likely stay operating too.

Paul,

As I asked above, how did you come up with this distribution? What time period, which data, and what kind of analysis?

I've downloaded their stats in the past, and I found that the median output was about 45% of average. I think it's more useful to use average output as a benchmark: nameplate is misleading, as that's not the design output.

To make a useful comparison we really need to know what the duration of the very low generation periods are. It is obviously easy to use demand management and pumped storage to deal with gaps on the order of minutes or hours but 10s of hours or days (or the winter/summer difference for solar) is much more of a problem.

I'm unsure of how true this is but one problem raised for the UK is that we get some winter periods of high demand from low temperatures, little wind and overcast sky that can last days - this would be a test of any storage or power import system (cut offable industry and keeping some gas (maybe stored bio gas?) back-up for these situations would seem a must).

I have to agree with other comments on this thread though - if its not BAU it must be doom seems absurdly simplistic at best.

I agree - a useful analysis would require a simulation of various strategies, rather than a simple statistical analysis using just one or a small number of statistical parameters.

One thing to keep in mind: if serious outages are very rare, than you probably want backup sources that are much less expensive per unit of capacity, even if they're much more expensive per kWh. For instance, if 220M EREV vehicles participated in V2G, the ICE vehicle backup generators could power the entire grid for a week. They'd only be about 25% efficient, but it wouldn't matter.

Another key point: you're not likely to have to backup the whole grid. If, for instance, we had a grid like this:

Average KWH market share:
40% wind
25% nuclear
20% solar
10% hydro, wave, other
5% biomass peakers

The renewable sources are unlikely to disappear entirely; the nuclear will be constant; and the biomass peakers could be cheap and have only 10% average utilization, but have peak capacity of 50% of the grid.

It really wouldn't be hard to make a non-FF grid work.

So now put yourself in the chair of the grid operator. They are not so worried about kWh, as that is just who gets paid what, they are worried about kW (demand) and making sure they can meet it.

The Ontario grid currently has a peak demand of 25,000MW and 34,000MW of non wind generation available, or 36% extra capacity, and lets assume this is a desirable safety margin (their extreme hot weather scenario shows that it is, just)

For that hot, still, summer day, we assume wind contributes near zero, which has happened for 8 hour stretches, so we need the non wind part to be 136% of demand. With your mix, we then have;
peak demand = 100/136*(25%+20%*50%+10%+5%) Note this assumes solar is at 50% on at this period (early evening, high temps, low sun)
result is peak demand is 37% of grid capacity.
In other words, we have gone from needing 36% overcapacity to needing(1- 100%/37%)= 170% overcapacity, to maintain our system reliability.

For the Ontario system then, to meet our 25,000MW demand, we need the non wind and half the solar to add up to 35,000MW, and that is 50% of your grid capacity. So the grid now looks like;
Wind 28,000MW
Solar 14,000MW
All Other 28,000MW
Total Capacity 70,000
Peak Demand 25,000

So we brought "all other" down from 34,000 to 28,000MW, or 6000MW, by building a total of 42,000MW of wind and solar.
With your nameplate price of $1.4k/kW for wind, that is $392Bn and for solar, with a nameplate price of $6k/kW we have spent $840bn!
This is an eye watering total of $1.23trillion, to replace 6,000MW of fossil fueled generation. That is $205k/kW!

So, it is indeed FAR cheaper to do DSM, which is typically $1-2k/kW

So, my original point was, and still is, that wind cannot effectively replace dispatchable peak load power generation. It can, over the course of time, save fossil fuel, for sure, but to maintain system reliability, it cannot be relied upon. The only way is by adding complementary storage (which I then consider to be dispatchable generation, and can store power from any source, not just wind), or reducing demand.

Even if EV's spur the construction of wind turbines, if the increase Ontario's peak by 1000MW (and off peak by say 10,000MW), we cannot rely on said wind to meet more than 1% of this additional 1000MW.

The way I see it, wind power (alone) can displace fuel used, but can;t close down other plants. if you want to close them down using wind, then you need to couple it with storage, and you need to settle upon how many kW of wind+ storage is needed to replace a kW of dispatchable generation. Then work out your nameplate price - my guess is it won't be less than nuclear

Paul,

There are several basic things we need to discuss here, and come to agreement on (I hope). These include DSM; capacity factors versus capacity credits; the law of large numbers; geographical dispersion; and the difference between KWH market share and nameplate capacity. Now, it's tempting to try to deal with all of these, but as a practical matter conversations that sprawl all over the place don't seem to make any progress. So, let's try to deal with one thing at a time, starting with DSM.

First, it doesn't make sense to consider the grid design without DSM and time of day metering. That shouldn't be an afterthought, it should part of the basic design. Currently, there are large peaks due to summer A/C and winter heating, and there is a bit of DSM and time of day metering going on in Ontario (more than enough to show it works), but it's not widely implemented. Once it's widely implemented, overall demand will fall somewhat (perhaps 5-10%), the relative seasonal peaks in demand will fall even more (perhaps 15%, depending on how aggressive the rate structure is), and the "hard generation" safety margins won't need to be as large, as DSM will provide "negative power" that's every bit as reliable as hard generation.

Does that make sense?

Nick,

Agreed it's better to look at these things in order, but some can be dealt with pretty quickly.

On DSM; Yes, we can and should do a lot, time of day charging is one way of achieving, that and there are others too. Since everyone agrees on this, we can leave it there, and turn our attention to meeting the (reduced) demand after we have done all of this.

We then have a lower peak/average ratio for the overall demand, which favours baseload generation. This is good because baseload is more efficient (e.g CCGT instead of simple cycle GT). BUt add in the variable sources like wind and solar, and put them at the top of the dispatch order, and we actually need more peaking rather than baseload capacity, though we will use less "fuel" overall. It is the equivalent of doing a 10 mile city drive instead of a 20 mile highway one, we use more fuel per mile, but less fuel overall as we have cut back the driving more than enough to make up the difference.

Indeed our "hard generation" margins can shrink, to maybe 110% of peak demand instead of 135%. For this thought exercise, we can simplify it to say hard generation must equal 100% of peak, and hopefully our peak is only 70% of what it used to be.

So far, so good, now let's look closer at the supply side, and we have two issues;
1. Where the kWh's come from, on average (i.e. how much of what different"fuels" do we use. And we all agree, the less fossil fuel and the more renewable, the better.
2.Where do the kW come from, for meeting the peak demand periods.

This, to me, is the only real sticking issue, and I would characterise it like this; What is needed to be done to bring some portion of wind (and solar)
into the "hard generation" category? Presently, I rate them as 0% of nameplate in this category, because you cannot turn them on whenever you want, and the probability distribution shows there are inevitably times when they produce less than 1%.
And if we can;t bring it into the hard generation category, by some means, then we have no choice but to have hard gen equal to 100% of peak load.

For geogrpahical distribution, meaning bring it in from somewhere else, I regard this as "soft" generation in the same way as wind itself. Unless you can get a contractual guarantee from the neighbouring system, to supply you with X at any time you like, you have the prospect that they may say no.

A summer peak likely means hot weather everywhere, so there may not be any excess to import. But let's assume there may be. If we are looking to assign some portion of wind to the hard category, by importing (wind)power from somewhere else, then the wind needs to be blowing there. We'll assume the wind has the same distribution in the adjacent system, but is completely independent - i.e. low wind in A does not necessarily mean low wind in B.

Lets assume we set the "hard" cutoff as being what we can depend on 90% of the time. Now, for 10% of the time, our 1017MW is producing less than 45MW, or 4.5%, and ditto for the adjacent system, which we'll assume has the same 1017 capacity. Now, to import from them when we have less than 45, they must have more than 45, which, of course is 90% of the time.
So, on the 1 hour in 10 that we can't get our 45, there is a 90% chance that we can get it from next door. But that means, on average, that 1 out of a 100 we can't. The exact amount of the shortfall is a distribution in itself, but to keep it simple, it means that 1 hour out of 100, every four days, on average, our wind is producing nothing, and we can;t import, so what then? We have already done all the DSM possible, so as I see it our options are storage, or more hard generation, and storage, really, IS hard generation.
And, unfortunately, the wind is not totally independent in the adjacent system, it may be time shifted, but the seasonal maxima and minima still apply.

This has also ignored the fact that to do this, we need to have interconnections between the systems, which we currently don't, so there is a significant expense. Has other benefits, of course, but this expense, primarily to be able to import wind from elsewhere, must be acknowleged.

So, to boil it down, what must be done to bring some portion of wind into the "hard" category, so that it can displace other hard generation, for if it can't we are stuck with having to have (at least) hard gen equal to 100% of peak, whatever that peak may be.

Just as a follow up, have a read of this article, and note the comments from the Bonneville Power
Authority.

http://seattletimes.nwsource.com/html/localnews/2011931473_windpower23m....

I think, at some point in the near future, the power authorities will be saying to the wind industry that they can only maintain their first in in line status if they can maintain x% of dispatchable/baseload power.

On DSM; Yes, we can and should do a lot, time of day charging is one way of achieving, that and there are others too. Since everyone agrees on this, we can leave it there, and turn our attention to meeting the (reduced) demand after we have done all of this.

Hmmm, we can't move on quite so fast. We've reduced the peaks, due in large part to time-of-day pricing, and reduced the reserve margins needed, but DSM is far more powerful than that. Don't forget, if we're talking about a zero-FF world, then it's now a very different world. Most vehicles are electric, and most major Industrial/commercial load, and the majority of consumer/residential loads could have, and should have been evaluated for DSM appropriateness, and wired to be responsive.

More later...

Nick, the problem is that, today, we aren't in a post FF world, and we don;t have discretionary control over all these loads. Try getting a car factory to shut down at the drop of a hat because there is no wind and see what happens.

That is an extreme case, but now matter how much DSM we do, at any given time, there is a demand that has to be met, after we have turned off every load possible. Given that we have the ability to build GW and GW of wind today, it will get (is being) built much faster than we can do DSM, unfortunately.

The system operator will face this issue constantly, and for any system, each year they will have a design peak demand. We'd like to see that trending down each year, but it still has to be met, every time it occurs. This is the fundamental unsolved problem, and gets worse as wind power grows. I maintain my position, that you need to consider what must be done to re-package (at least a portion) of wind electricity so it can be moved into the hard generation category.

If the way we meet peak demand, by reducing and/or shifting it, then wind power cannot claim the credit for eliminating fossil fuels, DSM does. DSM has broken trail for wind and solar to thrive. We can do DSM without large scale wind, but we can;t do large scale wind without DSM.

Try getting a car factory to shut down at the drop of a hat because there is no wind and see what happens.

As it turns out, I was once working in an auto complex where 3/4 of the lights were turned off to reduce power demand during a supply crunch.  Competing companies were shut down because their contracts were all-or-nothing.

This was the practice because paying workers not to work for one day a year was cheaper than paying to build enough capacity (or having backup generation on-hand) to supply needs during extreme weather.  Simple tradeoff.  If supply was less regular, the tradeoff would favor more backup or storage.

you need to consider what must be done to re-package (at least a portion) of wind electricity so it can be moved into the hard generation category.

Compressed air appears to have a great deal of potential in that area.

If the way we meet peak demand, by reducing and/or shifting it, then wind power cannot claim the credit for eliminating fossil fuels, DSM does.

So if wind supplies 50% of your energy through various DSM and storage measures, wind doesn't deserve the credit?  Sorry, I don't buy it.

What is needed to be done to bring some portion of wind (and solar) into the "hard generation" category?

Storage can do that.  Some DSM measures have characteristics of storage; making ice for future A/C requirements, or charging EV batteries to the minimum required instead of full, are all ways of managing instantaneous power demand.  Any of them can be used to make good use of unschedulable generation resources.

We have already done all the DSM possible

You really think so?  ROTFLMAO!

yes, storage can do that. I see it as the equivalent of the "balance of system" stuff needed for solar (inverters, mainly). What I am getting at is that wind, to be more than a minority player, must package itself with storage. This changes the economics, in both good and bad ways - energy is more useful (dispatchable) but more capital intensive - and how many kWh, and kW storage per kW of wind turbine?

"We have already done all the DSM possible"
You missed the context - this is our hypothetical example, certainly not representative of today. But if we can do as much DSM as we want, we still can't get demand to zero, so we will have some level we have to meet. It might be 70, 50, even 30% of today's peak, but whatever it is, we still need to know how we can meet it, with a suitable reliability factor/capacity margin.

Look at the off gridder house, like Ghung, he has done all the DSM he can (by not building demand in the first place, and being able to shift what he has). He has solar, + wind + DC storage. How big is his inverter relative to his peak and average loads? How much storage?
He is willing to accept reduced system reliability, and the possibility of having to turn all loads off, in exchange for all renewable generation, and a reasonable storage and inverter capacity, but can we accept that for the grid, where we can't turn all loads off?

You missed the context - this is our hypothetical example, certainly not representative of today.

And not representative of anything we're going to see, either.  The 19% of capacity represented by nuclear isn't going away any time soon, and hydro doesn't look to disappear either (though rainfall affects it quite a bit).

You're coming at this from a vastly different, and IMHO fundamentally unrealistic, viewpoint.  If we had the all-RE generation system you're postulating, our consuming systems would also be radically different.  Evolving from where we are now is going to involve a ramp-up of RE generation, DSM and storage systems along with each other.  Anyone can erect a straw man to knock down, but making a realistic argument against real-world possibilities isn't so easy.  You're not doing well, and you'd do much better for your case if you'd try harder.

Let's remember who postulated what. Nick said "eliminating FF with renewables, especially wind, is easy"

And he also said

Another key point: you're not likely to have to backup the whole grid. If, for instance, we had a grid like this:

Average KWH market share:
40% wind
25% nuclear
20% solar
10% hydro, wave, other
5% biomass peakers

I am arguing that we can;t get to this point, of an all RE grid.
As for the 20% nuclear, not everywhere has it (e.g Hawaii, Alaska, western Canada, Australia, NZ), or wants it. But I am splitting hairs there. For this context, all I really acre about is generation that is dispatachable, and generation that is not (i.e. wind, solar), and what is required to make it dispatchable, or how much I have to de-rate it to get a reliable %

I don;t think it follows either that An all (or even high) RE system means that the consumption is vastly different. Almost all utilities have this mandate for 20% RE, and many people would like to see this go much higher. many utilities (including Ontario) have committed to phase out coal, replacing it with wind, but is this achievable? I think it is not, without something else, like your paired storage, or massive DSM. And if we had paired storage and massive DSM, I'd lay odds that we could meet the needs without needing a huge build out of wind, so it is still optional. Meanwhile, when people talk of wind displacing 20%, or X, of capacity, this is a very rubbery number, because, as we seem to agree, you can't replace the capacity, only supplement it. Any real capacity replacement is by storage or DSM. Sure it makes the system more amenable to wind, but equally more amenable to CCGT.

My real world viewpoint is this;
- Wind is being built rapidly, and usually subsidised to do so
-Already some electric systems are approaching stability issues from wind fluctuations
-large scale DSM takes time to implement, especially at the consumer level. I have been doing it for ten years, and most utilities for longer, but there is still lots more that can be done - wind will get built faster
-The wind industry promotes that it can replace FF generation, to a substantial degree, more than 20%
-we both agree that it (alone) cannot, it needs DSM and storage, and smart grids, (and V2G) to do it.
-the wind industry does not acknowledge this, and does not want to pay for the necessary things to enable the grid to function on wind (or RE>20%)
Who then, does, and pays for, the ramp up of storage and DSM? I am a supporter of a carbon tax (we have one here in BC, though far too small to make any difference) And how fast can it go? Without mandates, which are unpopular, it will take decades.

I don;t think any of that is unrealistic. What I think IS unrealistic is the high RE system that Nick postulated in the first place - I am glad you agree.
-

Nick, was away from the computer all day but here is your answer;

I got my numbers from the graph on Web HUbble Telescopes website, which is based on four years of hourly data (35000 data points) of Ontario's wind power production;
http://2.bp.blogspot.com/_csV48ElUsZQ/S-Sn9mKBNII/AAAAAAAAASc/N61wa5BKm3...

You can find the original spreadsheet, in csv form, here, under Wind Generatio Output;
http://www.ieso.ca/imoweb/marketdata/windPower.asp

And a report from them, about their system, and have a look at graph 7.2.1

http://www.ieso.ca/imoweb/pubs/marketReports/18MonthOutlook_2009nov.pdf

Now, given that the data is four years, I will concede that it may underestimate the picture, as it does not fairly represent wind capacitya dded in thos four years. so I pulled down the spreadsheet myself and took just the last 12 month's data (14May to 14May) and re-plotted it, in the same style, as the probability of exceeding any given power output, based on 8760 one hour intervals.
Here then are the up to date numbers, bigger, but the same story;

99% of time, 9MW, 0.9% of 1017MW
95% - 26MW, 2.5%
90% - 40MW, 4%
80% - 77MW, 7.7%
73.9% - 101.7MW - 10.0%
70% - 118MW, 11.8%
60% - 161MW, 16%
50% - 213MW, 21%

I got this by sorting the hourly intervals in ascending order by power, so 0 to 1017, and then numbering them from 0 to 8760. % exceedance is then 1-N/8760, so 100% of time (less one data point) it exceeds zero, and 0% exceeds 1017.
I don;t know how to post a graph here, but the shape is identical to what WHT came up with.

So, literally, 4380 of the 8760 hours had a power output of less than 213MW, this it the median power.
The average was 279MW, which we can take to be the capacity factor for all wind (assuming 100% availability).

But my key point remains that for a good chunk of time, the power output is negligible. 1% of the time is 87 hourly intervals where the production was less than 9MW, and the 10% mark means 876 hours where it is less than 101.7MW. So clearly there are many times that wind contributes very little, and other generation must be used. I would go so far as to say, that the wind power does not allow for more than 10MW of other controllable generation to be decomissioned.

Now, this is with wind as 1017 out of 35,000MW of generation, and a 25,000MW peak demand.
If we scale it up by 10x, to 10170MW, which would then 22% of the grid capacity our 99% exceedance is 90MW, and our 10% capacity is still not reached for 26% of the time. Of course, we would probably measure its % of grid using capacity factor, which would then be 2790MW, or 7.25 (of 35,000 +2790)

But at what point can Ontario decommission its aging 4000MW Nanticoke coal plant, which is currently 11% of the grid? And without reducing the grid reliability factor, keeping in mind Ontario's desire to NOT increase dependance on imports from Quebec, who are under no obligation to supply power when needed - they will only supply when available, and they can usually get higher prices in the US.

We can;t rely on getting 2790 for for 60% of the time, if we want 90% reliability for wind, we have to de-rate the 10170 to just 400MW, but even then, it leaves 876 hours a year that we have to come up with power from somewhere else.

So what percentage exceedance level do we assign to the wind power, as being equivalent to controllable peak capacity? How much do we need to add to start decomissioning other generation plants? When we say "20% of the grid capacity" which number should we use? Certainly not nameplate, as we rarely get it. Average capacity we only get for 40% of the time, Even the 10% of capacity will leave us short for one hour out of every four.

if we can reduce peak demand, permanently, we can start to decommission other plants accordingly, but assuming peak load stays the same, at what point can we truly replace the coal plant with wind, and maintain grid reliability?

The way

Paul,

Thank you for the data - that's very helpful.

I'm champing at the bit to discuss all of this, including statistical and geographical considerations, but I think we need to try to discuss one thing at a time to make progress. Please see my reply to your other recent comment here: http://www.theoildrum.com/node/6480/629534

But at what point can Ontario decommission its aging 4000MW Nanticoke coal plant, which is currently 11% of the grid?

As a hypothetical, how about when wind either supplies 11% of the grid directly and NG-fired turbines running on the saved fuel can satisfy the base load, or when wind supplies some greater amount (say, 16%) to offset the direct production of the plant plus around 30% losses in storage?

When the average is 297 MW out of 80+ times that much peak design load, you're so far from any of those milestones there's no point in worrying about it for years.

As a hypothetical, how about when wind either supplies 11% of the grid directly and NG-fired turbines running on the saved fuel can satisfy the base load
Well, the fuel we saved was coal, not NG, so we are just switching to another FF (although it can be produced renewably, though not yet at this scale) but what, exactly, does the 11% mean? 11% based on nameplate capacity, which is rarely achieved, or on average capacity, which is only achieved or exceeded 40% of the time, or the production level that can be relied upon 90% of the time, which is 45MW out of 1017MW nameplate? 11% of the annual kWh is one measure, but it's not much help if you are often running into trouble meeting instantaneous demand. What portion do we deem reliable enough to constitute the 11%? And this scenario implies we need to build at some new CCGT to replace
at least part of the 4000MW of coal, but how much?

or when wind supplies some greater amount (say, 16%) to offset the direct production of the plant plus around 30% losses in storage?
Now you are talking. So now we have a "wind+storage system", what is the nameplate capacity, and cost, of the "system"and what is the availability of the nameplate capacity, and for how long? (e.g. an 8 hour period windless period) . How many MW of turbine nameplate and storage do you need to give 100MW for 8hrs, or even 1hr, guaranteed? And, where can the storage be built - i it dependent upon underground caverns like the CAES proposals I have seen?

When the average is 297 MW out of 80+ times that much peak design load, you're so far from any of those milestones there's no point in worrying about it for years.

This is true, but it skirts the issue. At some point, wind will be enough to cause stability problems, as has been talked about here (http://seattletimes.nwsource.com/html/localnews/2011931473_windpower23m....)
Ontario isn't there yet, but it could be, and other places like Bonneville already are. Or do we draw the line at 20% (and again, 20% based on what?) In other words, we can have wind, but as long as it is only a minority part. Solar will have the same issue, so how then do we replace the remaining FF to get to a renewable grid, other than nuclear?

Well, the fuel we saved was coal, not NG

You miss the point.  If the wind supplies enough energy that the saved NG can handle the 4 GW base load when the wind isn't there, the coal won't be missed.  Pretty much anything that saves NG will do, including DHW and space heat.  If you use winter wind to supply space heat in lieu of NG, that NG can replace coal in the summer even when the wind is dead calm.

How many MW of turbine nameplate and storage do you need to give 100MW for 8hrs, or even 1hr, guaranteed?

Meaningless question.  Even a 1 megawatt turbine can put enough energy in a store to supply 800 MWh of power, it's just a question of how long it takes.  If you were taking 200 MW from your Ontario wind farms and storing the excess, the 97 MW average stored at 70% efficiency would build up 800 MWh in less than 12 hours (on average).

And, where can the storage be built - i it dependent upon underground caverns like the CAES proposals I have seen?

Cavities can be mined in rock or dissolved out of soluble minerals like halite or potash.  Air can be stored in porous formations such as sandstones, which require no mining at all.

AFAIK the entire state of Michigan overlies a thick deposit of halite; solution-mining halite is simple bordering on trivial.  I'm sure some entrepreneur in Michigan would be happy to store Ontario's wind power, for a fee.

how then do we replace the remaining FF to get to a renewable grid, other than nuclear?

I don't have a problem with nuclear (I rather like it), so I don't see the conflict.  It's just a question of what's quicker to install and matches the demand curve better.

{duplicate}

We can't have it all:

I'm guessing the ideal speed is some where between 50 to 60 mph. There should be a date in the simi(5 to 8 years) near future that all new vehicles are governed and optimized for this ideal speed. If you don't like the idea, stimulate the economy and stock up on todays vehicles.

Why do you believe the "ideal" speed limit is between 50 and 60mph? The national speed limit during WWII in the US was 35mph.

If the speed limit was set at 35mph, passenger vehicles could be much, much lighter, because protecting occupants in a 35mph crash is much easier than protecting them in a 60mph crash. Of course, the aerodynamic advantage of 35mph over 60mph goes without saying.

There are some interesting electric vehicle designs out there. The Myers DUO(http://www.myersmotors.com/) has three wheels to avoid passenger car safety regs, but it is essentially a small, light car. There are several good motorcycle designs, also. The Brammo (http://brammo.com) and the Zero(http://zeromotorcycles.com) have taken the lead in the US market.

At least reduce to 55 mph ... then work downward

http://www.drive55.org/

http://drive55.org/images/stories/tdiclubchart300.gif

It's all about time vs. energy saved. The idea needs to be sold to the public and be realistic. Just look at todays resistence to 55 MPH. Which I believe should have been reinstated yesterday.

Aerodynamic advantage really don't start to show up until about 50+ MPH. Aerodynamic resistance is square to the speed. In addition todays cars optimum economy is also at about 50+ MPH. 35 MPH just doesn't seem realistic vs. the additional time to travel, maybe in 50 to 100 years from now it will be.

DownToTheLastCookie said,

"Aerodynamic advantage really don't start to show up until about 50+ MPH."

That is only true because of the massive amounts of spare horsepower most cars have.

The rule " Aerodynamic resistance is square to the speed" applies with equal force if you double the speed from 5 to 10 mph as well as if you double it from 50 to 100mph.

This is why streamlined enclosed bicycles are so much faster than open bikes with an upright rider, and why the tiny cars used in "eco-racing" using electric power or very tiny gasoline engines are so attentive to aerodynamics...when the horsepower is limited, aerodynamics and rolling resistance (tire to surface) become all the more important...some of the most aerodynamic racing cars have been in the smallest bore racing classes, where every bit of efficiency was needed, even though the top speed of the vehicle was far below the big engined cars...so much so that high horsepower racers began giving away the top speed/efficiency advantage in exchange for wings to create downforce while cornering...unthinkable in a very small bore car because the aerodynamic drag cannot be tolerated.

RC

Of course, the aerodynamics of open bikes with an upright rider make a brick look good, and bikes have very low rolling resistance and drive-train losses.

When you work with a normal vehicle, aerodynamics don't become important until well above 40MPH.

duplicate

A speed limit of 35 mph (or less) would also have the very interesting advantage of being more compatible with human powered vehicles. Bicycles and cars could share the same road space.

Another advantage is that no one will want to take a car across states at 35mph, which makes bus and rail traffic more palatable.

I think the slide down the speed/weight/driving cliff will be gradual.

Reduced speed limits are in place in of parts of London - either in residential areas or some of the parks. 30mph has dropped to 20mph. And yes, at this speed good cyclists can easily stay with, or dictate the traffic flow. Slower cyclists still need, or would benefit from, a protected cycle lane. Urban cycling has doubled in parts of London in the last 10 years and I see no reason why it shouldn't continue to grow.

Obviously to many a limit of 20mph means drive at 30mph, but it is a slow learning curve to moderate drivers' behaviour. It is well known in London that 'average' traffic speeds during the day times are similar to those of 100 years ago - in the 10-15 mph range.

One other advantage of the EVs - they are quiet! Our milk & juice etc is delivered to our door in the early morning hours (5-7am) by an electric 'milk float'. EVs are much quieter in the morning - as anyone who has had some diesel truck or taxi sitting outside their house, idling, at 5am will appreciate.

If I had a magic wand, I'd have General Motors get the unemployed in Michigan back to work by creating an EV division. This would be a much larger effort than the Volt, in fact I'd likely scrap the Volt as it is based on the "old" way of thinking which is to give the consumer an electric version of the gasoline car they are used to.

We don't need a one size fits all EV, we need a minimum of a dozen different models, each with a unique focus.

For example, there will always be a need for a long range, single passenger, commuting EV. There are plenty of people who have commutes of 50 miles or more one way. They are going to need an EV that can go 65 mph. Then there is the need for a six or seven passenger EV that travels shorter distances at low average speeds. You will need a two passenger EV that carries a heavy cargo load for use by businesses. The list goes on and on.

What will have to change is our assumptions on what is important. Must we have air conditioning as standard on EVs or can those sold in the cooler climates be designed so it's not needed by using appropriate color choices, venting, and window treatments. Although EVs have the capacity for great acceleration, we can limit it to improve efficiency.

The consumer can have limits forced on them, but they can't live with a design that is not suitable for the basic use of the EV. Providing a range of seating options, top speeds, driving ranges, and cargo capacities will ensure faster adoption of Evs.

A side note. Perhaps Evs will inadvertently help with the obesity epidemic in America. Additional weight equals shorter range.

One question conspicuous by its absence is, "Why have automobiles in the first place?"

The idea that we can continue to occupy the land as we have during the hydrocarbon era most likely won't be viable in the coming years of energy famine. People will have to get reaquainted with ideas like living near where your food comes from and spending your most of your life within 50 miles of your birth. Or consider how the levels of consumption will decline as the energy and hydrocarbon feedstocks are depleted into economic insignificance.

Clinging to the automobile is just another form of denial. Better we should re-engineer how we get our food, fuel, and fiber so that personal transport on demand is no longer a necessity to participate in the economy. The end of the automobile is inevitable. Might as well jump rather than be pushed.

Clinging to the automobile is just another form of denial. Better we should re-engineer how we get our food, fuel, and fiber so that personal transport on demand is no longer a necessity to participate in the economy. The end of the automobile is inevitable. Might as well jump rather than be pushed.

This seems possible for an individual to do, but not for society at large. I can't find one example in modern history of people doing so. Jared Diamond does mention somewhere an island tribe that all at once banished pigs, but we can only guess that was for ecological reasons.

Electric cars and cycles simply are not really right for all places. I live in a mountainous and sparsely populated area. During summer months I could use an electric vehicle to travel the approximately 20 mile round trip to buy what food I don't produce here on the homestead, but attempting that trip during the heavy snow months would be risky. I don't know how much energy would be wasted by spinning on ice, but I know quite a bit of gasoline is wasted that way.
I also think that there's a fair bit of preparation to be completed before encouraging folks to jump into an EV. I doubt that triple A or any local repair operations would be able to re-charge a battery like the ones you describe should it become discharged.
I do think that EV's can become a significant means of travel, but they are better matched for specific situations. Philadelphia strikes me as a pretty good situation- its pretty flat, good population density, has enough resources in place to build out an EV support system and certainly would benefit in the reduction of noise and air pollution.

No vehicle will ever be right for everyone. I have a four seat car that I use almost exclusively for commuting. Most years I have a passenger less than 5 days out of the 260 days I use it. I could cut my gas use in about half or more if it made economic sense to get an electric vehicle with enough range to get to and from work. This vehicle could be a light 1-2 seater with limited bad weather driving abilities. I also have a van that I use for my family of 6. Getting an EV with enough range, seating, and cargo capacity would be difficult. If oil gets expensive enough, necessities will become nice to haves very fast. If gas gets really expensive many people may decide that maybe low powered scooters aren't so bad after all. A 50cc scooter manufactured today can get 100-150 mpg. I can be manufactured quickly and cheaply. There are no limiting resources (rare metals used in batteries). People who are more well off will be able to purchase new EV's, people who aren't will need to find alternatives or do without. There are some viable alternatives. Some of them aren't as nice or flashy as what we have now, but there are alternatives.

The most practical electric vehicle is an electric bicycle. There is no need to hold out for a new battery technology that might some day replicate the energy density of gasoline. Electric bicycles which will competitively carry a person through the urban landscape are available today! The primary obstacle to general use of electric vehicles in this country is cultural momentum acquired with historically cheap oil. Many of us have come to expect to travel in a steel-reinforced shell which might protect us from a collision with other moving shelters. A detailed cost-benefit comparison based on dollars and GHG emissions can be found below:
http://knol.google.com/k/energy-global-warming-and-electric-bicycles

Electric "bicycle" (2 or 3 wheels) for the future now in testing ...

http://endless-sphere.com/forums/viewtopic.php?f=28&t=17848

http://www.wix.com/trivelo/the-trivelo-project

Not enough people can afford the Volt or Leaf to make it economically viable or sustainable.

I completely agree. In fact, you could make a 2 seat tandem trike from heavy duty bike parts and off the shelf electric bike motor/battery that would pretty much Kick Ass - viz the Stokemonkey. It could EASILY be built for less than $5k.

Powerwheels - Jammin Jeep Barbie.
Durable, well built 2005 models still working. whole host of parts can be added on lasts 1-2 hrs at 5 mph on 12V battery.

http://www.hobbymasters.com/powerwheelspartsdiagrams.aspx
So many models

yet so simple
http://www.hobbymasters.com/ProductImages/powerwheels/breakdowns/H3427.pdf

The best electric vehicle is the passenger train.

Needs no batteries.
Extremely efficient.
Drives itself -- you can even drink booze while riding!
Fixes and maintains itself.
Doesn't need insurance.
No parking needs.
No traffic tickets.
Very safe.
Runs fine in all kinds of weather, including rain, snow, hail, hurricanes, etc.
Can be used by children.
Carries any number of people. Take all fifty of your friends along.
Needs much less infrastructure (one track vs. many many roads and parking lots).
Uses regenerative braking.
Can be built and maintained with hand tools (extensive paved roads cannot be maintained with hand tools).

and best of all ...

Much cheaper!

We might need another thousand of these "electric car of the future" posts to figure this out. But, I'm sure we'll get it eventually.

In terms of personal electric vehicles, I've been saying that the natural solution is little battery-powered scooters for distances of less than five miles. Like George Gilder used to say, listen to the technology. If you look at everything described above, you should come to the conclusion that personal electric vehicles make the most sense when they are very small, very light, run at low speeds, and for short distances. Which also makes them very cheap, under $1000.

Can we please get past the "we're gonna run Suburbia on batteries" stage?

What a real train system looks like:
http://www.newworldeconomics.com/archives/2009/122809.html

Trains need drivers, conductors, and maintenance facilities, too.
And mechanics to keep them running.
And people to clean up the mess that some of the passengers leave.
And they have to have insurance. All it takes is one accident....

(I commute by train. I also live within walking distance of the station. Most of the other passengers drive to the station.)

"Trains need drivers, conductors, and maintenance facilities, too.
And mechanics to keep them running."

And all it takes is one major railroad workers strike to paralyze a city or a region to end the idea of rail reliability...the problem is not with the technology.

People seem to forget that we left trains for cars once before...when I was a child, I could walk 1 mile in a tiny town in centrel KY and then be able to ride on a train to any major city in the country...or I could take a bus to any major city in the country.

Now, the only way out of that small town to even get a decent job is either to drive or to walk out (a full days walk to the nearest town of any size, some 22 miles away) To be able to leave the hometown for a job, I worked as a busboy and dishwasher in a kitchen and saved every dollar to get my ride to freedom (a Chevrolet Chevette stick shift for about $400 dollars). That is the world without cars as it exists today...it would consign millions of the poorest people in the nation to servitude to one local employer in serf conditions. We so take it all for granted, don't we...:_-(

RC

Since my employer procured a GEM NEV a little over a year ago, I have been using it routinely for business trips during the work day to the bank, post office, and other nearby destinations around town. Based upon this personal experience using an electric car:

1) For such short, local trips, the NEV is quite adequate. It gets me where I need to go and back again just fine. The maximum speed is 25 mph, which is just a little slow on a street with a speed limit of 35 mph, but our small town streets are not all that busy, especially mid-day, so that hasn't been a problem. If traffic backs up behind me I just find a place to pull over and let them pass. It takes me a couple of minutes longer to get where I am going in the NEV than it would in a conventional ICE car. So what?

2) Remembering to plug the NEV in to recharge is no problem, one immediately gets into the habit. Establishing a parking place with electrical access in the first place was actually more of challenge. Many people will find that their existing parking lots will not work unless expensive wiring is extended there. We found a place around the back of our building that worked better for parking the NEV and plugging it in. That is slightly inconvenient, but it did perhaps serve to reinforce the habit of plugging the vehicle in when done; if it were just parked in the lot, it might have been easier to forget.

3) One potential problem one has to watch for: a thrown breaker or other disruption of your recharging power supply. This happened once, and I didn't notice that the battery was not fully charged. Fortunately, there was enough charge to get to where I was going and back again, but it taught me a lesson: remembering to plug it in after each trip isn't enough, you also have to check the charge status each time you turn it on. It is probably a good idea to have some procedure to check this often if you are not driving the NEV on a daily basis.

4) Uphill is a bit of a challenge for this NEV, there are a few places around our area where it probably couldn't make it. Fortunately, the places where I need to go don't involve very steep routes.

5) No a/c (only fans), and the "heater" only barely works (and is a big drain on the battery). That hasn't really been much of a problem, but we have tended to not use it so much during the worst of the winter weather. I don't know how well it would handle icy or snow-packed streets, either - I didn't attempt that. The windshield wipers are adequate for warm-weather rain, but I wouldn't want to test them out during a winter storm; the lack of heat on the windshield could become really dangerous in hazardous winter weather. On the other hand, taking the NEV during rainy days really is considerably more pleasant than would be riding a bike or going on foot.

6) This NEV is fine for carrying one or two people or a driver and a light load (the daily mail, for example). The upper load capacity limits are much lower than what one would expect for even a small conventional ICE vehicle, however.

7) One of the big advantages of any type of EV, and something not mentioned in the above article, is low maintenance. Compared to what is required to maintain a conventional ICE automobile, an EV or any type just doesn't need all that much in the way of routine maintenance. Check the tire pressure, a little lubrication occasionally, change out the wiper blades when worn out, and the usual washing and cleaning, and that's about it. The batteries may have to be changed out after a few years, but if you avoid accidents or other damage, that should be about the only big ticket item. Low maintenance was one of the reasons why the early EVs were so popular - you didn't have to be a mechanic to own and operate one.

8) Using this NEV has impressed upon me how much overkill it is to drive even a small, fuel-efficient conventional ICE automobile around for short-distance local trips around town. It really is ridiculous when one thinks about it. On the other hand, the NEV is worthless for trips out of town, we need conventional ICE automobiles for that. If it were only possible to own one vehicle for all trips - local and long-distance - then an NEV (or probably just about any EV) won't do for most people. The ideal situation, to my way of thinking, would be to have an NEV for local trips, plus either own an ICE auto outright, or else have easy access to a car rental outlet if one's long-distance travel needs are not too frequent. (For the interim, that is, unless and until a more dense and adequate mass transportation system is built.)

This is a stupid, though useful, article. The author doesn't seem to grasp that the real upshot of his review is that cars-first transportation is criminally stupid and ultimately impossible. And he doesn't even mention the manufacturing side of the equation, which only multiplies the point.

What makes you think I don't grasp the real upshot of my review? Follow the first link in the article, I would say:

http://www.lowtechmagazine.com/2009/10/get-rid-of-cars-ride-a-bicycle.html

How could you possibly know what the author doesn't grasp? Just because he focuses on the limitations of EV doesn't mean he doesn't recognize that the better approach would be to substantially eliminate autos altogether. Besides, your comment was just mean and uncalled for.

I think we need to get beyond autos but I know that has zero chance of happening.

Exactly. Eliminating cars all-together would save so many problems that it might as well save the world. But 999 in 1000 people don't agree. So we have to talk about how to reduce the harm. The move to heavy and fast EV's will make things only worse, because we are going to waste an awful lot of energy and resources on something that will never work.

Just eliminating cars doesn't fix the problem.

Suburbia designed for cars + no cars = failure

Suburbia designed for cars + bicycles = failure

Suburbia designed for cars + electric doodads = failure

Suburbia designed for cars + trains = failure

Suburbia designed for cars + buses, horses, whatever = failure

Pedestrian cities (pre-1800 style) + trains+ bikes = success

Unfortunately, we still have a lot of bike fetishists around here. Just as we have to kick the mental car habit, I think we have to kick the mental personal transport device habit. Bikes aren't necessary in a properly designed pedestrian city.

Life Without Cars 2009 Edition
http://www.newworldeconomics.com/archives/2009/121309.html

But that's exactly what I mean: eliminate the car and everything else has to change. For instance: suburbia would disappear.

Kris, enjoyed the article. But I have to chuckle whenever someone talks about eliminating suburbia and living within walking or biking distance to work.

I know real estate values have tanked in some parts of the country, but in most places people do have equity in their homes or own them outright. There is simply no economic way to leave all those houses empty. Do you really think a bank will give me another mortgage on a property in the city if I left them with a house they can't sell? Will I give up a house that is paid for just to save a 45 minute commute each way?

These new homes in the cities don't currently exist, they will have to be constructed at today's prices. That is a barrier few people can overcome.

Like all problems today, I think we are looking at a laundry list of solutions. Ranging from public transportation, to car pooling, to EVs, to scooters, to, yes in some cases, moving into the city.

"Eliminating surburbia" should not mean "leave your house empty". It could also mean providing suburbia with shops, facilities and employment. In other words, move parts of the city to suburbia. Make suburbia more self-sufficient.

Good point.

Kris De Decker said,

"Eliminating surburbia" should not mean "leave your house empty". It could also mean providing suburbia with shops, facilities and employment. In other words, move parts of the city to suburbia. Make suburbia more self-sufficient."

That is the way it will have to be done. X points to the equity people still have in their homes in money, but there is another kind of equity tied up in them: Energy equity. If we accept the terms of a resource and carbon constrained future, we simply cannot garbage out the energy invested and the carbon locked up in these homes.

This is one area TOD has caused me to become more and more aware of: A major sports stadium, for example, is a huge energy sink. We have tons of coal or gas tied up in it, as well as the carbon release used to create it. Any structure is embodied energy. (Think of the pyramids or a great cathedral, the millions of calories consumed to deliver millions of manhours of hard labor tied up in it) Just junking it out means that investment is lost.

The same is true of a home. This is why I have, for the moment, tried to find a suitable already built home rather than expend more energy trying to build a new one...no matter how efficient the new home is, the bulk of the energy it would ever use and carbon it would ever release would be expended in the building of it.

What is tragic beyond words is that so many almost new suburban homes were built with no concern for energy use, thus locking us into a level of waste and excess consumption for at least the next 50 plus years. This waste is built right in. Do we now attempt to waste even more in re-relocating the population of the suburbs back into the city? That would be catastrophic, but the good news is that it simply cannot be done...the money, energy and resources simply cannot be afforded to make it happen. We are married to suburbia in some form whether we like it or not (sorry JHK) for the rest of our lives at least.

RC

Living within walking or biking distance of work assumes that you'll keep the same job with the same employer for some foreseeable length of time. That paradigm is gone, blown away with the same financial winds that turned mortgages upside down.

Unless you're a subsistence farmer or live in a big metro area with good public transit, there's no way to plan a place to live and a place to work that won't require a car at some point.

Living within walking or biking distance of work assumes that you'll keep the same job with the same employer for some foreseeable length of time.

Not necessarily. I spend my entire career working in the same small area. 80% of the Canadian oil industry has its head office within a 1 square kilometre area of downtown Calgary, and the service companies try to crowd in as close as they can get to them.

It was very convenient. Every time I got laid off by one company, I just walked half a block down the street and found a new job with their more successful competitors.

The U.S. oil industry is vastly different and is scattered all over the country, but that's their problem. I worked for a number of giant U.S. multinationals, but their Canadian office was always half a block from my previous job.

You could just BUILD a big metro area (or a small metro area) with good public transportation, according to Traditional City design guidelines.

Then you would have solved the problem, and you don't have to whine all the time.

Michael Dawson:

Speak for yourself! I'll never give up my car - ICE, electric, whatever it may be.
Commuting 60 miles in Hummers vs. riding your bike to the train station is a false choice. If we drive smaller, more efficient vehicles shorter distances, we really can prolong the age of oil.

And if we face a Mad Max future because of it - well, bring it on.

Some are indeed trapped in their lifestyle choices, by their own inertia and shortsightedness.

There is too much Luddite nonsense in the Peak Oil movement. If Peak Oil results in the end of point to point transportation you won't be able to walk anywhere as the bodies would be piling up in the streets. It would result in a complete collapse.

There is too much Luddite nonsense in the Peak Oil movement.

Their hearts are in the right place. Heck, what sensible person likes faux-military SUVs driving aggressively on our streets? And who can disagree that we have too much consumerism?

Sadly, it's wishful thinking to suggest that PO will get rid of SUVs, or other objects of fashion. They're a much hardier pest than that.

I think you can't really look at the question of "having it all" without looking at the Extended Range EV, as exemplified by the GM Volt. The Volt saves money and weight by reducing the size of the battery, and adds a cheap gas-fueled engine as a range extender. With mass production, it won't cost any more than the average ICE vehicle.

It's 40 mile range covers 80% of the average person's driving, and could easily cover 90%, if you changed driving patterns just a little.

It's likely to get about 50MPG in ICE mode (twice the average US MPG), so overall it will use only 10% as much fuel as the average US ICE vehicle. That reduces fuel consumption by 90%, which is good enough - we could supply the remaining 10% from ethanol.

The perfect is the enemy of the good.

The series hybrid is not a new idea. See the Lohner Porcshe developed in 1899. It even had wheel hub motors.

http://en.wikipedia.org/wiki/Lohner_Porsche

It's true. The series hybrid (aka EREV) has been reinvented many times. Any good engineer looking at an EV will ask the same question: "Why not add a generator as a backup for the unusual times when you run out of fuel?"

So why didn't the EREV concept take off? Because of dirt cheap gasoline - it just wasn't necessary. Now, of course, in a world of somewhat expensive fuel, it's competitive.

Looking at this chart

I don't see the competitiveness of series hybrids being any different today than they were in 1918.

Something is missing.

First, you're right that there are other factors: they include that series hybrids weren't perfectly suited for lead-acid: 1)LA has a low power per KWH, so that larger batteries work a bit better, and 2) LA has a lower cost and shorter lifetime than li-ion, so the small battery of an EREV isn't as useful.

2nd, battery lifecycle costs have declined since then, so that the competitive price point for EVs has dropped to around $2.50-$3/gallon.

3rd, much of the "price" of fuel I'm discussing here is in the form of 1)external costs, like security, balance of trade, pollution, etc, and 2) expected price increases.

1. We are confusing a plug in series hybrid and a plain series hybrid. A series hybrid would only need a tiny battery to buffer the engine or regenerative brakes. After all what size batter does a series hybrid diesel locomotive have?

But I find it hard to believe battery chemistry has been the huge hurdle for plug in series hybrids and only with the Volt has this hurdle suddenly overcome. After all battery chemistries with energy densities greater than lead acid have been around as long as automobiles.

2. Ev's are still not competitive and Gas is 2.85 in the US. Significantly higher than that in other parts of the world.

3. External costs are just that external. They don't figure into the costs a consumer pays. So until they are internalized they are rather moot.

We are confusing a plug in series hybrid and a plain series hybrid.

True. I'm talking about plug-in series hybrid, aka EREV. I'd estimate that a reasonably large battery (between 10 mile and 40 mile range) is the optimal point to minimize costs. I'd estimate that a 40 mile range is about the right point to optimize convenience and usefulness.

But I find it hard to believe battery chemistry has been the huge hurdle for plug in series hybrids and only with the Volt has this hurdle suddenly overcome.

When it comes right down to it, ICE vehicles were considered "good enough". The EV advantages just weren't important enough to give it enough oomph to be a major player. A range extender would have solved the range problem, but at extra cost, and no one saw any need for EVs, even without range problems. Now, we do, right? If we do, then how do we describe this new need? I'd use the language of external costs, discussed below.

Ev's are still not competitive

Sure, they are. The Leaf is cheaper than the average new vehicle (the Volt may be too). When you figure in fuel and maintenance savings, it's substantially cheaper. Now, you may object that the current price depends on a tax credit, and I would reply that the credit is a temporary adjustment for lack of production volume and resulting economies of scale. I could also argue that it's an appropriate internalization of external costs -see below.

External costs are just that external. They don't figure into the costs a consumer pays.

You and I figure them in. The government figures them in with tax credits, subsidies, etc.

OTOH, I'm confused. Do you agree that external costs are real costs, or not? If you do agree, then you have to agree that EVs are cost justified. Now, there may be practical problems in getting society to recognize and externalize these costs, with carbon taxes, EV tax credits, etc, but that's a strategy question, not a cost question.

When it comes right down to it, ICE vehicles were considered "good enough". The EV advantages just weren't important enough to give it enough oomph to be a major player. A range extender would have solved the range problem, but at extra cost, and no one saw any need for EVs, even without range problems. Now, we do, right?

No, not really. I don't see the need for EVs or EREV of any sort. I would argue most people never will. See the gas price graph above.

Until you internalize these costs they don't exist for the consumer at the dealership.

But lets say you manage to do just this via high gas taxes. You'd have gas prices something along the line of Europe or Japan. Don't see any EVs there. Or maybe you try to go the california CARB route. Didn't work for the EV1 or RAV4EV. Subsidies aren't sustainable politically or fiscally.

I don't understand why now all of a sudden EVs and EREVs are suddenly competitive in your mind. They have been tried time and time again for as long as the automobile has been around. They have always failed in the market place.

What has changed?

No, not really. I don't see the need for EVs or EREV of any sort. I would argue most people never will. See the gas price graph above.

I'm puzzled. By the same logic, there's no need for rail, or bicycles, or any kind of adaptation to PO or AGW.

Until you internalize these costs they don't exist for the consumer at the dealership.

Or, you can even the playing field by giving EVs a tax credit, which has been done.

You'd have gas prices something along the line of Europe or Japan. Don't see any EVs there.

There are a number of factors to help explain this:

1) A different capital cost to operating cost picture.

EVs and PHEVs trade a higher purchase price for lower fuel consumption.

In Europe, fuel prices are 2-3 times as high as in the US, but due to historical factors (shorter distances, higher fuel taxes due to the high % of imports), average car in Europe uses about 1/3 as much fuel as one in the US. Further, European taxes on new cars are generally much higher in the US.

Thus, the economic case for EVs and PHEVs is actually worse in Europe, and the lack of EVs and PHEVs in Europe really doesn't add any useful information to the question of how competitive electric powertrains really are with oil in the US.

2) Pure EV's still can't compete on convenience with ICE vehicles. Even in Europe, fuel costs are only a part of driving costs, and the lower cost of an EV hasn't been quite worth the inconvenience. The logical transition from an ICE to an EV is the PHEV, which for some reason wasn't explored seriously until very recently when GM took that path. Now that GM is pursuing PHEV extremely seriously, they're planning an Opel version for Europe.

3) Europeans have fewer garages, as their housing is much older.

4) Tax preferenced diesel occupies the high-MPG niche.

and perhaps most importantly,

5) there were large barriers to entry (billions in R&D and retooling, as well as resistance from ICE oriented manufacturers) for PHEV's, and there wasn't an obvious need for them. There was resistance from people in the industry who's careers would be hurt. This ranges from assembly line workers and roughnecks to automotive and chemical engineers. And, you've got to give them respect and compassion: they're people, and deserve to be helped as much as possible during a necessary transition away from oil.

Until we find a way to help these people, they're going to desperately fight any proposals to transition away from their industries, by honest attacks or dishonest: whatever works. You can't really blame them: they're just trying to protect their lives and families.

Biofuels, fuel cells, nuclear power, carbon sequestration all involve more chemical/process engineering R&D, and building of plants and retirement of old technologies. Isn't reduction of greenhouse gases is gonna be a golden age for the Chemical Engineering?

I suspect that this kind of thing is much more attractive to students and professors than it is to engineeers with 10-20 years of experience, who've attained high salaries in large companies due to their narrow expertise in a particular area, in that company. For them, I suspect any change which threatens their company threatens them personally.

On the other hand, the momentum has now shifted: most of those R&D $ have now been spent; the technology is better tested; the cost comparisons have shifted; and there's enormous pressure for PHEV's from regulators.

--------------------------------

I don't understand why now all of a sudden EVs and EREVs are suddenly competitive in your mind. They have been tried time and time again for as long as the automobile has been around. They have always failed in the market place.

Again, the same can be said for rail for any use outside of commuting, or transporting those who can't afford cars; or for bikes, and most other things discussed on TOD to deal with PO and AGW. Why so negative?

Wow, you wrote an incredible number of words yet still cannot answer a very simple question. What has changed to make EREVs competitive in the market today?

you wrote an incredible number of words

You asked about Europe, I answered.

What has changed to make EREVs competitive in the market today?

Well, I've already answered you.

The answer is that they were competitive in 1910 - tens of thousands were sold, and they outsold ICE vehicles for some years. Then widely available, reasonably cheap gasoline arrived and ICEs were not cheaper, but they were more convenient. EVs were no longer needed.

They are more competitive now, because the external costs of oil have gone up. This the reality cost-wise, and now with tax credits at least some of those external costs have been internalized, so it's also the reality price-wise.

Also, batteries have improved - li-ion is a much better chemistry than lead-acid, and it's becoming much cheaper (as measured per kWh-discharge cycle). Power electronics have also become much better and cheaper. Heck, such electronics and software didn't exist in 1910.

So. EREVs are price competitive now. So are EVs - the Leaf is doing extremely well in it's pre-ordering phase.

The answer is that they were competitive in 1910 - tens of thousands were sold, and they outsold ICE vehicles for some years. Then widely available, reasonably cheap gasoline arrived and ICEs were not cheaper, but they were more convenient. EVs were no longer needed.

That's right. Something changed. Pave roads changed the playing field. You need to point to something like that today to change the playing field again. PO and AGW are not doing it and you can point to no mechanism for them to do it.

They are more competitive now, because the external costs of oil have gone up.

Plain not true. I know you want it to be but its not true.

Also, batteries have improved - li-ion is a much better chemistry than lead-acid, and it's becoming much cheaper (as measured per kWh-discharge cycle).

And I pointed out above that batteries were just as capable in 1910 as they are today. Did you not read the above article?!? So this is not the game changer to make EVs or EREVs competitive.

There is no point talking to you. You don't debate you merely proselytize.

Pave roads changed the playing field.

Yes, longer range gained a greater value. Well, the range extender of an EREV fixes that.

PO and AGW are not doing it and you can point to no mechanism for them to do it.

I'm just baffled. Obviously PO and AGW have created additional costs. The US $7,500 tax credit is real, and on the books. There is an additional $5,000 in California - that makes the price of EVs and EREVs really cheap. I don't know what else to say.

They are more competitive now, because the external costs of oil have gone up. - Plain not true.

Again, I'm just baffled. You don't think PO and AGW have created additional external costs for oil??

batteries were just as capable in 1910 as they are today.

No, they're really not. L-ion batteries are very different. Heck, lead-acid is somewhat better than it was then. You don't recognize the existence of li-ion batteries??

Did you not read the above article?!?

Sure - that's not what the article said. The article said that the enormous improvements in energy density of batteries have been countered by cars that use more energy per mile. It didn't address the cost of batteries at all.

The Prius is a partial-electric vehicle, and Toyota has sold about 1.3 million of them. The Prius is the best selling vehicle in Japan. Things have changed.

Actually, what changed was the invention of the electric starter motor, so you didn;t have to hand crank the engine. Having hand crank started a 12hp irrigation diesel, I can tell you it is not for the faint hearted!

This development also spelled the end for the steam car.

The ICE was now the fastest starting longest range, instant refueling vehicle - the most convenient. The price of gas had nothing to do with it - if you could afford a car, you could afford the gas.

the invention of the electric starter motor,

When you think about it, this was the first step in the electrification of the ICE vehicle. Small, but real.

The price of gas had nothing to do with it - if you could afford a car, you could afford the gas.

Very likely true - cars were very expensive up until the Model-T. Which reminds me - the Ford assembly line also must have made a real difference in the whole equation.

Henry Ford's assembly line revolutionised manufacturing. You may (or may not) know the reason why he only made it in black is that black paint dried the fastest - to use other colours would slow down the production line.

I think is still a lesson from the model T. Ford's goal was to produce an affordable car that everyone could buy, he did so be doing away with many of the unneccessary frills, using a small engine and drivetrain - it was probably the worst performing car of it's day, but was the best seller.

A modern electric equivalent - simple, small and cheap - would also do well, in my opinion, but the carmakers are of the view that is must be competitive feature and performance wise, with ICE cars. I think a lower spec car, a two seater, that is PRICE competitive with smaller ICE cars would be a better bet.

Another thing about the model T - it was designed to run on both ethanol and gasoline - the first mass produced flex fuel vehicle. 100 years later flex fuel vehicles are touted as being a great new thing, but still only available on specific models - we have gone backwards in this respect.

As for electrification of the ICE vehicle, I think "complication" is sometimes a more appropriate term for modern ones.

he only made it in black is that black paint dried the fastest

Yes, I knew that once (and promptly forgot it, of course).

it was probably the worst performing car

Yes, it was the Yugo of it's day.

I think a lower spec car, a two seater, that is PRICE competitive with smaller ICE cars would be a better bet.

It's been tried. The first Honda Insight was exactly that, and it didn't sell.

Another thing about the model T - it was designed to run on both ethanol and gasoline

Yes, it also made use of soy fibers. The license plate got nibbled by cows, and had to be replaced by metal...

No, not really. I don't see the need for EVs or EREV of any sort. I would argue most people never will. See the gas price graph above.

I'm puzzled. By the same logic, there's no need for rail, or bicycles, or any kind of adaptation to PO or AGW.

Yep. That's pretty much the problem.

Again, I'm puzzled. You don't agree that PO and AGW have created real costs? That oil prices are likely to rise?

No they have not created real costs. Not at the gas pump where they are needed to make EV's competitive. Look at the above gas price chart. And all evidence from the last couple of years makes me believe that PO or AGW will not result in a sustained increase in gas prices.

Its pretty simple. Gas prices rise, economy contracts, gas prices fall back to the long term average. Rinse and repeat. EV's stay forever non competitive.

If they did then yes, you have your smoking gun that would convince me EVs of any sort will become competitive with ICEs.

No they have not created real costs.

Ah - you're talking about prices, not costs. Well, what about the $7,500 federal tax credit, and the $5K California tax credit? That makes the Leaf cost about $20K. Gas savings amount to another $5K. That makes the effective price of the Leaf about $15K. That's cheap.

Now, the Volt may start $5k higher, but with volume that will fall. The Volt has no range problem, and will sell extremely well.

Another factor: Federal CAFE, which requires 37 MPG in a few years, and higher levels later. That will force partial electrification of new car fleets.

Gas prices rise, economy contracts, gas prices fall back to the long term average.

Gas prices are around $3 now. At that price EVs and EREVs are price competitive. The Leaf and Volt prices are competitive with tax credits, and their prices will drop with large volume production.

Now, I don't expect EVs and EREVs to displace ICE vehicles overnight. But, social recognition of the external costs will grow, and EV/EREV prices will fall. We'll get there, sooner or later.

Gas prices rise, economy contracts, gas prices fall back to the long term average. Rinse and repeat.

The long-term average of oil prices fell for decades, until PO.  Since PO it has been climbing.  Today's prices are 2-3 times as high as a decade ago.

EV's stay forever non competitive.

The economy only contracts because high-priced oil sucks too much money out of it.  Enough EVs, and the influence of oil prices shrinks and the contraction gets smaller; oil prices remain higher, and EVs stay competitive even without policy measures.

The economy only contracts because high-priced oil sucks too much money out of it.

The whole question of the impact of oil on the economy is very long and complex, which is why I didn't respond to that before.

The impact of oil on the economy is generally overestimated on TOD - I would say that the various mitigating responses to oil (substitution, efficiency, conservation) are more important in capping the price of oil than a recession response. One bit of evidence for this: Richard Rainwater (who has become a billionaire in large part by market timing oil prices) predicted the peak in oil prices quite accurately, based on the mitigating responses he was seeing before the peak.

True. I'm talking about plug-in series hybrid, aka EREV. I'd estimate that a reasonably large battery (between 10 mile and 40 mile range) is the optimal point to minimize costs. I'd estimate that a 40 mile range is about the right point to optimize convenience and usefulness.

And batteries capable of this have been available since the turn of the 20th century. See the above article for EVs with 100 mile ranges available for the past 100 years.

Did no one think of putting a small engine inline with these commonly available EVs? Obviously not. Porchse did it in 1899.

So what has changed? Not batteries capable of "between 10 mile and 40 mile range".

Uhmm...haven't we discussed this already?

PO and AGW are new. The economic vulnerability of oil importers is new. Oil-war induced terrorism is new.

Again, those are all external costs. They do not make EREVs any more competive.

Tax credits that internalize those costs do exist. Tax credits are on the books, really they are.

Also, of course, some people are willing to take those costs into account. Heck, people take a lot of things other than price into account. Otherwise, why would the average price of a light vehicle be $28K, instead of $12K??

Tax credits are not sustainable on any sort of scale needed to make EV's competitive.

And yeah, some people drive priuses cause they think they are saving the planet. There's hardly enough of them to make a difference.

Tax credits are not sustainable

That's a social choice. They're cheaper than oil wars. OTOH, they won't be necessary forever, as economies of scale with large volume production will reduce EV/EREV prices.

some people drive priuses cause they think they are saving the planet.

Why so disrespectful?

There's hardly enough of them to make a difference.

They're the best selling vehicle in Japan. There a 1.3M on the road, and growing fast.

Sure, they are. The Leaf is cheaper than the average new vehicle (the Volt may be too). When you figure in fuel and maintenance savings, it's substantially cheaper. Now, you may object that the current price depends on a tax credit, and I would reply that the credit is a temporary adjustment for lack of production volume and resulting economies of scale. I could also argue that it's an appropriate internalization of external costs -see below.

You are missing the elephant in the room. The Leaf comes no where close to competing with ICE vehicles on range. Until you solve that problem its a sideshow at best.

The only EV that comes close is the Tesla at $100k.

To claim EVs are competitive you need them to be feature competitive *and* price competitive. To claim any thing else is just silly.

The Leaf comes no where close to competing with ICE vehicles on range. Until you solve that problem its a sideshow at best.

I agree, which is why I think the Volt and plug-in Prius are far more important. They have no performance compromises at all, yet they reduce fuel consumption by 90% and 75% over conventional vehicles.

Nick, I'm tired and don't really want to keep going around and around in circles with you again.

Let me just point out that two posts above this one you claimed that EV's (the Leaf in particular) were competitive with ICE's.

But just now you agreed that EV's (the Leaf in particular) are not competitive.

It's pointless arguing with you.

Yes, I'm getting a little discouraged too - we don't seem to be making progress.

Let me say it more clearly. The Leaf is price competitive, but I suspect that the majority of people won't be happy with it's range. Therefore, EREVs like the Volt will be much bigger sellers for quite a while to come.

The Leaf comes no where close to competing with ICE vehicles on range.

A Leaf towing a generator on a small trailer would be fully competitive with an ICE vehicle.  It could even use a special lightweight low-capacity battery pack so it could carry extra payload; you'd swap it in at the rental shop when you picked up the trailer, and swap it out again when you dropped the trailer off.

Existing CAN bus data protocols are already capable of handling the control requirements to do this.  All it takes is for an auto exec to get together with a rental agency and make it happen.

If it was available at a reasonable price, I'd get a small, lightweight EV with a generator trailer, like this:
http://www.evnut.com/rav_longranger.htm

http://www.dansdata.com/modularcar.htm

For everyday city driving, you don't have to lug around the motor. For long trips outside the city, you attach the trailer. Instead of buying the trailer, you could hire it - that's a simple business model using basic technology.

It's an interesting idea. Technically, it could certainly work.

OTOH, I think most people don't like trailers, so they'd have to save a lot to be worth it. I don't think they really save very much over building the generator into the car.

Oil isn't the only nonrenewable resource that is peaking. My question is will there be enough resources to mass produce enough electric vehicles to replace the internal combustion engine. Rare earth metals are a critical resource for the mass production of EV batteries. Currrently, China controls over 95% of the world's supply of rare earth metals. That might change, thanks to climate change. According to the latest issue of the National Geographic (the Greenland issue),

Greenland Minerals and Energy Ltd., an Australian company, has discovered what may be the world's largest deposit of rare earth metals on a plateau above the town of Narsaq in southern Greenland. The rare earths are crucial in a wide variety of green technologies—hybrid-car batteries, wind turbines, and compact fluorescent lightbulbs....The development of the deposit at Narsaq would fundamentally shift global markets and transform Greenland's economy

But let's not get our hopes up too high, just yet. For although

Narsaq's reserves could sustain a large-scale mining operation for well over 50 years...,there is a major obstacle to developing it: The ore is also laced with uranium, and Greenland's government has a complete ban on uranium mining. "We haven't changed those regulations and are not planning to," Kleist says.

Kleist is Greenland's popular prime minister.

Btw, this electric vehicle also only requires 8 kWh/100 km but at 120 km/h:
http://www.monotracer.ch/index.php?option=com_content&view=article&id=54...

In addition it also fits 2 people and requires less space on the road and accelerates faster than a Porsche.

And 20m2 of PV cells correspond to 3 kW. In other words: 8 hours of sun = 300 km.

WANT!!!!!

Edit:  Ah, I see it's using the AC150 drivetrain; OF COURSE it's a rocket.

Interesting article, though it takes stances that are more opinion than fact.

If today's supporters of EV's would dig into the specifications and the sales brochures of early 20th century electric "horseless carriages", their enthusiasm would quickly disappear.

(?) Not at all. 100 miles range is more than double the average roundtrip commute.

Nobody has investigated how much energy it takes to produce a Tesla Roadster battery, or any other EV battery for that matter

There have indeed been examinations of embodied energy in batteries. NiMH, for example, takes 24,500,000 BTUs (or 7180 kWHr) per 1000 lb of battery.
http://www.osti.gov/bridge/servlets/purl/201715-9UFfKK/webviewable/20171...

One vehicle that is far improved over the Fritchle (fully enclosed from elements) is the Aptera, which only needs 20kWh of LiON battery to go 120 miles.
http://www.apteraforum.com/showthread.php?t=4211

Check out this Japanese solar car that won its international race

http://www.energydigital.com/Japanese-solar-car-wins-international-race_...

The Japanese have it down

Some of those onboard electronics are certainly fluff and have little to do with getting from one place to another, and a good chunk of the rest is only required to babysit the complex machine that is the ICE. It doesn't seem to take a whole lot of electronics for a car to drive itself on 90% of a typical cross-town trip on a network of semi-open lanes reserved and designed for self-driving vehicles, with a speed limit of ~25mph and some separation from pedestrians and the like such as a railing or low chain-link fence. Autopilot probably wouldn't take one off the network, at least initially, so one would probably still have to navigate between the network and one's point of call, but not having to react to traffic or road conditions for most of one's trip is in itself a major selling point.

Long long ago I read an article by someone from Washington state who reverse-engineered traffic congestion and bragged about being able to break jams. His thesis was, roughly, that traffic jams are caused by drivers having varying protocols on how to handle congestion. Cars that drive themselves would not only embody a standard protocol when dealing with exceptional road conditions, but can communicate their navigational intentions with one another and with other traffic controls for a smoother and faster ride.

The upshot of all this 22nd-century technology? The need for fast cars is caused by human drivers who want to minimize time being attentive to the road so that they can do other things. Eliminate the human driver from the equation, free up the intellectual energy they would have otherwise had to spend watching the road for other tasks like communicating, working, studying, knitting, etc., and I believe low speed would be far less of a hindrance to uptake of such a vehicle. You have to wrap that pill up in something to make the animal swallow it, right? If it's a neighborhood EV, it should know the neighborhood, right?

Interesting article. Most would consider this an opinionated "hit piece". I don't have the time right now to go line by line and point out the errors. Suffice to say the author is committed to a particular ideology than uncovering and presenting facts. It is also interesting that the article doesn't cover any Neighborhood EVs (NEVs).

As my moniker shows I'm obviously very interested in electric vehicles and I've been waiting for a highway capable, affordable EV for a long time. Nissan Leaf seems to be the first such vehicle to become available and I shall definitely try to get one in December (since Seattle is one of the early roll out areas).

The most important facts to remember are that - it is not some how the "fault" of "electric cars" - as this piece tries to argue. The problems are elsewhere - in the society we have constructed.

The biggest problem is speed. Reduce that max allowed speed to 35 mph and everything will start looking rosy.
- We will first of all save alot of gas, since high speed vehicles use more gas. Drag is proportional to square of the velocity.
- Electric vehicles don't have to strong and heavy to pass the crash test
- NEVs can then be used on all roads (for which we have all paid)
- Lightweight, super efficient vehicles can be built
- There will be far fewer serious accidents

I don't have the time right now to go line by line and point out the errors.

It took me almost a month to research and write this article (see the list of sources), so if you call this "an opinionated hit piece" then you better do take the time to go line by line and point out the errors.

I also don't understand your intro since your conclusion is the same as mine: reduce the speed.

Kris, I hope you were somehow getting paid for this, rather than just volunteering a month to then get some of the comments you are getting.
I am a regular reader of your Low Tech Magazine, and I always find your articles well researched and well written, including this one. It seems that some (not all) of the commenters here have missed your tagline at the start "we don;t need better batteries, we need better cars". There is no question that more appropriate cars can be built, and the niche players (like Aptera) are trying, but the main makers are still trying to build to "have everything". I am sure there is a market there for something in between the Aptera and the Leaf, but no one seems to be filling it yet.

Ultimately, as you have pointed out in a previous article, the problem is not going electric, it's going wireless. Electric trains and trolley buses (lots of them where I live in Vancouver) have none of these problems and are far cheaper in the long run.

That said, cars will still be around for some time, but ones like the Loremo seem a good bet, though I also like this concept from Australia;
http://www.unisa.edu.au/solarcar/trev/

Paul, thanks for your concern :-)

I am not getting paid for this, unfortunately. I was hoping to receive extra traffic on my website and get some advertising income, but although the story seems to be very popular on metafilter, reddit, stumbleupon & elsewwhere, they all link to this page, not to the original article. This got me a bit depressed, I must confess.

But I don't mind the nature of the comments. I got used to the fact that most people don't read a story carefully. And there are always enough people giving very thoughtful comments, even if they don't agree with me. I learn a lot from these.

I made a special effort to link to your article in Low Tech magazine becasue of this.

http://www.lowtechmagazine.com/2010/05/the-status-quo-of-electric-cars-b...

I hope others do as well.

Alan

Busy with the oil spill and other issues.

Alan, that's very nice of you, but let's keep it like it is. There's links enough to my site on this page.

Hear Hear.

While it is perfectly reasonable to point out errors or possible errors, it is not reasonable to just to use throwaway lines like "this is stupid".

Anyway, generally speaking, this piece has evoked a lot of useful and interesting discussion. Just a couple of bad apples.

I posit that your obsession with range is insane. A function of the ridiculous manner in which a fraction of the population lives forty or fifty miles away from where they work and drive to their workplace alone in an internal combustion engine powered vehicle. A clearly unsustainable model that was at the root of the problem we face.

For those who didn't jump into that life style a range of thirty to fifty miles between charging is more than sufficient for a commute or for running around town and doing whatever needs to be done. You don't need a complicated infrastructure of charging stations, you simply charge the damn thing overnight. Range in fact is a code word for what is wrong with the way we have built out our cities. They are not viable. They are going to have to contract. The idea that every human needs a personal conveyance that hurtles them around for hundreds of miles by themselves is going to be a thing of the past.

Short range, without a backup (i.e. an on board generator) is a recipe for trouble. Picture what happens when inexperienced EV drivers "run out" of juice on the freeway. Some circumstance ( a traffic jam?) might cause an unexpected detour, and increase the chance of running out before you get home. It wouldn't take too many such occurrences causing needless traffic jams before the authorities would act.

Keep in mind that a range of 30-50 miles represents the equivalent of 1-2 gallons of fuel - most of us a re filling up when we get down to that level. With an EV and 30 mile range, every drive would be be rolling the dice - if enough people are driving them, you are bound to roll "snake eyes" sooner or later.

I think this is an important point.

kdd, you mention peak load caused by fast-charging BEVs. Why not charge a big stationary bank of batteries, then dump it into the vehicle all at once? They could use retired EV batteries, which still have plenty of capacity left.

Wouldn't station owners save money this way? They could buy cheap night power, and they'd save money on the flat-rate commercial capacity fee.

Thoughts?

This is being done already with sodium sulfide batteries on Long Island to compress natural gas for buses during the day shift. See
http://www.sandia.gov/ess/About/docs/kishinevsky.pdf

Formica, kdd,

Perfectly possible if you had a standard size Li Ion battery pack - rated at say 16 to 20kWh capacity and standard mechanical dimensions.

Small cars would carry one pack, larger MPVs and SUVs and pick-ups could carry 2 packs.

Simply drive through an automated pack replacement station and replace the pack in minutes.

The key to ev acceptance is that the owner is never responsibe for the battery. A 20kWh Li Ion pack might cost $20,000 to produce. No one wants that sort of financial burden. Better to get the utility companies to own the batteries, and their disposal obligations, and then they sell you the juice. The faster you want to recharge - the higher premium they put on the power. Think of it as a similar business model to owning a mobile phone. Nobody pays the full price for the hardware, you just choose a service plan that suits your lifestyle. If you are in a hurry and you need a 30 minute rapid charge - then you are charged close to $1 per kWH at a fast recharge station. If you just want to recharge overnight at home - then the power is the usual 0.10 to 0.15 per unit.

I recently did a brief study to show that the UK would benefit from a maximum of 10 million evs. That represents a replacement of some 30% of existing small cars. There is probably a similar practical ICE vehicle replacement figure for north America.

The power generators (and renewable power generation) would benefit from having a more constant load - and the evs would be recharged on the excess capacity available during off-peak times. Based on a population of 10 million evs, it would add considerable energy storage to the grid - and if you adopted a vehicle to grid (V2G) scheme to allow power to be recovered from ev batteries at peak times. If you took the V2G option as part of your battery service plan, then you would get your power at an even lower rate.

As more evs are introduced, the utilities will find even more creative business models to keep them charged.

The Nissan LEAF and the Mitsubishi iMiev return better than 4 miles per kWh drawn from the grid. If you calculate the equivalent gasoline these small cars would use, compared to the additional coal and natural gas needed at the power plant - you will see that the ev makes considerably better overall use of the hydrocarbon fuel, and offers the power plant a guaranteed load for load balancing.

I will dig out some of my figures to better illustrate the above points.

2020

"The key to ev acceptance is that the owner is never responsibe for the battery."
Well, maybe, but then, what incentive is there for the owner to look after said batteries? Letting the batteries run right down, and high current withdrawals (i.e. fast acceleration) will reduce both battery life and performance, in the same way as abusing an engine will. Look at the way people drive rental cars, if the batteries are not theirs, they will not care, and the life and performance will suffer. No one will accept a replacement pack that is not in near new condition, so that means batteries will have to be retired before the end of their life, same as how rental car companies can't keep their cars for too long. If people owned the batteries, they would look after them like they do an engine, and as the battery performance deteriorated, they would learn how to drive accordingly, squeezing as much utility out of the battery as possible. Take away the ownership responsibility, and you take away the care factor too.

As for the grid situtation, plugging in EV's does not necessarily make for a more constant load, it actually decreases it, unless carefully controlled. If people plug in their EV's when they get home, you are adding to the evening peak. If you have them set on a timer to start charging at say 10pm, then that is better, as long as you don't have a million of them plugging in at once. Better still is having them charge at 2am or so. But you can't control or predict this, unless you have the v2g system. But then, with this expensive system, the power companies are in control, and the priority is for flattening grid loads, not having fully charged cars. To be meaningful,the V2G needs cars plugged in during the day, and the companies will actually discharge them to soften the peaks. But then when you get an urgent call to see a client crosstown to drive in the afternoon, you car is suddenly "half empty" - what then?
For night charging, a hot summer night and lots of A/C load in southern California, so the utility doesn't want to charge the EV's until it cools off, but it doesn't, so then they have no choice but to charge them, adding to the load.

My point here is that you can use V2G to do some stuff, but you cannot RELY on it, as, ultimately, the EV's have to be charged. The only way the utilities can get a "guaranteed load" is if the EV is plugged in all the time - like a hydro plant ready to go. Anything less is not"guaranteed", it has a "probability of availability" attached to it, which must be less than 100%. And when it is not available, , then you have to have both the grid and generating capacity to handle it. It is the same problem as with wind turbines, they produce cheap energy when they operate, but you cannot guarantee when they will, so you have to be prepared for when they aren't operating, which means you still need other sources of electricity.

The utilities will indeed come up with creative business models, but keep in mind that this decreases convenience to the car owner. It would just get priced in, where peak charging does indeed becomes expensive, but I think the first time someone gets caught with an undercharged car from V2G, that would be the last time they would let that happen.

For fuel consumption, 4 miles perkWh is the energy equivalent of 3.6MJ per four miles, or 0.9MJ per mile. Burning coal at the power plant has an efficiency of 35%, and you lose 10% in transmission, so you are at 32% To get the 0.9MJ, you needed 2.8MJ of coal per mile. If we have the same car running on gasoline, like a Toyota Yaris, (32mpg combined), and gasoline has 137MJ/gallon, then we are at 4.3 MJ per mile. if we look at the state of the art in hybrids, the Prius, at 50mpg, then we are using 2.74Mj/mile. So, if the electricity is coal based, then there is no energy saving with an electric compared to a hybrid, and only a small one compared to a small gasoline car. There is an OIL saving, though not if the car is powered by nat gas or ethanol...

Paul,

Time-of-day pricing can be automated and invisible to the owner. The utility sends price signals to the meter, or to the EV. The EV chooses when to charge, based on the owner's preferences. Most people will set their car to charge at 2 am to take advantage of the lower price. The minority that wants to charge right away can pay the peak price for power.

V2G is nice, but is completely different from dynamic charging. We don't need V2G to get most of the benefit of load-leveling from EV charging.

Let me say it again, because this seems to be hard to communicate:

We don't need V2G for load leveling!!

OTOH, V2G will be quite powerful when it arrives: imagine 220M vehicles (230M vehicles, with 95% availability), each able to put out 4KW (that's only 220V and 20 amp service): that's 880GW of peak power!

Nick, We are both big fans of time of day pricing, but I think it needs to be highly visible to the owner. If you knew that gasoline was always half price from 11pm to 6 am, you would go to quite some effort to make sure that was when you filled up.
Time of day charging should be implemented, continent wide, regardless of EV's.

But as to dynamic charging, yes it is not the same as V2G, but it cannot really be called "load levelling" as it is not shifting any existing loads. All it is doing is adding new load during the off peak times. This is a good thing, but I don't think load levelling is the right term for it.

V2G as a concept is fine, but heaven help us in the future if we have replaced 220m ICE vehicles with 220m electrics. Hopefully, by the time electrics have displaced ICE substantially (2-3 decades) there will only need to be 100m or so vehicles on the roads. To get enough Ev's to do meaningful V2G is at least a decade away, so we cannot wait until then to address peak load problems.

I will agree that ev's, properly done, should not add to the peak, but they are a long way off reducing it. Time of use rates are the place to start there.

Nick, We are both big fans of time of day pricing, but I think it needs to be highly visible to the owner.

I agree. In fact, I recently installed a whole-house energy monitor. The display sits in the living room, where everyone can see it. It's changing people behavior, too....

The point is, though, that it's easy. You can set up EV charging paramters once, and forget it.

V2G as a concept is fine, but heaven help us in the future if we have replaced 220m ICE vehicles with 220m electrics.

I don't know why - if the power is cheap and low-impact, why not?

To get enough Ev's to do meaningful V2G is at least a decade away, so we cannot wait until then to address peak load problems.

1st, dynamic charging is the important thing. My point: there is no conflict between pursing PO mitigation with EVs and mitigating AGW with wind power, in fact there's a powerful synergy. 2nd, even 1M EV's could provide 10GW of demand or supply (with V2G) when needed. 3rd, grid services from a small number of EVs can be disproportionately valuable - you'd be surprised.

I will agree that ev's, properly done, should not add to the peak, but they are a long way off reducing it. Time of use rates are the place to start there.

I certainly agree that time of use rates are the place to start. OTOH, adding demand at night is awfully valuable to wind providers.

V2G as a concept is fine, but heaven help us in the future if we have replaced 220m ICE vehicles with 220m electrics.-
I don't know why - if the power is cheap and low-impact, why not?

Because then nothing else has changed - we consume excessive land and energy (for transport) and all the other resources, particularly water, that go along with suburban sprawl. I sincerely hope that along thew way we reconfigure the cities and economy so that a car is an option, not a necessity, for the majority of people.

Well...I know what you mean. I like cities - that's where I live.

OTOH, I don't know why we would care about energy consumption, if it's cheap and very low-CO2/pollution.

Moving everyone into cities is a very, very slow and expensive proposition. I hope we become affluent enough to afford it, but it's not a major solution to our energy problems in the next 20 years.

Why not do it twenty years ?

It took just 21 years (1950-1970) to trash virtually every piece of prime commercial property in the USA (called "downtowns") and well built, well located, established neighborhoods (called inner cities).

Post-Peak Oil, it should be much easier to trash badly built, poorly located Suburbia. Especially with some government help (turn about is fair play). Say, a 3/4% mortgage "risk premium" if the house is > 3 miles from Urban Rail or Electric Trolley Bus, 1/2% if > than 1 mile and 1/4% if > 1/3 mile.

Pay for city street maintenance with gas and EV taxes (instead of property taxes as is today), and maintain them to a high standard, while neglecting Suburban infrastructure.

The risk premium covers the greater risk of default and losing value.

Bet we could do it in less than 20 years with a little government help. As I said, turn about is fair play.

One a week mail service in Suburbia, 6 days/week in the mailperson can walk the route. Keep city streets up, but stop maintaining highways leading to Suburbia. At the end reduce lanes (6 > 2) and so forth. No federal aid to schools where over 3/4ths of the kids do not walk or bicycle to school. And so forth.

With appropriate gov't help and a MASSIVE urban rail program, etc. maybe in just 12 years ?

Alan

It took just 21 years (1950-1970) to trash virtually every piece of prime commercial property in the USA (called "downtowns") and well built, well located, established neighborhoods (called inner cities).

That was post-WWII, with a dramatically expanding economy, and a shortage of housing. Those downtowns and inner cities weren't abandoned - they mostly received new occupants, just at lower income levels.

Now, we have an enormous oversupply of housing. No one will agree to subsidize building new, unneeded housing. Further, no one will agree to raise the price of mortgages in order to trash the suburbs - that's really, really unrealistic.

And, they shouldn't, at least to save energy. The costs (and additional embedded energy) of such a program are many, many times the energy savings.

Downtowns and inner cities were abandoned. Boarded up stores and homes were, and still are, common. Some have been rehabilitated after decades of abandonment, others have not.

It is entirely proper to find city streets from gas and EV taxes. Today I subsidize suburban sprawl every time I drive (almost always on city streets).

It used to be that a mortgage could not be >30% of your income. See foreclosures.

A more restrictive (and hence more likely to attract investor $) would be mortgage plus transportation <45% of income.

Post-WW II, city home buyers had to pay a higher interest rate than suburban home buyers. Why not the reverse, especially since Suburbia is so dysfunctional ?

YOu underestimate the energy cost of Suburbia and VASTLY overestimate the energy cost of building energy efficient, durable TOD.

Lumber is a very low energy building material and it can make up the bulk of new construction.

Use CHP to reduce the HVAC and hot water infrastructure costs of TOD. (and 3/4 or 1 ton of a/c for an energy efficient TOD home vs. 8 tons for a McMansion certainly saves). Note that residential a/c lasts only about 15 years on average (several variables in that #)

Some materials can be recycled from McMansions (bathrooms and kitchens come to mind, although I would leave the plastic shower stalls) with energy and capital savings.

Alan

Downtowns and inner cities were abandoned. Boarded up stores and homes were, and still are, common. Some have been rehabilitated after decades of abandonment, others have not.

The essence of white flight was that blacks moved in, and whites stopped moving in, and started moving out faster. Blacks replaced whites. Some stores and houses were abandoned, but most weren't.

It is entirely proper to find city streets from gas and EV taxes.

I agree. I would note that both cities and suburbs pay for streets with general funds, so the difference isn't that dramatic.

YOu underestimate the energy cost of Suburbia

Well, we've been through this before. Suburban housing and transportation can be made to be just as efficient as we want it to be. Existing suburban housing is more efficient than existing city housing.

VASTLY overestimate the energy cost of building energy efficient, durable TOD.

Unneeded housing is mighty inefficient.

Lumber

Don't forget - NOLA is not dense urban living. You don't build medium or high rises with wood.

McMansions

The most efficient use of McMansions would be multiple occupancy by suburbanites - far cheaper, more efficient than building new housing.

Over half of downtowns went empty or the lowest form of use (storage, seasonal use only, etc.) and quite a few cities have half of their peak population.

I have seen five and six story housing built with wood.

Efficient TOD housing need not be Manhattan. New Orleans ties with NYC for fewest VMT by residents and we have a MUCH more human scale built form.

Existing urban housing is more efficient if the age of construction is included. Most New Orleans housing was built before fiberglass insulation was invented for example. Fiberglass in walls vastly improves energy efficiency.

It takes great amounts of energy to run and maintain a McMansion, including teh supporting infrastructure.

If Suburbia collapses into 1/4th it's existing ft2 (as I suspect it might), that will be fine. I suspect the residual fraction of Suburbia will be close to commuter rail.

Alan

quite a few cities have half of their peak population.

What city would best exemplify this, would you say?

I have seen five and six story housing built with wood.

But do you think that would be the norm, if we moved 3M more people into, say, Chicago or Boston?

New Orleans ties with NYC for fewest VMT by residents and we have a MUCH more human scale built form.

Yes, and NO is very low density - it's nothing like any of the major big cities, and hardly an example of high density urban living (and yes, that's taking the parks into account). My wild guess is that it's rail heritage is an accident of it's history, not a result of it's current density.

Existing urban housing is more efficient if the age of construction is included

Sure, but new suburban housing has gotten more efficient too. The fact is, existing suburban housing is more efficient than urban housing, and we can build zero-energy passiv-haus-type buildings in the suburbs much cheaper than moderately low-energy urban housing.

It takes great amounts of energy to run and maintain a McMansion, including teh supporting infrastructure.

Not really. Besides, put in new windows, maybe a heat pump - much, much cheaper than moving.

I have seen five and six story housing built with wood.

But is this something that could be built today?  More to the point, can 5+ story residential buildings with acceptable acoustic and fire-resistance properties be built out of lumber?

I mention acoustics because noise from neighbors is part of the unwanted social interactions of too-dense living.  People will leave cities just to be able to get a good night's sleep, and if you ignore such considerations your TOD ideas are doomed to fail.

This was new construction in Baltimore (a couple of blocks from Johns Hopkins) and Portland.

Wood has good acoustic properties (especially, my observation, higher frequencies). Construction (some batts between the studs on interior walls is Step 1, heavier dry wall, offset studs and otehr techniques for Step 2).

Almost everyone can acclimate to whatever background noise.

Alan

PS: I miss the sound of birds when I visit Suburbia. I can hear one right now as I type.

Almost everyone can acclimate to whatever background noise.

That's highly...unrealistic. It depends on the listener, the sound, and the situation. A wonderful sound becomes unbearable in the wrong circumstance, and people will go to great lengths to find the right arrangements - I have.

They absolutely can be be built five or six stories, using engineered lumber (parallams, LVL's, etc) . Four floors is the limit for stick frame in Canada, but go engineered and you can go much bigger
Heavy timber construction can have a longer stand up time in fire than steel, which once it reaches the yield temperature loses strength rapidly. The normal building fire codes deal with all this, how to design 1, 2 hr firewalls (for either steel or wood) and so on. The biggest risk fior fire is during construction. Once built and properly sprinklered, they are as fire safe as a steel building

As for noise, that can easily be dealt with using acoustic insulation and acoustic drywall (sound deadening equal to 8x layers of standard drywall).
For ceilings there are ways to decouple the attachement from the beams to limit noise transfer, etc.

Condo developers have not been know for going to great lengths on soundproofing, but that is because they are cheap, not because it can;t be done. A simple revision of the building code to ensure a certain level of sound attenuation for party walls and floors will address this.

For a good example of what can be built with wood, if you really want to get creative, take a look at this;

http://www.topboxdesign.com/road-bridge-krusrak-by-onix-and-achterbosch-...

There's no question that noise problems can be prevented. Sadly, existing building codes set pretty low standards, and I don't see much movement towards improvements here.

Have you seen any sign of improvements in general building practices, professional building standards and/or codes?

Actually, I have sen an improvement, though not in codes. Any of the "green" buildings being built today (LEED certified, etc), are being built better. This is usually because you have a client who wants a quality building (better than minimum) and is prepared to pay designers to do it. Depending who clever they are, the building itself can be no more cost, and even cheaper, but there is more design work involved.

Condo developers, on the other hand, build a product, or say they will build a product, to then sell. They spend lots on things you can see (finishes) and the least possible on what you can't. Any custom building is invariably built to the higher standard. Building codes normally run 10-15 yrs behind the state of the art, so they will get there eventually. But with a trend towards densification, now is the right time to take a good look at these - the next generation of housing can and should be built much better.

So, it looks to you like essentially all custom buildings are now being built more or less to a "green"/LEED standard? That's encouraging. What % of buildings would you estimate are custom? My wild guess is less than 10%.

with a trend towards densification

Is that trend really there, or are we just hoping? I see more in-fill development in more established areas, but that seems to simply reflect the relative prosperity of the folks living in those areas. Sub-prime buyers bought new tracts in exurbs because that's where building was cheapest, and those exurbs are now suffering due to the concentration of sub-prime buyers.

Building codes normally run 10-15 yrs behind the state of the art

Have you seen any systematic moves to "green" building codes lately? I haven't seen much - I suspect everyone is afraid to do anything that might supress new construction coming back.

Much that energy rebounds back into the vehicle. Nearly 100% with a large tree or solid brick wall and quite a bit even when another car is hit.

Of course, EVs killing and injuring more people in the "other car" or pedestrians due to their greater mass is also a social negative. All other issues being equal, I would like to live in a world where the dominant EV is an eBike.

Alan

I didn;t say that all custom buildings are green, but they are generally built to better quality standards. However, almost all green buildings are custom, so yes, it's only a small % of some markets. For gov/institutional buildings, more than 50% planned or being built are green. Numbers are very different for commercial and residential. For single family residential, well, it has virtually stopped, so we'll se what the next generation is like.

The trend towards densification is partly illustrated by the fact that in many cities, the only housing being built is infill - there is virtually no new construction in suburbs, and no new suburbs being built. Here in Vancouver, where we didn;t have the sub prime problem in the first place, first time buyers are buying condos, because that's all they can afford. First time buyers are the only ones buying, because they didn;t lose any money from the property downturn - if they still have their jobs, and most here do, buying a first house is cheaper than it has been for years. Vancouver has a new rule now for allowing "laneway"housing, a carriage house facing onto the back lane. This is proving VERY popular, and is certainly densification. Most permits are for areas near transit lines, or proposed ones.

As for green building codes, you just need to live in a progressive city. Vancouver is adding all sorts of requirements, like pre-plumbing for solar hot water, electric car charging circuits. Check it out at;
http://vancouver.ca/greenestcity/cityservices.htm

This is not a perfect approach, but it's a good start, and clearly lays out the city's aims - you could do a lot worse, (and many cities do)

They are not suppressing new construction, but there is general agreement that new construction will fit these guidelines, if the developers don;t like it they can go elsewhere. They don;t want to go elsewhere, of course, as Vancouver is the desirable place, so they follow the cities guidelines. Very simple, debate and decide the policies, make the rules very clear, and stick to them. That way, people who want to do otherwise either change or go elsewhere. There is no attitude of "construction jobs at any cost" here. Vancouver is the only major city in Canada to not have a Wal-Mart, and the people will fight tooth and nail to keep it that way!

... if the electricity is coal based, then there is no energy saving with an electric compared to a hybrid, and only a small one compared to a small gasoline car. There is an OIL saving, though not if the car is powered by nat gas or ethanol...

No, you're leaving things out again.  If the EV is charged from a gas-fired CCGT, the pipeline-to-plug efficiency is ~54%; if you have a mix of perhaps 20% nuclear, 20% wind, 10% hydro and 50% NG (not far from where Iowa and Texas could be in a few years) and half the NG generation is simple-cycle turbines at 46% before line losses, you're still getting 47% average (and 94% based only on fossil-fuel consumption).  This is way beyond what any light-duty ICE vehicle, burning CNG or anything else, can get.

EP, I specifically said IF the electricity is coal based, which is what we are trying to eliminate. And we have a long way to go before it is eliminated. Adding EV (or any other) loads to the grid will only slow this process down

Adding EV (or any other) loads to the grid will only slow this process down

This is where we started this discussion!

My point: EVs support wind power by adding night time demand and a large amount of easy powerful DSM, so adding EVs only accelerates wind power installation.

Given that powering an EV with the average utility mix of generation sources generates less CO2 than gasoline (or diesel), there's no conflict at all between EVs and AGW mitigation.

IF the electricity is coal based, which is what we are trying to eliminate.

You're talking as if there is only one goal here.  There isn't; oil is equally important, and more important to the state of the economy.  Furthermore, you've got the goals wrong; eliminating coal isn't as important as eliminating its emissions, which is a different thing.  We can still use some coal as e.g. chemical feedstock and stuff any oxidized carbon underground.

Adding EV (or any other) loads to the grid will only slow this process down

No, it will speed it up.  In addition to providing a massive DSM resource to support wind, EVs will eliminate the political constituency for coal-to-liquids.

This could indeed ease the strain on the grid, but there is a price to pay when you introduce another layer of batteries. When charging a battery straight from the grid, part of the energy gets lost. When further charging the car battery with the stationary battery, again part of the energy gets lost. So it seems to me that you can limit the problem of peak demand only by reducing the overall energy efficiency of the system.

First, most of the load-balancing value of EV's comes from charging at the right time (either at night, or during the day at peak wind/solar output), so there are no losses.

2nd, li-ion is about 93% efficient, which would reduce losses from V2G or larger storage schemes (service station or central utilities) compared to other battery chemistries.

I propose we analyze this problem from the standpoint of automotive morphology and the nature of human nature. A taxonomy of automobiles would run something like this: Version 1.x is based on horse drawn wooden carriages; V 2.x is steel ladder frame with wood body and four wheels; V 3.x is all steel construction with two sets of full-width seats; V 4.x is unitbody construction. We are at the Nth iteration of Version 4 and have been stuck there for 40 years, albeit with constant improvements. One could say that the introduction of electronics and safety and pollution control priorities could argue for inclusion of Version 5. But the basic morphology of the vehicle was set back before WW I.

Humans are a generalist species that prefers extensive utilization of resources over intensive. We use movement as a problem solving tool. Research by Yakov Zahavi and Cesare Marchetti have shown humans to have had a markedly constant daily travel time since the Neolithic period. The distance changes with the mode of travel, but the time, about 1 1/2 hours per day, stays the same.

This need to move has also had a powerful effect on human habitation patterns. We are tropical savanna animals and want to spread out as much as possible. The fact that there is only one New York City and thousands of suburbs in America should tell us something about our preferences.

We prefer our travel to be isotropic and schedule free. That explains the explosion of interest in the modern safety bicycle at the end of the 19th century. This was the first invention which significantly increased travel speeds (by a factor of four over walking) for the general public at a lower cost than horse ownership and maintenance.

As TOD readers know, most of the time (close to 7/8ths) we travel alone on a regularly scheduled run (the daily commute). A range of 100 miles for a BEV is a functional distance, considering the need for power robbing HVAC, side trips, unexpected problems, etc. We have no end of multi-purpose vehicles but no single purpose vehicle specifically designed for this most important daily travel task. Radical downsizing is the "no brainer" solution; however, it is one rarely investigated, because we are so married to existing automotive morphology.

A narrow, three-wheel, tadpole shaped, in-line two-seater with moderate performance appears to be the morphology with the most promise. It needs to cost half as much as a standard sedan and occupy half the garage space. An automotive version of the Piaggio MP3 scooter is what I would use as a starting point.

Many argue for increasing the density of cities as a transportation and infrastructure problem solving solution. However, solving the problem through real estate would cost several orders of magnitude more than just dealing with transportation needs directly. One could also argue for more public transport, but that entails tieing society down to linear, scheduled transport with high public infrastructure costs. With today's anti-tax mood, that's not going to be easy to accomplish.

Simple, single-purpose, low cost, and recyclable are the parameters that will need to be followed for effective personal transport.

We are tropical savanna animals and want to spread out as much as possible. The fact that there is only one New York City and thousands of suburbs in America should tell us something about our preferences.

This seems plausible, but does it come from your intuition, or is there research that backs this up? After all, American suburbs aren't quite as common in some countries; a lot of people are moving to pretty dense cities in some parts of the world; a pretty large % of Americans does live in pretty dense urban settings; and more would move there if not deterred by the much higher costs of central city living.

I'm referring to Anatomically Modern Humans (Homo sapiens sapiens). That's us. We've been around for roughly 130,000 years. Our evolutionary history is quite well known, from archaeological finds and genetic analysis. There is a huge body of juried literature available detailing our origins, but any college anthropology intro text will give you a pretty good introduction.

There are numerous dense cities around the globe, especially in the developing world, where, I would posit, that the density does not come from choice but is a function of poverty. As individuals become wealthier, they tend to spread out. Certainly, the automobile and the ready availability of land in the U.S. made us especially prone to spread out (read The Crabgrass Frontier); however, if we didn't want to spread out, we wouldn't have, even if the ability to do so were there. We are, after all, social territorial carnivores, and spreading out is a way to avoid conflict over resources. We are a very dangerous animal. Sometimes we forget we're animals.

density does not come from choice but is a function of poverty. As individuals become wealthier, they tend to spread out.

BS !!

Take Paris (the poor live the suburbs/slums) or Manhattan from Central Park south or San Francisco and many more.

What you postulate as a fundamental of human nature is just herd instinct, white flight from integration, post WW II government policies and an aversion to social contact/preference for social isolation.

The obesity epidemic is proof that the Suburban Way of Life is NOT natural, or healthy.

Alan

Alan,

I think I'm agreeing with you here, but I have a quibble: I don't think obesity is a data point for unnatural living. I think it's perfectly natural for humans to gain weight in the presence of abundant food (I'd guess that the average weight of NOLA residents, like the US in general, supports that). It will take our culture quite a lot of work to counteract that natural tendency.

40 years ago food was as available (if not more so, see seafood) as today but morbid obesity was less than a third of today (from memory).

I have heard no reports of famine in Germany or France (quite the opposite) but their rates of obesity are a small fraction of ours.

The biggest factor in US obesity (a contra-survival trait) is Suburbia and an auto dependent lifestyle.

EVs will just prolong that.

Alan

40 years ago food was as available (if not more so, see seafood) as today

No, food is far cheaper and more widely available today. Forty years ago fast food barely existed, and food prices are far lower compared to income.

I have heard no reports of famine in Germany or France (quite the opposite) but their rates of obesity are a small fraction of ours.

German obesity rates are fairly close to those of the US. France, of course, handles food rather differently.

The biggest factor in US obesity (a contra-survival trait) is Suburbia and an auto dependent lifestyle.

I'm sure it's a factor, but not the biggest. Again, are NOLA and Manhattan rates of obesity very different? I'd be very surprised. Further, obesity rates are rising dramatically everywhere in the world, not just in the US.

Actually, I'd say another factor is very important - modern paranoia. Modern US parents are obsessed with "stranger-danger", and therefore don't allow their kids to walk or bike very much. I blame Oprah...

Manhattan has about the lowest rates of Obesity in the USA.

Food, as a% of median income, was cheaper 40 years ago.

Fast food is a symptom of the auto dependent lifestyle (plus a major corporate profit center which means lots of resources are devoted to expending it).

Alan

Manhattan has about the lowest rates of Obesity in the USA.

First, I'd be curious to see your source, in order to see the quantitative difference. 2nd, what about NOLA?

Food, as a% of median income, was cheaper 40 years ago.

I believe that's really not correct. Do you have a source? Here's something current up to 1997: http://www.ers.usda.gov/publications/sb965/sb965.pdf . Look at figure 29, on page 70. Also, figure 8 on page 49.

Fast food is a symptom of the auto dependent lifestyle

??? Fast food is more popular among the very poor, who don't even have cars. Obesity rates are higher among that group, as well.

Obesity rates

USA - 30.6%
Germany - 12.9%
France - 9.4%

http://www.nationmaster.com/graph/hea_obe-health-obesity

We are 2.3 times as obese as the fat Germans (not that the Germans do not have a problem).

How will EVs help with this ?

Alan

Manhattan has the lowest overweight + obese % of any of New York State's 62 counties.

http://www.nytimes.com/2009/07/22/nyregion/22fat.html

1970 food prices
http://www.thepeoplehistory.com/70sfood.html

CPI is 5.66 times higher today (median income is stagnant vs. 1970).

All in all "same food" prices are comparable, as are median incomes (stagnant since 1973). So 1970s food prices are NOT cheaper, but neither are they significantly more expensive. Cheaper food did not cause an explosion of obesity since 1970 !

Alan

Obesity is more dependent upon education and income. 56.2% of the population had a Bachelor's degree in Manhattan.

Nearby in the South Bronx 25% of the adult population is obese.
http://www.nyc.gov/html/doh/downloads/pdf/dpho/dpho-bronx-obesity.pdf

Obesity rates: USA - 30.6%, Germany - 12.9%

That's interesting. That's a wider difference than I've seen elsewere.

We are 2.3 times as obese as the fat Germans

That overstates things a bit. Obesity is a threshold, so Germans could be, on average, 20% overweight, and Americans 25% overweight, and the difference in obesity rates would be very large.

How will EVs help with this ?

1) they won't hurt. 2) there are a lot of other causes besides suburbia - note my comment on "stranger-danger". 3) even if suburban living reduces exercise, this isn't an energy related question.

More than 62 percent of Bronx residents are overweight or obese, higher than the rate in any of New York City’s other four boroughs, while the comparable figure for Manhattan was just over 42 percent — the lowest of any of New York State’s 62 counties. The city’s other boroughs had overweight and obesity rates more closely in line with the statewide average of nearly 60 percent: 58.6 percent in Brooklyn, 57.7 percent on Staten Island and 57.6 percent in Queens.

So Manhattan's rate was 42%, vs a state average of less than 60%. So, perhaps Manhattan's average weight is 3-4% lower. That's not a big difference.

CPI is 5.66 times higher today (median income is stagnant vs. 1970).

That's not the right comparison. Median income is stagnant because women entered the workplace - you have to use household income, or total income.

All in all "same food" prices are comparable

What's your source for that? AFAIK, food commodities, adjusted for inflation, are much less expensive than 1970. Flour, chickens, dairy products...all cheaper, adjusted for inflation.

What you postulate as a fundamental of human nature is just ... white flight from integration

If you can separate integration from crime and abysmal public schools, you might have a point.  The average US murder rate in 2008 was 5.4 per 100,000, NOLA was 63.6 (Michigan right at the average, Iowa less than half the average).  NOLA schools are among the worst of the worst (as are Detroit's).  And any attempt to keep known troublemakers out of good neighborhoods, or enforce standards of behavior so that teachers can teach instead of trying to manage chaos, runs into the civil-rights enforcement apparatus.  For people who can't afford gated communities and private schools, distance is the only solution.

Look at it this way:  if you were willing to crack down on the human problems, you could get rid of the distance and the fuel consumption currently used to maintain it.

... and an aversion to social contact/preference for social isolation.

Humans did not evolve in massive cities.  Too much social contact overwhelms and causes overstress.  People need to be able to control it, and removing that control eventually breaks things.

runs into the civil-rights enforcement apparatus.

What are you thinking of here?

People need to be able to control it

I'm not sure what you mean - could you expand on this?

runs into the civil-rights enforcement apparatus.

What are you thinking of here?

Things like Arne Duncan complaining that schools only seem to suspend blacks... in the context of a big enforcement push in the Education's civil rights division.  In other words, he wants racial quotas for punishments instead of schools that are safe and orderly.  Duncan would force schools to ignore blatant racial bullying (sorry about the source but the WaPo link is expired) if too many students of "oppressed" origin were being punished for it.  In practice this means the offenders get carte blanche, the schools become battlegrounds and anyone with the means moves away.

You cannot force people into cities if that's the result.  Instead of today's low-grade civil war, you'll have a hot one.

could you expand on this?

People need safe spaces away from irritating people, and especially those who are hostile or predatory.  Or just too damn crowded; lots of people overstress in crowds.  I'm sure Alan, with his tropism for crowds, sees very few of them; he may think they don't exist.

You might find this interesting:

"Schnur, who runs a Manhattan-based school-reform group called New Leaders for New Schools, sits informally at the center of a network of self-styled reformers dedicated to overhauling public education in the United States. They have been building in strength and numbers over the last two decades and now seem to be planted everywhere that counts. They are working in key positions in school districts and charter-school networks, legislating in state capitals, staffing city halls and statehouses for reform-minded mayors and governors, writing papers for policy groups and dispensing grants from billion-dollar philanthropies like the Bill and Melinda Gates Foundation.

Bill Gates, along with Education Secretary Arne Duncan; Teach for America’s founder, Wendy Kopp; and the New York City schools chancellor Joel Klein could be considered the patron saints of the network....

Schnur, who is 44, became interested in education when, as an editor of his high-school newspaper, he read a draft of an article from a student who had transferred from a Milwaukee public school to his school in the suburbs. “She was savvier than any of us on the editorial board, but the draft was just so terribly written,” he told me. Schnur added that “the more I got to know her, the more I became obsessed with why public education hadn’t reached people like her.” After graduating from Princeton, he worked in the Clinton campaign and then landed an education-policy job in the Clinton administration.

Schnur recalls that when he met Barack Obama before his Senate campaign in 2004, and heard him talk about education, “I figured this guy could be the great education president — in 2017.” When Obama moved up the timetable, Schnur joined his 2008 campaign as a policy adviser. Six months later, he was working as a counselor to Education Secretary Duncan. "

http://www.nytimes.com/2010/05/23/magazine/23Race-t.html

The decline of all inner cities is a direct result of government policies, starting with VA loans only for new homes and not existing ones and new highways.

It was not caused by a social problem, it was caused by a sustained series of governmental policies. The social problems are the result. And racism and white flight are at the root of the social problem called Suburbia.

Post-Katrina there is a consensus that the schools are getting dramatically better. I live two blocks from what will likely be the best public school in the entire USA.

The problems of New Orleans are being solved with sky high levles of citizen involvement and lots of hard work together.

Alan

Do you mean things like black racism against asians in Philadelphia?

Silly question, I know (of COURSE you don't).  But that's one of your biggest problems, and ignoring it won't make it go away.

When NOLA's murder rate falls to 5.4/100k, and the biggest problem in schools is finding teachers smart enough to head all the AP classes, let me know.  I'll be glad to hear it.

I'm referring to Anatomically Modern Humans (Homo sapiens sapiens). That's us. We've been around for roughly 130,000 years. Our evolutionary history is quite well known, from archaeological finds and genetic analysis. There is a huge body of juried literature available detailing our origins, but any college anthropology intro text will give you a pretty good introduction.

I'm not aware of anything in that literature that specifically supports the idea that suburbs are a more natural habitat for humans. The connection seems pretty speculative, and in my experience speculation about the evolutionary origins of our psychology are pretty unreliable.

There are numerous dense cities around the globe, especially in the developing world, where, I would posit, that the density does not come from choice but is a function of poverty..

There are a lot of densely packed slums. Those people are fleeing even greater rural poverty. I don't see support there for the idea of people preferring to spread out.

if we didn't want to spread out, we wouldn't have, even if the ability to do so were there.

Sometimes we spread out, sometimes we don't. The history of humankind is one of gradually increasing population density, and greater urbanization. Heck, the average modern US suburb is probably denser than the average 19th century US central city.

Evolutionary psychology is a powerful analytic and predictive tool for understanding human culture and behavior because it focuses on deep structure rather than surface variability.

Ancient cities were dense because transportation was dependent on walking speed of three mph. Zahavi and Marchetti show that this limited those urban areas to a radius equal to a one and a half hour walk from the outskirts to the city core and back. Ancient Rome had a huge traffic problem, which it solved by limiting wheeled traffic to night time hours only. On the Northern Plains, towns were placed at seven mile intervals on the railroad because that allowed any farmer to go to the grain elevator and back with a horse drawn wagon in one day.

Robin Dunbar has done extensive studies on natural community size in primates, including humans, and has determined that the mean size for human communities is 150. Worldwide, this is about the size of a standard village. It is the maximum number of friends one can reasonably maintain on Facebook. It is the size of a Hutterite colony when it gets ready to split in half. When we live in dense urban areas, we recreate artificial villages. We're not ants, for whom thousands of individuals in a colony are the norm.

My main point, however, in my original posting is that a transportation problem should be solved with transportation methodologies, not through real estate change, which would be several orders of magnitude more complex and costly. If commuting is the number one transportation need, then the best morphology to accomplish that task should be the primary design tool. Instead we get more iterations of the same shapes we have used for the last century.

Evolutionary psychology is a powerful analytic and predictive tool for generating interesting hypotheses. Then they need to be tested, to ensure our prejudices and incomplete understanding of evolution haven't gotten in the way.

Robin Dunbar has done extensive studies on natural community size in primates, including humans, and has determined that the mean size for human communities is 150.

That's interesting, but does it tell us anything about density?

a transportation problem should be solved with transportation methodologies, not through real estate change, which would be several orders of magnitude more complex and costly.

I agree.

If commuting is the number one transportation need, then the best morphology to accomplish that task should be the primary design tool.

That's one approach, and it's a good one as far as it goes, and to the point that it doesn't involve problematic compromises. Another is a drive train that uses cheap and low-CO2 electricity.

The sprawling suburbs of the U.S. are really a result of the economics of the country. It has been built on cheap gasoline and cheap land. Europe is different in that gasoline is much more expensive and land is much more expensive.

The problem is that gasoline is no longer cheap, even for Americans, and U.S. population density has been rising (in the areas which are heavily populated, i.e. those parts which are with 100 miles of an ocean or Great Lake) to the point where they are close to European densities, and increased competition has been pushing up land prices.

This completely wrecks the economics of the vast, sprawling suburbs of America. I think we've been seeing the fallout from this in the past couple of years. The U.S. needs to completely rethink the layout of its cities. Actually, it should have done this 40 years ago when the handwriting appeared on the wall in giant, flaming letters. But most people missed it, or at least tried not to read it.

Europe is different in that gasoline is much more expensive and land is much more expensive.

Europe is different because it's older, and was built pre-auto.

gasoline is no longer cheap, even for Americans

As someone else has pointed out in a comment, gas hasn't yet become expensive.

U.S. population density has been rising

But not because of gas prices.

This completely wrecks the economics of the vast, sprawling suburbs of America.

An EREV like the Volt restores those economics. Even if Volt costs don't come down with economies of scale (which is a wholly unrealistic idea), the premium would be far, far cheaper than rebuilding American cities.

I made a very similar post a little while ago (albeit no technicalities)

Electric cars: 100 years and no progress.

http://theanphibian.livejournal.com/311440.html

And this was just based off 2 different articles I saw on the internet. The observations, however, fall well in-line with this post on TOD and I'm happy to see the observation echoed in different places.

In terms of a solution, I'll give a hat tip to Will Stewart for mentioning it first, but the Aptera addresses the exact problems highlighted by this post. It does it in a modern way that makes sense. Advancing technology can continually reduce friction. I bet it's a lot more comfortable than those old vehicles too! There is a comfortable solution available, but not without changing lifestyle.

Aww, might as well add two cents. I've been gnawing on this bone for a half century, with help from my extremely technical and theoretical dad (designed megawatt sonar, etc.)

In 1987 Honda produced the Helix, a luxury long-wheelbase 250 cc watercooled OHC variable belt drive scooter. They ran forever. I'm driving one now,~70mpg, very very quick in the city. It's easily runs the freeway, 65+ top speed two-up, well protected from the elements. There are now dozens of imitations for $3k. My other car is an Echo, 130K+, total repairs <$1k, avg mpg 35.

The rest of the world has solved the transportation conundrum. The USA hasn't. We insist on mobile cages because we can't think. A few of us do, and hang out here and other places, but NEVER use the assumption that Americans will get sane about transportation. There are little blips of understanding this in the article, but basically, it was a month of useless research.

Sorry, but http://www.worldchanging.com/archives/004398.html and 40mph freeway right-hand lanes with ferocious enforcement by (of course) motorcycle cops would solve most of the problem, by allowing sane electric hybrids. Re onboard charging for emergencies: Small generators like the Honda EU series are very reliable and economical and clean. I have one of them, also.

I had an electric motorcycle in 1981, but I'd never recommend it to people who drive in real life conditions. You need a shelter for the nasties. Most places have rain, many have ice and snow. Stay real.

Check out the taxibus or avego on der Google if you do nothing else. They actually make it work.

The rest of the world has solved the transportation conundrum. The USA hasn't.

You seem to take the view that the author of this article is American. I am not. I live in Europe, and I can assure you that the rest of the world has not solved the tranportation conundrum. It's worse in the USA, but it's bad everywhere.

Kris,
You forgot to include a third option

Realistic electric vehicles: scenario 3

A vehicle having the size and performance of a sub-compact ICE vehicle, able to travel in EV mode for most trips(40miles), a modest battery size (5-10kWh usable power) rechargeable overnight at home(or at work) and a back-up high efficiency engine able to run on ethanol giving a range of 300miles.
This sounds like my next vehicle purchase here in Australia with gasoline at about $US4.50 a gallon, running costs will be similar, but who really thinks we will have these low prices in 5-10years.

As Nick suggested a vehicle such as the Chevy Volt appears to be what most people will want, and be able to afford, and should be a dramatic improvement on 1910 electric vehicles.

Neil, fair enough, but can I also include scenario 4?

A downsized ICE-vehicle like the Loremo (thanks to Michael Murphy, who pasted this link in the comments at Low-tech Magazine):

http://www.loremo.com/englisch/02der02_varianten.htm
Engine/Power: 2 cyl turbo-diesel /28 HP
Top speed:106 mph
0-63mph: 20 seconds
Mileage Rating: 120 mpg
Weight: 1.000 lbs
Price: 17,000 euros

Good luck designing an electric or hybrid that does better than that, if you also take into account the embodied energy of batteries & charging infrastructure. And it looks more like a Tesla Roadster than an Aptera: http://www.loremo.com/

Sounds a little like the Nash Metropolitan I learned to drive on. Except for that mileage! And who knew that the Metro was overpowered at 50 HP?

Engine: 4 cyl 1500 cc / ~50 HP
Top speed: 75 mph?
0-60: 25 seconds
Mileage Rating: 27-30 mpg
Weight: 1,785 lbs

Price: ... sold to a buddy for $100 in 1969.

Nash Metropolitan, there is an idea body for an electric car project!
When I was in the auto service business back in the 1970's, our local AAA representative had one, drove it all over central KY...and it was 15 years old with over 100,000 miles on it and still looked great (cute and adorable would be better words actually :-)

RC

It certainly looks like a good idea. I wish it well.

I have to admit I like the idea of an EV, even a partial EV like a Volt, just because I like the feeling of not using any oil at all1.

It has to be said that an EV like the Volt is a lot of fun to drive - it will be a much, much easier thing to sell.

1Embodied energy can come from renewable electricity - liquid fuel is a lot harder to make low-impact.

Kris,
Good luck designing an electric or hybrid that does better than that, if you also take into account the embodied energy of batteries & charging infrastructure.

I think the Chevy Volt has a 180mpg rating, assuming that only 8kWh is available form a 16kWh battery( a very conservative estimate to allow for 10year battery performance).

Nick has discussed battery energy( about 2% of lifetime energy use?)

Charging infrastructure?? Already exists, for overnight charging.

Nick has discussed battery energy( about 2% of lifetime energy use?)

I thought we all agreed that we had no clue and that we badly need an LCA of a battery.

Charging infrastructure?? Already exists, for overnight charging.

Then why are so many cities investing in charging posts for daytime charging?

I have an update, from correspondence with the Center for Transportation Research, Argonne National Laboratory.

The correct way to calculate the embodied energy of the battery is to multiply the 109.7 KJ per km (30 watt-hours per km) by lifetime range of about 250K km. That gives 27.4 GJ.

Now, the average US vehicle would use about 859 GJ, and a Volt would use about 190GJ (82 gas, and 108 from electricity), so the battery represents about 3% of an ICE and 15% of a Volt's lifetime consumption.

Thanks for writing them, Nick

But of course the Volt, being a hybrid, has just a tiny battery compared to that of the Tesla - and that's the vehicle we were talking about.

So now we know how to calculate it, we can conclude that the original calculation in the article was correct (if the LCA you refer to is sound of course): it takes almost 30,000 kWh to produce the Tesla Roadster battery. This must be close to 50 percent of a Roadster's lifetime consumption.

Well, I'm not really concerned with the Tesla. I think a 52 kWh battery is unnecessarily large, right now, and so do the people at Tesla - that's not their long-term goal.

I think an EREV like the Volt is the primary "EV solution" for our consumption of oil for personal transportation, for quite some time. Wouldn't you agree that we need to pay the most attention to the best "EV solution"? The Volt's battery requires only about 15% or less of it's lifetime energy consumption.

I would note that we shouldn't convert the joules in the paper to kWh, and then compare to the electricity consumed by the vehicle. Look at the original paper - we're talking single digit grams/km emissions due to battery embedded energy. That's an indication that the battery manufacturing inputs were primary energy, not kWhs, and therefore much less CO2-intensive.

Also, we should really compare the embedded energy of the battery to the lifetime consumption of the status quo: the ICE we're replacing. It's only about 3% of that.

why are so many cities investing in charging posts for daytime charging?

Eliminating range anxiety, which further encourages people to buy EVs.  This eliminates noise and reduces emissions in cities, making them more livable.

Daytime charging can be combined with grid regulation (short-term variation in charging rate), so there are benefits there as well.

Building much more transit and bicycle parking, lanes, showers, etc. while simply reducing the # of cars will have a significantly greater (by an order of a magnitude or two) impact on improving livability in cities.

Quite frankly cars make auto sewers that NO ONE wants to be around.

Best Hopes for Congestion Charges a la London, 25% discount for EVs

Alan

The Volt is appears likely to use about 1 gallon of fuel per 230 miles of overall driving, given typical US driving patterns. Of course, it's not hard to modify driving patterns to reduce that further.

Every time we visit this subject I suggest the idea of standardized, easily changeable batteries and battery stations. Never any response. If batteries were standardized (only a few types for many vehicles) and all of the electronics were contained in the "module", they could be changed out quickly. This would solve many issues: charge time, diagnostics/repair, range, upgrades to technology, etc. There is no reason every manufacturer or model needs a unique battery. So simple, it's stupid. Every ICE car doesn't use a different kind of fuel. Why should every EV require a unique/permanent battery?

The current issue of Homepower has a DIY article on refurbishing the battery in a 2003 Civic Hybrid http://homepower.com/article/?file=HP137_pg84_Lamb (memebership required). It says that new replacement batteries are no longer available, only refurbished battery cells. If car batteries were standardized this wouldn't be a problem. Slide the old one out, slide a new one in.

Ghung, I like that idea, but that leats to the next step...simply lease the battery pack from the power utility. The problem then becomes theirs, they have the economics of scale, they should get the carbon credits IF they are producing clean power (solar, wind, natural gas better than coal) and the risks of unpredicted battery failre could be insured by a pool of money from the utility industry, the car manufacturer and the government...a pool to insure the finincial viability, and since the battery would be paid over the duration of the lease, the big expensive hit of the battery at the front end would distributed over time. As you say, a standardized size battery pack easily exchangable. This really would be the way to do it. Will it happen? The odds are not good, unless draconian carbon caps come into place worldwide.

RC

I was thinking that the oil companies would be a good candidate. Subscribe to BP's battery service. Pull in and a robotic system reads the radio ID from your car or scans a bar code. It removes your old battery and selects a new one that has been diagnosed and fully charged, inserts it and you are on your way. Don't forget your free cup of "Rocket Java"!

Every time we visit this subject I suggest the idea of standardized, easily changeable batteries and battery stations. Never any response.

I've pitched that before and I don't remember you chiming in.

I've also pitched the RUF design http://www.ruf.dk/ in the past - and that doesn't get a whole lotta love either. The gent who was pitching the 1-2 man gondalas didn't get love.

The only way I see a radical change like RUF happening is if there is a magical hot desert temp superconductor developed so that a RUF grid is also the new superconductor grid. And my imagination doesn't think of such superconductors.

The battery issue would go away if only the hearts of a forsaken child could be used as the magic pixy dust to make EE Stor's EESU's work. But you'll note that the 1st reference to EESU in this thread is right now - thus showing how many think EESUs will come into existance.

RUF gets no love because the design has a huge tunnel running through the center of the vehicle's passenger and cargo spaces.  In other words, it's a completely bone-headed concept.  And the gondolas are no use where the track doesn't go.

I like the idea of PRT (eliminating the problem of robbery on transit is a huge plus), and PRT can do things like moving human traffic to mezzanine levels of buildings.  But if you make major screwups in the design, don't expect to get anywhere.

1) Robbery on transit is a non-issue, sorry.

2) PRT is useless gadget bahn that will not work in the real world.

One minor example, much cheaper to rent a PRT than a hotel room for working girls (and boys).

Used condoms left behind will likely be an issue (one of many) with PRT.

Alan

Robbery on transit is a non-issue, sorry.

6.3 major felonies PER DAY on NYC subways in 2008.

Nobody believes you, Alan.

E-P,

I think we need to know how many rides per day this applies to,and how that compares to overall NYC crime rates. Surprisingly enough, my guess is that the NYC subway is rather safer than the streets above.

"the NYPD's John Hall tells the Post crime is "so low that it's getting more and more difficult to keep it there,"

If we count only murder, rape, robbery and aggravated assault as serious felonies, the NYC subway system has about as many as my entire county.  That's awfully high for something used for only a short fraction of the day, where you have almost none of the really dangerous situations like domestic disputes.

If NYC and other cities with public transit had a system of passive ankle bracelets for criminal types and banned them from the buses and subways (and allowed stores and such to ban them at the entrances also), it wouldn't just make transit safer.  It would provide exactly the kind of negative social sanction needed to shift cultural values in the right direction.

If we count only murder, rape, robbery and aggravated assault as serious felonies, the NYC subway system has about as many as my entire county.

Are we sure that's how the NYC statistics are reported? What's the population of your county?

Considering the # of major felonies/day in NYC, and the large ridership, it sounds like riding the subway is no particular risk, UNLIKE DRIVIONG TO WORK !

The number of people killed and seriously injured each day driving (likely to be worse with EVs, greater mass) dwarfs any public health risk of transit.

I know 5 people that have died in auto crashes and over a dozen with life altering injuries from cars. I do not know a SINGLE person that has ever been even robbed on transit.

Alan

likely to be worse with EVs, greater mass

That's a complex physics question: I can think of arguments either way.

What's your theory of how that would work?

Look at the mass of a Prius vs. a gas car of comparable interior volume. It is heavier.

EVs will put as much battery in as they can get away with, and use as skinny a tire as they can as well (worse handling & braking).

One mile range of gasoline weighs MUCH less than one mile range of battery (and we do not drive around with the full weight/tank of gas all the time, while EVs do).

Engineers will search for every chance to reduce structural weight in an EV, much more aggressively than with a car. Shaving safety margins will happen.

I find tires on Prius and Hinda Insight to be unsafe (but I am in the outer <1% of concern here. I have high performance rain tires on 16 x 7.5" rims on my 65 HP M-B 240D :-) Porsche tires on a car that "gathers momentum". But I can outbrake and out corner/evade 99+% of the cars on the road. And that extra margin has been useful a couple of times.

Driving is an inherently dangerous activity and I recognize that and try to minimize my risk. Few VMT coupled with a "safe" car.

Still a Camry has better tire/weight ratio than a Prius last time I looked.

Alan

Look at the mass of a Prius vs. a gas car of comparable interior volume. It is heavier.

A little bit. A Tesla, with a 900 lb battery, is only 350 lbs heavier than the Elise: many other components can be eliminated.

Besides, you still haven't shown why a heavier vehicle will be less safe. Two arguments against: 1) EVs have a lower center of gravity, so that weight gives better handling; and 2) objects with greater kinetic energy in a collision suffer less harm.

EVs will put as much battery in as they can get away with

Not true. The Volt engineers expect to keep the kWh constant, and reduce mass over several vehicle generations.

and use as skinny a tire as they can as well (worse handling & braking)

While I partly agree with you, I'd note that's partly a consumer choice:the Prius, for instance, has different tire options.

Engineers will search for every chance to reduce structural weight in an EV, much more aggressively than with a car. Shaving safety margins will happen.

That's speculative, and based on an out-moded weight-first design philosophy. Regenerative braking makes weight much less important.

I find tires on Prius and Hinda Insight to be unsafe

Yes, I don't like their handling. I'm looking forward to a greater selection of vehicles. The Volt looks much better.

I'm amused by the contrast between your views, and those of the people in this discussion advocating for something closer to bicycle tires.

objects with greater kinetic energy in a collision suffer less harm

The exact opposite is true, with the caveat that if the extra weight is for structural strength (NOT EVs) then that can offset it.

Alan

The exact opposite is true,

If you hit another vehicle, or anything that can move, like a sign or a small tree, then greater mass means that the other object changes speeds more than you do, so it absorbs more of the kinetic energy of the collision.

Further, it's not the kinetic energy absorbed by your vehicle that's most important, it's how fast the occupants decelerate, and that may be independent of the deformation of your vehicle.

The RATIONAL fear is to fear driving, NOT transit !

Alan

http://betterplace.com

Look at the video of their electric taxi project in Tokyo.

Every time we visit this subject I suggest the idea of standardized, easily changeable batteries and battery stations. Never any response.

I like Better Place. It's a perfectly good idea for specific niches, like Israel and Denmark.

OTOH, it has a real infrastructural challenge, which is much larger in the US. I think PHEVs like the plug-in Prius, transitioning to EREVs like the Volt, are really the best solution for the broadest range of places and people.

If there's anything I like to aim sunshine at, it's inside-the-box thinking:

If we want more speed, we have to sacrifice range. If we want more range, we have to sacrifice speed. If we want to keep the (energy) costs of the charging infrastructure within reasonable limits, we have to sacrifice speed or size. The lesson to be learned here, is that we cannot have it all: range, speed and size.

Can't?  Of course we can.  We just can't do it purely with existing batteries.  There's always:

  1. Plug-in hybridization.
  2. Continuous power supply.

There are a number of concepts for the latter (some more workable than others), but just running on rails with power supplied through a conductor (overhead or third rail) obviates the range issue at whatever speed the roadway allows.

Suppose you can only get 40 miles of electric range with a standard battery in a car like the Volt.  People with long commutes can fill the trunk with auxiliary batteries.  For long trips or when you want to carry cargo, you remove the aux battery and buy liquid fuel.  Even if fuel is $7.00/gallon, at 50 MPG you're only spending $0.14/mile.  Relatively small amounts of biofuels can be supplied at prices like that.

The point is, if we act soon, we can manage.

Engineer-Poet,

Or why not plug hybridization with LPG bottles. You don't carry the LPG bottles all the time, but if you get ready to go somewhere, you go by Walmart or a convenience store and pick up a couple and hook them up just like you do for your gas grill! There are convenience stores and Walmarts nation wide to swap out the bottles, just like the traveling RV crowd already does...infrastructure is already built!

Keep thinking outside the box...:-)

RC

You call this "thinking outside the box"?

I call this straightforward engineering thinking. Trying to fix a non-tech issue with more technology.

None of your proposals does better than a downsized ICE vehicle like the Loremo mentioned above.

Filling the trunk with auxiliary batteries will add more weight and thus lower range, by the way.

"You remove the aux battery." How do you do that at home? It will weigh at least 200 pounds.

"Continuous power supply"? Trains: yes. Trolleybuses: yes. The same concept using individual vehicles is just needlessly complicated.

Go read the entire works of engineer poet to grasp the thinking and evolution of thought.

At one time s/he wanted to burn the forests to char and make zinc air batteries.

None of your proposals does better than a downsized ICE vehicle like the Loremo mentioned above.

I didn't realize the LoReMo could be driven without liquid fuel.

Filling the trunk with auxiliary batteries will add more weight and thus lower range, by the way.

Really?  So if I take a Chevy Volt with a 16 kWh battery and put another 160 kg, 16 kWH battery in the trunk, you think the extra 160 kg is going to REDUCE the range despite the doubled capacity?  I'm sorry, such thinking is so disconnected from reality that it can only be called delusional.

If people can add batteries as required to eliminate fuel consumption in their average daily driving, that's what some of them are likely to do.  It pays to do this if you use it every day.  The sustainer engine handles the exceptions.

"You remove the aux battery." How do you do that at home? It will weigh at least 200 pounds.

You make it a bunch of 3 kWh modules of maybe 70 pounds each (figuring 375 lb for the internal pack), that's how.  You buy or rent what you need, drop them into a rack (possibly removable) and plug them in.  If you can't manage this yourself, any mechanic could do it in minutes.

I didn't realize the LoReMo could be driven without liquid fuel.

So what? Is coal better than oil? Are coal ash spills and mountain top removal preferable to oil spills? What matters, is overall *energy* consumption. And yes, also when it concerns renewable energies. I quote tstreet above:

"Yes, we could run EVs on solar and wind, but additional renewable electricity should be focused on replacing electric uses, not satisfying the needs for new uses."

Really? So if I take a Chevy Volt with a 16 kWh battery and put another 160 kg, 16 kWH battery in the trunk, you think the extra 160 kg is going to REDUCE the range despite the doubled capacity? I'm sorry, such thinking is so disconnected from reality that it can only be called delusional.

Such thinking is very simply called "a mistake". There is no need to get poetic. What I meant is that you will lower the overall efficiency. Adding more and more batteries will give you a better range, but with every extra pound the gain will be smaller. Your car will become less fuel efficient.

You make it a bunch of 3 kWh modules of maybe 70 pounds each (figuring 375 lb for the internal pack), that's how

That's still a lot of weight for many people, and it is far from practical. And most people don't have a mechanic in their garage.

Is coal better than oil? Are coal ash spills and mountain top removal preferable to oil spills?

Whenever the subject of EVs comes up, EVERY TIME someone starts with the "COAL IS BAD!" chant as if no EV can run on anything else.

Well, I'll tell you something.  Mountain-top removal is more permanent than oil spills, so I'd say it's worse.  But coal ash spills are far more limited in extent, so they're probably better.  And the elimination of risk from foreign suppliers of oil makes the coal-powered EV the lesser of two evils.

But the EV can just as easily be powered by a CCGT:  a fuel with half the GHG/BTU as coal, burned at 70% greater efficiency.  It can also be powered by hydro, wind, solar or nuclear.  It doesn't care.  PV electricity at 25¢/kWh in the typical EV costs about as much per mile as gasoline at $3.00/gallon in the typical car; wind is a small fraction of that.  I note that you don't mention those options, because it makes your argument look weak.

That's still a lot of weight for many people, and it is far from practical. And most people don't have a mechanic in their garage.

So you go to the mechanic and they do it for a few bucks.  Or you go to the place where you leased the extra batteries in the first place, and they swap out the ones getting toward end-of-life and give you new ones when you come back.  None of this is rocket science.

If you want to do it at home, engine stands and exercise machinery suggests ways to move batteries between a car's trunk and a storage rack.  This is an undergrad-level exercise in mechanical design.  I'm not a mechanical engineer, and I could do it.

Spurious objections to trivial aspects of such an effort suggest that you have no substantive criticism.  This means it's time to take on the job, and damn the torpedoes.

But this comment wouldn't be complete without juxtaposing your objections with some quotes from the article itself:

What I meant is that you will lower the overall efficiency. Adding more and more batteries will give you a better range, but with every extra pound the gain will be smaller.

But in the post you said,

Electric motors are (generally) most efficient around 75 percent of their rated load. Their efficiency drops dramatically below 25 percent.

So the losses from hauling more battery weight (perhaps 10% to double the Volt's capacity) are offset by greater efficiency in the motor at greater power, as well as smaller battery losses due to lower per-cell currents during both acceleration and regeneration.  Further, aerodynamic losses will be unaffected.  If there's a 3% penalty from a 10% weight increase, the Volt's projected AER would fall from 80 miles to... 77 miles.

This doesn't look crippling to me.  It looks like an EREV with a "pick-your-battery" option (double the Volt's 16 kWh, for some people, perhaps half as much for others) would be a great way to eliminate the vast bulk of liquid fuel demand.  Even outright absence of fuel would no longer be a crisis, just a nuisance to be planned around.

So you go to the mechanic and they do it for a few bucks.

You *walk* to the mechanic, or you *drive* to the mechanic? In the latter case, there goes (part of) your extra range.

If you want to do it at home, engine stands and exercise machinery suggests ways to move batteries between a car's trunk and a storage rack. This is an undergrad-level exercise in mechanical design. I'm not a mechanical engineer, and I could do it.

I will pass the instructions to my mom and my grandmother. And to my friends who do not own a garage and park their cars on the street.

It might help if you would stop looking at the world exclusively from your own viewpoint.

you *drive* to the mechanic?

Yes.  If you're going to be running on extended range (fuel) anyway, what's a few miles?  It's down in the noise.

It might help if you would stop looking at the world exclusively from your own viewpoint.

Why?  There are only a few people in the world who can design an iPhone, but there's at least a billion who can use one.  I'm sure I can design something that almost anyone can use, and that's what counts.

You call this "thinking outside the box"? - I call this straightforward engineering thinking.

You can think of that way, but for some reason it seems to be very hard for a lot of people to envision a world filled with EVs, a world where because of EVs (and other substitutes) oil has become obsolete for almost everything.

The transition to such a world in large part via HEVs, PHEVs, and EREVs seems obvious to me, but it seems to be very hard for many to imagine.

Filling the trunk with auxiliary batteries will add more weight and thus lower range, by the way.

Keep in mind that regenerative braking mostly eliminates the importance of weight for fuel efficiency. It's a new paradigm for vehicle designers, where weight is far less important, and aerodynamics is most important, followed by drive train friction, rolling resistance and parasitic loads, in roughly that order.

Lots of disinformation out there on evs. My daily driver is a Suzuki Swift I converted to electric. It has LiFePO4 batteries. Top speed 90 mph, range at 30 mph is 100 miles, at 60 mph 55 miles. About 6 months/2400 miles to date, no problems. All energy for it comes from solar panels on the roof of the house. It uses about 200 Wh/mile, which would be 2 cents/mile at 10 cents/kWh. I just plug it in when I return home for the day and its full in 2 to 4 hours, depending on how much the batteries have been discharged. The early evs like the one shown in the photo in the article had about 15 mile range. I agree that evs won't meet the expectations of most people in the U.S. But then, I expect they won't have a lot of choice in 20 years or so.

Interesting anecdote. But then what exactly is the disinformation you are talking about?

Oh, I forgot to mention. The cells ("batteries") I use have a projected life of 3000 cycles, which is around 10 years at my usage. Purchase price was $344.00/Wh.

Very interesting. Do you have a blog where you detailed how you built it?

I think this article sets up a straw man and then puts a lot of effort into knocking it down. We don't need an EV with the range of a gasoline car. We need an EV with a 40 mile range with a small, efficient gasoline generator to recharge the battery on the fly for extended range. That is exactly what a chevy Volt is. The first version will be expensive but the price of Li ion battery will come down as technology improves and mass production kicks in.

This is already happening. The cost has come down from US$1000/kWHr to about $450 in 3 years. It is expected to drop to $120 within 10 years.

Within 10 years, it will be cheaper to make and maintain a serial hybrid EV than a gasoline car.

It is not me setting up a straw man. It is the public, the electric car companies, the battery manufacturers and the government. Everybody talks about (and invests in) nanotech batteries with fabulous energy densities, batteries that can be charged in 2 minutes, and batteries that can be automatically swapped in stations everywhere. So obviously, the need for an EV with the range of a gasoline car is there.

And is it a straw man? A short range has a serious risk, as Paul noted above:
http://www.theoildrum.com/node/6480#comment-626138

It is not me setting up a straw man.

You are the one who wrote the article about range being a major, insurmountable shorcoming of EV cars. That is what I am responding to. My point is very simple: we don't need EVs with 300 mile range. Forty mile range is good enough as long as you have a generator onboard to charge the batteries when they run down.

And is it a straw man? A short range has a serious risk, as Paul noted above

Short range is a risk only without a generator backup as Paul clearly notes in his comment.

So do you agree that a car like Chevy Volt can reduce gasoline consumption by 80%-90% and therefore in the near future we don't need EVs with 300 mile range?

My point is very simple: we don't need EVs with 300 mile range. Forty mile range is good enough as long as you have a generator onboard to charge the batteries when they run down.

Sorry, but then we are not talking about EV's anymore. My article deals with EV's, purely electric cars. The Nissan Leaf, The Mitsubishi i-MiEV, the Fritchle Model A Victoria. You use the on-board generator as a deus ex machina.

I don't believe that the Chevy Volt is the solution. It is too big, too heavy, too fast, and it will have (with two engines and a battery) a high embodied energy. As I said before, you could reach the same reduction in gasoline consumption by radically downsizing the ICE vehicle, and this without raising the embodied energy of the vehicle.

Sorry, but then we are not talking about EV's anymore. My article deals with EV's, purely electric cars. The Nissan Leaf, The Mitsubishi i-MiEV, the Fritchle Model A Victoria. You use the on-board generator as a deus ex machina.

OK then I misunderstood the scope of your article. I agree that pure EVs with 300 mile range with all accessories turned on are not yet practical.

I don't believe that the Chevy Volt is the solution. It is too big, too heavy, too fast, and it will have (with two engines and a battery) a high embodied energy.

Volt will have one small, fuel efficient ICE and an electric motor. Too big, too fast, too heavy for whom? Sounds like your opposition to it is philosophical. The Volt, like any other product, will succeed or fail in the market on its own merit. When the battery costs come down, serial electric hybrids will be more affordable than comparable gasoline cars. In the meantime, let the ones who can afford the "embodied energy" buy it. They will do the rest of a favor by consuming less gasoline?

As I said before, you could reach the same reduction in gasoline consumption by radically downsizing the ICE vehicle, and this without raising the embodied energy of the vehicle.

Yes, but eventually there will be no more gasoline. Serial electric hybrids like Volt are the bridge technology which allows us to transition to pure EVs while waiting for the battery technology to catch up.

Serial electric hybrids like Volt are the bridge technology which allows us to transition to pure EVs while waiting for the battery technology to catch up.

Downsized ICE vehicles could be a similar bridge technology, no?

And the downsized ICE's have the advantage that they get people and society into small cars and away from the large car mindset, which the Volt is attempting to preserve.
If need be, to re-power a Loremo with electric could probably be done, with reasonable performance. To turn a Volt into an all electric is less of a proposition, as it is too big and heavy.
Get the vehicle platform right - small, light and efficient, and more possibilities open up - you could change powertrains without junking the vehicles.
Only then can it be truly called a bridge solution.

People who buy cars like Volt and Leaf will subsidize battery research which will eventually make long range EVs practical and affordable. Either way I am not against small ICE cars. People should have the option of buying them if that is what they want. What I don't understand is the opposition to cars like Volt and Leaf. Let people who want them, buy them. Your opposition to them seems mainly philosophical.

Your opposition to them seems mainly philosophical.

You mentioned this before. I wonder what you mean by it.

Actually, it's the people that *don't* buy these vehicles that are subsidising the ones that do. There is a $7k tax credit on the volt and something similar on the leaf. Battery research is happening around the world (and has been for decades) without a single EV being sold. The development was driven by mobile electronics, and latterly, hybrids. There are over a million Prius on the road, all with battery packs already - selling a few tens of thousands of these vehicles will not make much difference.

Actually, it's the people that *don't* buy these vehicles that are subsidising the ones that do.

I'd say that it's the people who pay their taxes for the additional $500B in US military spending on oil wars that are subsidizing oil.

I'd say that the tax credit is just a recognition of the external costs of oil,and therefore is just a Pigovian correction to make the cost accounting correct.

Batteries are now 'good enough".

selling a few tens of thousands of these vehicles will not make much difference.

True, which is why I'm looking forward to millions of EREVs. Heck, the Prius is the best selling car in Japan - the future is here.

A good article. Perhaps the battery industry will get a tech boost now from the need and push for better batteries. For now though, i'll stick to the electric bike. Eletric versions only fix one issue with cars as mass transit.

This subject fascinates me, as you can tell by all the late night posts I have done on this string after a hard days work!

The article was very good, and the history of the automobile is a fascination to me. Unlike many here, I openly admit to being an admirer of the automobile, I see it as much art as technology, a story of one of the great technical and artistic expressions of humankind in all of history. Humans could have survived without having invented the thing, but I am glad we did not have to. Thank you for your research effort, which is to be commended.

Some little thoughts: I notice there was not a lot of mention on the two main physical factors of resistance to automobile efficiency other than weight, excessive horsepower expectations and extra parasitic gadgets. These two are the physical realities all cars, ICE or electric must face: Aerodynamics and rolling resistance.

It is hard to believe the vintage electric cars shown were very aerodynamic, but in truth, the new electrics are not a lot better!

So we know the vintage electric cars suffered on the issue of aerodynamics, newer ones must be better because a modern car is normally designed lower to the road, and an enclosed car is generally more aerodynamic than an open one. Some say because the speed was low, aerodynamic doesn't matter, but that is a mis-perception. The aerodynamic penelty is the same if you double the speed, speed squares, aero drag cubes...so a car at 30 miles per hour is facing a much stiffer penelty than one at 15 miles per hour. We don't consider the low speed aerodynamic issue today because we have such an excess of horsepower, but as we know, enclosed steamlined bicycles are much faster than open riders sitting upright...even though the top speeds are low compared to automobiles.

An issue not mentioned at all in your article is rolling resistance. Notice the width of the tires on the vintage electrics...they are not much wider in tread facing the road than a bicycle or motorcycle tire!

The penelty of rolling resistance is no small matter, and as tires get wider, the tread facing the road creates ever increasing drag:

The rolling resistance (coefficient of friction) of a bicycle tire at 30 miles per hour range between 0.0022 to 0.005 (this from an article by bike tech review as cited by Wikipedia), while a car tire on pavement is given as 0.010 to 0.015 (cited from Wikipedia with sources given) This would be a magniture difference of 10 or more, depending on tire design. The extra width of modern tires cost a lot of efficiency, but are demanded for safety, comfort, and handling performance. Lower speeds would naturally reduce the need for such wide tires, some of the modern high performance cars have tires that look more like rollers, with just a small gap between them for the differential.

Another issue: With regenerative braking, there is no real need for massive brakes of the conventional type...the electric resistance can be used for braking. The most advanced of the turn of the century electrics was the Lohner Porsche, developed by Dr. Ferdinand Porsche, the founder of the auto company bearing his name, and the designer of the Volkswagen Beetle. Astounding this car was designed in 1901, when Porsche was 25 years old!
http://cache.jalopnik.com/assets/resources/2008/04/Lohner-Porsche-Electr...

This is a hybrid electric, with a gasoline engine charging batteries, which then powered the two hub motors seen at the front wheels. Notice there are no visible brakes, as the electric motors could be switched to resistance to stop the car.

The point is that the potential of the modern electric car is still relatively undeveloped despite these early efforts, because the materials and controls simply did not exist in those days, even though the ideas did. After the mass production of the Model T and the cost cutting, most development on the electric car simply stopped. The potential is huge, BUT...if fuel prices do not rise dramatically, there is only one incentive to push electric car development, and that is carbon release reduction. This may soon be mandated. If the current models of carbon release and resulting climate change are accepted, and the recommendations by the climatologists are taken seriously, it will mean the de facto outlawing of fossil fuel transportation. Electric vehicles would become essentially the only legal path forward. The carbon issue/climate issue are becoming the driver for forced change. Below, some aerodynamic ideas, and a fun little runabout that could work as an electric car of considerable efficiency:

For aerodynamic excellence, scroll about half down, the grey streamline prototype,the Mercedes Benz C111-111, with turbo Diesel engine it could achieve 21 miles per gallon at over 150mph for hours on end:
http://www.germancarforum.com/mercedes-benz-lounge/25993-mercedes-benz-m...

A small prototype idea, as cute as a 2CV, the electric sportscar in the mold of an Austin Healey Sprite from Geneva Switzerland!
http://www.youtube.com/watch?v=voI0gdmKWEY&feature=related

RC

An issue not mentioned at all in your article is rolling resistance. Notice the width of the tires on the vintage electrics...they are not much wider in tread facing the road than a bicycle or motorcycle tire!

That's a very good point. It is remarkable that 100 years ago there was a lot of talk about tires. In sales brochures and books the mentioned range was often accompanied by a note saying that it might be influenced considerably by the type of tires used. Correct me if I am wrong, but how I understand it is that rolling resistance is relatively more important than aerodynamics at low speeds, while things are the other way around at high speeds. Of course, at the high speeds we are used to now, bicycle tires would probably not be an option.

You raise many other good points, thanks for the thoughtful comment.

"Correct me if I am wrong, but how I understand it is that rolling resistance is relatively more important than aerodynamics at low speeds, while things are the other way around at high speeds."

Yes, that is exactly correct. So on a lower speed vehicle, the rolling resistance is a factor to be considered.

I also agree that given performance expectations and weight, bicycle tires are out of the question. But I do not dismiss the idea of motorcycle tires as an alternative...and now we are back to something along the lines of the trike concepts shown above. Thanks again for a fascinating article, and for having the nerve to discuss cars at all on this site...not really a friendly place for car discussions in general.

RC

The technology is here for very low rolling resistance car tyres. The stiff sidewalls of the "run flat" tyres are a part of that. The problem is indeed that the tyres are always too wide - and this is to get better "handling" which really means, as Kris has said, to be able to drive them like race cars.
if you are designing a car for modest speeds and driving, then narrow, medium profile tyres are just fine, say 155/75.

The rounded profile of a motorbike tyre is not needed unless the tires are being angled,

While the formulas hold at any speeds, aerodynamic resistance becomes a smaller and smaller part of the overall energy budget as speeds are reduced.

By the time you get down to twenty five to thirty mph , you have mostly exhausted the possibilities of saving energy with better aero if the car in question is initially reasonably areodynamically efficient.

Any additional substantial savings must come thru wieght reduction, lowered rolling resistance,more efficient driveline components etc;basically this means downsizing the car.

Any additional substantial savings must come thru wieght reduction

Regenerative braking mostly eliminates the importance of weight for fuel efficiency. It's a new paradigm for vehicle designers, where weight is far less important, and aerodynamics is most important, followed by drive train friction, rolling resistance and parasitic loads, in roughly that order.

The three main factors are rolling resistance, drive-train friction, and aerodynamics.

Rolling resistance and drive-train friction dominate at low speeds. There was a nice discussion and graphic showing how they relate on one of the Tesla blogs.

Here's my 2 cents on the findings of this article.

First, I will say a lot of the reason why battery range today is still 100 miles has to do with the changes that have happened to normal ICE vehicles. Not that long ago, I read an article than the Ford Model T (1908) got 20-something mpg, which is the same as the average fuel economy of the US fleet today. The reason is safety/feature/speed improvements have drastically increased the amount of work needed to be done to travel a mile. If you look at absolute efficiency (work done for energy input), modern cars are much better, but looking at mpg, they are not. So saying EVs have not improved in 100 years is a lot like saying CARS themselves have not improved in 100 years (which a lot of people would disagree with).

Also you have to examine the history of batteries too. The recent boom in rechargeable (aka storage) battery technology only occurred after the 70s-80s, with the introduction of cell phones and laptops. The storage battery market had pretty much been dominated by lead acid or NiCd at the time (NiCd introduced in 50s, major improvements like Nimh and Li-ion came in the 90s).

Cost is another factor. If you adjust the price of the 100 mile vehicles from that time period using inflation, I think you will find cars like the Leaf to be a lot cheaper, even though it has a lot more content. The 100 mile battery is largely a cost saving maneuver, since doubling the capacity will also double the cost of the pack, which would put the price out of the affordable range (adjusting for lower energy costs).

I have been busy with the oil spill and missed most of this.

I do not think EV's are worthy of governmental or social support, although they are certainly better than ICEs. Long term, I would hope that the market supplies 50 to 100 million or so to replace ICEs.

EVs do nothing to reduce auto deaths & disabilities. They do nothing to reduce obesity. Most importantly, they do nothing to reduce the enormous energy costs of maintaining Suburbia.

The concept that building EVs is somehow cheaper than Urban Rail and Transit Orientated Development is flawed. It fails to consider the life cycle energy and maintenance costs of Suburbia. EVs last, say, 15 years. A well built, efficient multi-story condo can last centuries.

We have TEN TIMES the retail space/capita of 1950. Quite frankly, no need for that much and quite an energy and resource sink.

Modern Suburban housing was recently built to last just 20 years before major maintenance (30 years was the older norm). Many times more street ft2, miles of water, sewer & electrical lines/capita in Suburbia. All in need of upkeep (and clearing snow, etc.)

Why devote resources to repair of over sized, wrong place, energy inefficient white elephant buildings ? And their supporting infrastructure ?

Forests will continue to grow trees regardless (and newsprint demand is way down and likely to stay down), so the most basic raw material for building TOD will not be lacking. And an energy efficient TOD three-plex or four-plex can be built with the just slightly more materials than one McMansion takes.

Why waste money on replacing ICEs with EVs when reducing VMT and the total # of cars is a better goal ?

Best Hopes,

Alan

I do not think EV's are worthy of governmental or social support

In theory, we should eliminate all subsidies, and institute carbon (and other pollution) taxes, as well as a US military cost recapture from oil. That would make both EVs and rail much more popular.

they do nothing to reduce the enormous energy costs of maintaining Suburbia.

The average suburban home is more energy efficient than the average urban dwelling.

EVs last, say, 15 years

That's highly unrealistic. EVs will last as long as we want them to. We throw away light vehicles far before their functional life is over, but we don't have to.

A well built, efficient multi-story condo can last centuries.

And the same is true for any dwelling, whether single family or condo. Heck, there are US homes built in 1670 that are still in use.

Modern Suburban housing was recently built to last just 20 years before major maintenance (30 years was the older norm).

Major maintenance? Roof maintenance, and painting of siding, perhaps? There's no question that suburban homes have more maintenance, but you're exaggerating a bit: there's nothing there that suggests that single family homes suddenly slide into the river at 20 years.

Many times more street ft2, miles of water, sewer & electrical lines/capita in Suburbia. All in need of upkeep (and clearing snow, etc.)

Somewhat more. OTOH, overall urban living costs are much, much higher. Higher FF costs won't change that.

Why waste money on replacing ICEs with EVs when reducing VMT and the total # of cars is a better goal ?

Urban living and rail transportation are good, but not especially for energy reasons. That's a whole different question.

Electric motors are (generally) most efficient around 75 percent of their rated load. Their efficiency drops dramatically below 25 percent.

This isn't generally true. The source you quote was talking about line-powered induction motors. Electric cars commonly use permanent-magnet synchronous motors. Although obviously the efficiency is still 0 at 0 load, the fall off is sharp enough that extra weight and cost is the main inefficiency of light loads.

The reason induction motors become inefficient at low loads is that rotor current falls as the slip drops near zero. This induces less voltage in the stator. If the stator voltage is constant, the difference is made up with reactive voltage. The power factor drops and the stator resistance continues to eat up real power. Drop the voltage (like with a motor controller) or increase the rotor flux (like in a synchronous motor) and the problem goes away.

And I just noticed this in the article:

Weight, comfort, speed and performance have eaten up any real progress

I'd say that comfort and speed are "real progress", and to the extent that weight and performance contribute to safety, they are too.

Face it, a car which can cover 100 miles at 70 MPH between charges has a lot more utility than one which can cover 100 miles at 17 MPH between charges.  Slashing the time required to travel by a factor of 4, after which you can do something else, is no small achievement.  Neither is a car you can sit in comfortably for an entire journey instead of having to take time out to recover one or more times en route.